Download as pdf or txt
Download as pdf or txt
You are on page 1of 126

UPSC ANALYSIS DOCUMENT FOR PRELIMS

ENVIRONMENT ANALYSIS
HOW TO READ THE MOST ACCEPTED BOOK SHANKAR IAS
FOR ENVIRONMENT IN CONSOLIDATION
BASED ON UPSC PYQ 2011-2023
(PART OF OUR PRELIMS MARATHON)
BY
ER NEELESH KUMAR SINGH
ALL INDIA RANK 442 UPSC CSE 2021
(UNDERSTANDING UPSC DEMAND THROUGH PYQ 2011-2023)
JOIN ME AT TELEGRAM – UPSC PRELIMS WITH NEELESH
TELEGRAM - UPSC PRELIMS WITH NEELESH
Learn ETHICS, ESSAY, SOCIOLOGY, CSAT BY VISITING THE CHANNEL (INTEGRATED MARATHON FOR 2024)
https://t.me/UPSCPrelimsWithNeelesh ©Copyright For Queries Whatsapp - 9310161970
INDEX FOR SHANKAR IAS (YOU MUST READ THESE CHAPTERS TOGETHER WITH PYQ 2011-23)
TOPIC CLUBBED SECTION FOR CONSOLIDATED PREPARATION Number of
NUMBER Questions
TOPIC 1 Ecology And Environment (Ch 1 - Ecology, Ch 2 - Functions of Ecosystem, Ch 3- Terrestrial Ecosystem, Ch 4 - Aquatic 26
Ecosystem)

TOPIC 2 Pollution (Ch 5 – Environmental Pollution, Ch 6 – Renewable Energy, Ch 7 - Environmental Issues, Ch 8 – 28


Environmental Impact Assessment, Ch 29 – Environment Issues and Health Effects) (Appendix – List of Key Activities
and Likely Associated Air Pollutants, Respiratory Irritants and Toxic Chemicals)

TOPIC 3 Biodiversity (Ch 9 – Biodiversity, Ch 10 – Indian Biodiversity Diverse Landscape, Ch 11 – Scheduled Animals of WPA 37
1972, Ch 12 – Animal Diversity of India, Ch 13 – Plant Diversity of India, Ch 14 – Marine Organism)

TOPIC 4 Protected Area Network, Conservation Efforts Etc. (Ch 15, 16, Appendix (Ramsar Wetland, Tiger Reserve, Elephant 25
Reserve, Mike Sites, Biosphere Reserve, List of India’s Biosphere Reserve in UNESCO WNBR, Natural World Heritage
Sites, Biodiversity Heritage Sites, Sacred Groves, Mangroves Sites in India)

TOPIC 5 Climate Change (Ch 17 - Climate Change, Ch 18 – Ocean Acidification, Ch 19 – Ocean Depletion, Ch 20 – Impact of 18
Climate Change – India, Ch 21 – Mitigation Strategies)

TOPIC 6 India And Climate Change (Ch 22 – India and Climate Change, Ch 25 – Acts and Policies, Ch 26 – Institution and 22
Measures, Ch 27 – Environmental Organizations)

TOPIC 7 Agriculture (Ch 24 – Agriculture) 3

TOPIC 8 World Organizations (Ch 23 – Climate Change Organizations, Ch 28 – International Organizations and Conventions) 32

TELEGRAM - UPSC PRELIMS WITH NEELESH


Learn ETHICS, ESSAY, SOCIOLOGY, CSAT BY VISITING THE CHANNEL (INTEGRATED MARATHON FOR 2024)
https://t.me/UPSCPrelimsWithNeelesh ©Copyright For Queries Whatsapp - 9310161970
ECOLOGY AND ENVIRONMENT ANALYSIS (BASED ON PREVIOUS YEAR QUESTIONS)
SIMPLY FOLLOW THE STEP-BY-STEP PROCEDURE TO COVER ENVIRONMENT PORTION FOR UPSC
Simply follow the step-by-step procedure
1. Before reading any chapter, first read the Previous Year Question asked from the chapter. At this stage, do not see the
solution for maximum benefit
2. Then, read the Instructions related to the chapter before reading the actual chapter which has been prepared by me based
on the analysis of 2011-2023 UPSC Question. You must read the chapter as per the instruction
3. Now read the actual chapter from the book (SHANKAR IAS 9TH EDITION or any source which has these chapters) keeping in
mind the PYQ questions and my instructions
4. Come back to the original questions again and then solve the question.
5. See the solution and see how many you solved correctly.
6. Work on your weak areas if any in the chapter as per your attempt
7. Congratulations, now you are the master of this chapter
8. Please note – When I say, some part is important, it does not mean that you should completely ignore other things in the
chapter. It is just that you must not leave those topics/concepts which are mentioned in the instruction specially.

TELEGRAM - UPSC PRELIMS WITH NEELESH


Learn ETHICS, ESSAY, SOCIOLOGY, CSAT BY VISITING THE CHANNEL (INTEGRATED MARATHON FOR 2024)
https://t.me/UPSCPrelimsWithNeelesh ©Copyright For Queries Whatsapp - 9310161970
TOPIC 1: ECOLOGY AND ENVIRONMENT (Ch 1 - ECOLOGY, Ch 2 - FUNCTIONS OF
ECOSYSTEM, Ch 3- TERRESTRIAL ECOSYSTEM, Ch 4 - AQUATIC ECOSYSTEM)
Total Number of Questions
asked from 2011-2023 26
My Instructions related to GENERAL INSTRUCTION
these chapters This is one of the most important sections of Environment. However, it is the easiest portion too. You must
read this portion diligently. Most of the topics here are extremely important

CHAPTER SPECIFIC INSTRUCTIONS


CHAPTER 1 (ECOLOGY) – ONE OF THE MOST IMPORTANT CHAPTER
1.1 History of ecology – not important
1.2 Environment and its component – Important – You must know biotic vs abiotic
1.3 Levels of organization – very important – also remember the levels – You must know what is difference
between individual, population and community, what is ecosystem and its components, what are autotrophs
and heterotrophs/phagotrophs, primary vs secondary vs tertiary consumers, what are
saprotrophs/decomposers/osmotrophs and their examples, what is ecotone, niche, edge effect, what is
biosphere
This chapter is fully important overall. Know the definitions of different terms used and their basics. You
should read this chapter very carefully

CHAPTER 2 FUNCTION OF AN ECOSYSTEM


2.1 Energy Flow – Important
2.2 Food Chain – Very important -know the type of food chain and their examples.
2.3 Food Web – Very important (know the different between food chain and food web)
2.4 Ecological Pyramid – Very Important (Energy pyramid is always upright) – Know the concept of pyramids
of number, biomass and energy/productivity, look for their examples – you may not need remember each
but have the idea, know what is upright and inverted pyramid

TELEGRAM - UPSC PRELIMS WITH NEELESH


Learn ETHICS, ESSAY, SOCIOLOGY, CSAT BY VISITING THE CHANNEL (INTEGRATED MARATHON FOR 2024)
https://t.me/UPSCPrelimsWithNeelesh ©Copyright For Queries Whatsapp - 9310161970
2.5 Pollutant and Trophic Level – Very Important – you should know what is bioaccumulation and
biomagnification
2.6 Biotic Interaction – very important. Know the difference between mutualism, commensalism,
ammensalism, predation and parasitism. NCERT examples related to them remains very important for UPSC.
2.7 Biogeochemical cycle – very important – know the difference between gaseous cycle and sedimentary
cycle and their examples, try to remember the flow of carbon, nitrogen, phosphorus, sulphur
2.8 Ecological Succession – Important topic – know the difference between primary and secondary
succession and autogenic and allogenic succession

This chapter is fully important. Know the definitions of different terms used and their basics. You should read
it very carefully

CHAPTER 3 TERRESTRIAL ECOSYSTEM


3.1 Tundra – Know the meaning of Tundra – important
3.2 Forest Ecosystem – It has been properly covered in GEOGRAPHY NCERT. You can leave the topic too. Do
not go deep in subdivisions. Simply know what are properties, distribution etc. of Coniferous, deciduous,
evergreen etc.
3.3 Indian Forest Types – Prepare it from Geography NCERT. Over subdivisions are not needed by UPSC
3.4 Deforestation – have a general read with understanding
3.5 Grassland Ecosystem – Subdivisions are not asked for grassland. Rather know the properties, distribution
of grassland, importance etc.
3.6 Desert Ecosystem – Important
3.7 Desertification – Very important
3.8 ISFR latest report is important. Report given is old. Latest report came in 2021 and if 2023 comes, read
that.

CHAPTER 4 AQUATIC ECOSYSTEM


4.1 Aquatic Ecosystem – Know the type of Aquatic Organism, factors limiting productivity etc. This portion
is important
4.2 Lake Ecology – Important topic
TELEGRAM - UPSC PRELIMS WITH NEELESH
Learn ETHICS, ESSAY, SOCIOLOGY, CSAT BY VISITING THE CHANNEL (INTEGRATED MARATHON FOR 2024)
https://t.me/UPSCPrelimsWithNeelesh ©Copyright For Queries Whatsapp - 9310161970
4.3 Eutrophication – Very important topic
4.4 Harmful Algal Blooms – you should know what is algal bloom, what is read tide etc. Important topic
4.5 Wetland Ecosystem – Important topic. Read full
4.6 Centre for Wetland Conservation and Management – Know under which ministry
4.7 Estuary Ecosystem – know what is estuary, characteristics etc. Important topic
4.8 Mangroves – Very important Topic. Read it properly. KNOW THE LOCATIONS TOO
4.9 Coral Reefs – very important topic. Read it full. Location too.
4.10 Government Measures to protect mangrove forest and coral reefs (or coastal ecosystem) – Important topic
4.11 Coastal Ecosystems – know the names of different plans/schemes/organisations etc – they are related to? And if
they are in which ministry. You should know what is blue flag? And associated coasts with states
4.12, 4.13 and 4.14 Ganga Action Plan, Namami Ganga Program and Clean Ganga Fund – All these are important

So, finally as already told – first 4 chapters are very important. So, remain focussed with them.

S.No. Question Answer Solutions


1 Which one of the following is the best description of c An ecosystem can be visualized as a functional unit of nature,
the term 'ecosystem'? (2015) where living organisms interact among themselves and also
a. A community of organisms interacting with one with the surrounding physical environment. This question is the
another based on the definition of the ecosystem.
b. That part of the Earth which is inhabited by living
organisms
c. A community of organisms together with the
environment in which they live
d. The flora and fauna of a geographical area

TELEGRAM - UPSC PRELIMS WITH NEELESH


Learn ETHICS, ESSAY, SOCIOLOGY, CSAT BY VISITING THE CHANNEL (INTEGRATED MARATHON FOR 2024)
https://t.me/UPSCPrelimsWithNeelesh ©Copyright For Queries Whatsapp - 9310161970
2 Which one of the following terms describes not only b Ecotone: It is a transitional zone between two ecosystems.
the physical space occupied by an organism, but also Examples are swamps, marshes and mangroves.
its functional role in the community of organisms? Habitat: Habitat is the physical environment in which an organism
(2013) lives.
a. Ecotone Ecological niche: In nature, many species occupy the same
b. Ecological Niche habitat but they perform different functions. The functional
c. Habitat characteristics of a species in its habitat is referred to as "niche"
d. Home range A niche is unique for a species while many species may share the
same habitat. No two species in a habitat can have the same
niche. Hence, option (b) is the correct answer.
Home range: The area in which an animal forages and spends
most of its time. The size varies according to the size of the animal
and its feeding habits.

3 Which of the following are detritivores? (2021) c Among the given choices, only Earthworms, Millipedes and
1 Earthworms Woodlice are detritivores.
2. Jellyfish Detritivores play a crucial role in the decomposition of organic
3. Millipedes matter. Decomposition is the process of breaking down complex
4. Seahorses organic matter into inorganic substances like carbon dioxide,
5. Woodlice water and nutrients.
Select the correct answer using the code given below:
a. 1,2 and 4 only On land, most detritivores are invertebrate insects such as mites,
b. 2,3,4 and 5 only beetles, butterflies and flies; mollusks such as slugs and snails; or
c. 1,3 and 5 only soil-dwelling earthworms, millipedes and woodlice. ln marine
d. 1,2,3,4 and 5 environments, they are mostly crustaceans or echinoderms such
as crabs and lobsters, echinoderms such as sea stars or sea
cucumbers, living on the ocean floor and sometimes called
"bottom feeders."

TELEGRAM - UPSC PRELIMS WITH NEELESH


Learn ETHICS, ESSAY, SOCIOLOGY, CSAT BY VISITING THE CHANNEL (INTEGRATED MARATHON FOR 2024)
https://t.me/UPSCPrelimsWithNeelesh ©Copyright For Queries Whatsapp - 9310161970
4 With reference to the food chains in ecosystems, b Transfer of food energy from green plants (producers) through a
which of the following kinds of organism is/are series of organisms with repeated eating and being eaten is called
known as decomposer organism/organisms? (2013) a food chain. Each step in the food chain is called trophic level.
1. Virus Decomposers form an important link between the living and
2. Fungi nonliving components of the ecosystem. Decomposers are
3. Bacteria heterotrophic organisms, mainly fungi and bacteria, which meet
their energy and nutrient requirements by degrading dead
Select the correct answer using the codes given below: organic matter or detritus. These are also known as saprotrophs.
a 1 only Decomposers secrete digestive enzymes that break down dead
b. 2 and 3 only and waste materials into simple, inorganic materials, which are
c. 1 and 3 only subsequently absorbed by them.
d. 1, 2 and 3

5 Consider the following statements: (2019) d Statement 1 is correct: Most turtles are omnivores (eat both
1. Some species of turtles are herbivores. plant and animal), but some of them are strictly herbivores (e.g.
2. Some species of fish are herbivores. Green Sea turtles). What a turtle eats depends on its species,
3. Some species of marine mammals are herbivores. where it lives and what food sources are available to it.
4. Some species of snakes are viviparous. Statement 2 is correct: Some species of fish such as Surgeonfish
and parrotfish are herbivores. They feed on reef algae etc.
Which of the statements given above are correct?
(a) 1 and 3 only Statement 3 is correct: Some species of marine mammals are
(b) 2, 3 and 4 only herbivores. E.g., Manatees, also known as seas cows feed on
(c) 2 and 4 only water grass, weeds and algae.
(d) 1,2,3 and 4
Statement 4 is correct: Viviparous snakes are snakes that give
birth to live young ones, while oviparous snakes lay eggs. Around
30 percent of snake species give live birth while 70 percent are
oviparous. Examples of Viviparous snakes: Olive Sea Snakes and
four species of Anacondas etc.

TELEGRAM - UPSC PRELIMS WITH NEELESH


Learn ETHICS, ESSAY, SOCIOLOGY, CSAT BY VISITING THE CHANNEL (INTEGRATED MARATHON FOR 2024)
https://t.me/UPSCPrelimsWithNeelesh ©Copyright For Queries Whatsapp - 9310161970
TRICK – When there are chances, some, possibility,
probability etc – in that case, that option is correct. There is
no water-tight compartment rule generally in environment.

So, when you see the word some- and you are clueless, mark
it as correct.

6 Which of the following have species that can establish d Mutualism is a close association between two species in which
symbiotic relationship with other both the species get benefitted. However, some mutualisms are
organisms? (2021) so intimate that the interacting species can no longer live without
1. Cnidarians each other as they depend totally on each other to survive. Such
2. Fungi close associations are called symbiotic relationships.
3. Protozoa Among the given choices, all three organisms form symbiotic
Select the correct answer using the codes given below: relationships with other organisms.
a. 1 and 2 only
b. 2 and 3 only Cnidarians: The sea anemone, a cnidarian gets attached to the
c. 1 and 3 only shell of hermit crabs for benefit of transport and obtaining new
d. 1,2 and 3 food while the anemone provides camouflage and protection by
means of its stinging cells to the hermit crab

Fungi: Certain fungi live in the roots of trees. The tree provides
nutrients to the fungus and, in return, receives help from it to
take up water and nutrients from the soil. Also, in organisms
called lichens, algae and fungus live together. The fungus provides
shelter, water and minerals to the alga and, in return, the alga
provides food which it prepares by photosynthesis.

Protozoa: Termites can eat wood but have no enzymes to digest


it. However, their intestine contains certain flagellate protists
TELEGRAM - UPSC PRELIMS WITH NEELESH
Learn ETHICS, ESSAY, SOCIOLOGY, CSAT BY VISITING THE CHANNEL (INTEGRATED MARATHON FOR 2024)
https://t.me/UPSCPrelimsWithNeelesh ©Copyright For Queries Whatsapp - 9310161970
(protozoans) that have the necessary enzymes to digest the
cellulose of the wood eaten by termites and convert it into sugar.

7 With reference to food chains in ecosystems, consider a The food chain may be defined as a "succession of organisms in
the following statements: (2013) an ecological community that constitutes a passing on of food
1. A food chain illustrates the order in which a chain energy from one organism to another as each consumes a lower
of organisms feed upon each other member and in turn, is preyed upon by a higher member of the
2. Food chains are found within the populations of a food chain."
species. Statement 1 is correct: The food chain describes who eats whom
3. A food chain illustrates the numbers of each in the wild. Every living being from one-celled algae to giant blue
organism which are eaten by others whales needs food to survive. Each food chain is a possible
Which of the statements given above is/are correct? pathway that energy and nutrients can follow through the
a. 1 only ecosystem.
b. 1 and 2 only Statement 2 is not correct: Food chains describe the feeding
c. 1,2 and 3 relationship between different species, and not within the
d. None populations of one species.
Statement 3 is not correct: Food chain doesn't illustrate the
numbers of each organism that are eaten by others

8 Which one of the following is the correct sequence of a Diatoms are primary producers in the ocean food chain. So,
a food chain? (2014) among the given choices, the food chain will start with Diatoms.
a. Diatoms-Crustaceans-Herrings Crustaceans feed on diatoms and herring is a type of fish which
b. Crustaceans-Diatoms-Herrings feeds on crustaceans. Therefore, the correct order is Diatoms-
c. Diatoms-Herrings-Crustaceans Crustaceans-Herrings.
d. Crustaceans-Herrings-Diatoms

9 Due to some reasons, if there is a huge fall in the c Statement 1 is correct: Butterflies are crucial pollinators and their
population of species of butterflies, what could be its contribution to global agriculture is valued only next to honey
likely consequence/consequences? (2017) bees. Nearly 90 percent of all plants need a pollinator to
TELEGRAM - UPSC PRELIMS WITH NEELESH
Learn ETHICS, ESSAY, SOCIOLOGY, CSAT BY VISITING THE CHANNEL (INTEGRATED MARATHON FOR 2024)
https://t.me/UPSCPrelimsWithNeelesh ©Copyright For Queries Whatsapp - 9310161970
1. Pollination of some plants could be adversely reproduce and as bee populations drop, the role of the butterfly
affected. becomes even more vital. A huge fall in the population of species
2. There could be a drastic increase in the fungal of butterflies would adversely affect the pollination of many
infections of some cultivated plants. plants.
3. lt could lead to a fall in the population of some Statement 2 is not correct: Though butterflies do feed on fungi,
species of wasps, spiders and birds. their primary food source is nectar. So, a huge fall in the
population of species of butterflies cannot lead to a "drastic
Select the correct answer using the code given below: increase" in the fungal infections of some cultivated plants
(a) 1 only Statement 3 is correct: A fall in their population would also lead
(b) 2 and 3 only to a fall in their predator population like the wasps, spiders and
(c) 1 and 3 only birds
(d) 1,2 and 3
TRICK: Extreme words such as
all/any/drastically/enormously/totally etc. are mostly wrong.
So, eliminating option 2, you could reach c.
.
10 Which one of the following is the correct sequence of c Among the given choices, the correct order of ecosystems in
ecosystems in the order of decreasing productivity? order of decreasing productivity is Mangroves > grasslands >
(2013) lakes > oceans.
a. Oceans, lakes, grasslands, mangroves
b. Mangroves, oceans, grasslands, lakes
c. Mangroves, grasslands, lakes, oceans
d. Oceans, mangroves, lakes, grasslands

TELEGRAM - UPSC PRELIMS WITH NEELESH


Learn ETHICS, ESSAY, SOCIOLOGY, CSAT BY VISITING THE CHANNEL (INTEGRATED MARATHON FOR 2024)
https://t.me/UPSCPrelimsWithNeelesh ©Copyright For Queries Whatsapp - 9310161970
11 Which of the following adds/add carbon dioxide to c Carbon dioxide (CO2) is released into the atmosphere by
the carbon cycle on the planet Earth? respiration, burning of fossil fuels for energy, and by
1. Volcanic action decomposition of limestone during the manufacture of cement.
2. Respiration It is also emitted during volcanic eruptions. Decomposition of
3. Photosynthesis carbon matter also releases the Carbon dioxide gas in the
4. Decay of organic matter atmosphere.
Select the correct answer using the code given below: Hence 1, 2 and 4 are the correct statements.
(a) 1 and 3 only Excess of CO2 in the air is removed by green plants. Green plants
(b) 2 only require CO2 for photosynthesis and they, in turn, emit oxygen,
(c) 1,2 and 4 only thus maintaining the delicate balance. Hence, option (c) is the
(d) 1,2,3 and 4 correct answer.

TELEGRAM - UPSC PRELIMS WITH NEELESH


Learn ETHICS, ESSAY, SOCIOLOGY, CSAT BY VISITING THE CHANNEL (INTEGRATED MARATHON FOR 2024)
https://t.me/UPSCPrelimsWithNeelesh ©Copyright For Queries Whatsapp - 9310161970
12 ln case of which one of the following biogeochemical c Phosphorus is a major constituent of biological membranes,
cycles, the weathering of rocks is the main source of nucleic acids and cellular energy transfer systems. Many animals
release of nutrient to enter the cycle? (2021) also need large quantities of this element to make shells, bones
a. Carbon cycle and teeth. Phosphorus is generally found as phosphates in ocean
b Nitrogen cycle sediments or rocks. Much of it gets deposited in the sea and
c. Phosphorus Cycle shallow sediments. The sedimentary rocks are the largest
d. Sulphur Cycle reservoir of phosphorus. The phosphorus cycle begins in
sedimentary rocks where it is released from these deposits by
weathering, leaching and mining. When it rains, phosphates are
removed from the rocks (weathering)and distributed in soil and
water.

CYCLE AND RESERVOIR


ITS TYPE
Oxygen The major reservoir of free oxygen that supports life is
(Gaseous) the atmosphere. In addition, water and carbon dioxide
are also two main reservoirs of oxygen. Ozone is
another
molecular form of oxygen which is present in a very
small quantity in one of the atmospheric layers called
the stratosphere. Oxygen is found as a component in
organic molecules of animals and plants. Further in
rocks of the lithosphere, oxygen is bound into chemical
compounds such as oxides and carbonates.
Undecomposed organic matter in the form of fossil
fuels and carbon in sedimentary rocks also represent
some of the reservoirs holding oxygen.

Nitrogen Nitrogen is tied up in four different types of res ervoirs


(Gaseous) at any time of its cycling. These are the atmosphere ,
soil, water and living organisms. Gaseous nitrogen

TELEGRAM - UPSC PRELIMS WITH NEELESH


Learn ETHICS, ESSAY, SOCIOLOGY, CSAT BY VISITING THE CHANNEL (INTEGRATED MARATHON FOR 2024)
https://t.me/UPSCPrelimsWithNeelesh ©Copyright For Queries Whatsapp - 9310161970
makes up nearly 80% of the earth’s atmosphere by
volume, which is the largest reservoir of nitrogen. The
total amount of fixed nitrogen in the soil, oceans and
the
bodies of organisms is only about 0.03% of that figure.
The atmospheric nitrogen reservoir of molecular
nitrogen is chemically inactive. The aqueous reservoir
of nitrogen is biologically more important to living
organisms and it comprises soil-water and the other
aquatic ecosystems such as oceans. Nitrogen is
present in these aquatic reservoirs in the form of
nitrates (NO3-) or ammonia (NH3 or NH4+) which can be
absorbed by plants and then be incorporated into
plant tissues in a variety of forms such as proteins,
pigments, nucleic acids and vitamins. These nitrogen
containing compounds are then transferred to animals
via food chains. Therefore, organisms are also one of
the main reservoirs of nitrogen.

Sulphur In the sedimentary phase, sulphur is tied up in organic


(sulphur cycle and inorganic deposits and is released by
has both decomposition and weathering, respectively. The
sedimentary pyrite rocks are one such example of a sulphur deposit.
and gaseous As a result, sulphur is made available for the use of
phases) plants. The gaseous phase of the cycle allows the
circulation of sulphur in the atmosphere.

Phosphorus The main reservoirs of phosphorous are rocks and


Cycle natural phosphate deposits. During the geological era,
(Sedimentary) phosphorous gradually accumulated in the ocean
sediments. These sediments, over tens of millions of
years of geological time periods were converted into
sedimentary rocks. These rocks are attacked by
TELEGRAM - UPSC PRELIMS WITH NEELESH
Learn ETHICS, ESSAY, SOCIOLOGY, CSAT BY VISITING THE CHANNEL (INTEGRATED MARATHON FOR 2024)
https://t.me/UPSCPrelimsWithNeelesh ©Copyright For Queries Whatsapp - 9310161970
weathering, leaching and erosion and phosphorus is
released and made available to other organisms.
The phosphorous found in the oceanic sink is
transported to the land as guano deposits when
marine organisms are consumed by birds. The guano
deposits have been exploited by humans as a source
of phosphate rich material which is used as a fertilizer.

13 Which of the following are nitrogen-fixing plants? a Nitrogen fixation is a process that implies the transformation of
(2022) the relatively non-reactive atmospheric N2 into its more reactive
1. Alfalfa compounds (nitrates, nitrites, or ammonia).
2. Amaranth Nitrogen-fixing microorganisms do what crops can't - get
3. Chickpea assimilative Nitrogen lor them.
4. Clover Bacteria take it from the air as a gas and release it to the soil,
5. Purslane (Kulfa) primarily as ammonia. It is the only suitable option for plants
6. Spinach because they can consume Nitrogen only from the soil and only
as nitrogenous inorganic compounds, which explains the
Select the correct answer using the code given below: importance of nitrogen fixation.
(a) 1, 3 and 4 only Nitrogen-fixing plants include: alfalfa, green, peanuts (aka
(b) 1, 3, 5 and 6 only groundnuts), soyabeans, lentils, cowpeas, chickpeas, amaranth,
(c) 2,4,5 and 6 only clovers, kulfa, spinach etc.
(d) 1,2,4, 5 and 6
Know this also (FROM IGNOU)
Nitrogen Fixers in Nature - The ‘Gifted’ Species

Biological nitrogen fixation remains mainly confined to a few distinct


nutritional types of prokaryotes, some of which are free-living while

TELEGRAM - UPSC PRELIMS WITH NEELESH


Learn ETHICS, ESSAY, SOCIOLOGY, CSAT BY VISITING THE CHANNEL (INTEGRATED MARATHON FOR 2024)
https://t.me/UPSCPrelimsWithNeelesh ©Copyright For Queries Whatsapp - 9310161970
others live in symbiotic association with eukaryotic partners. This
process contributes annually about 60 % (nearly 150-190 million
metric tons annually) to the

earth's newly fixed nitrogen budget. Nitrogen fixers have been


variously classified as free living (Azotobacter, Nostoc, Anabaena or
symbiotic (Rhizobium-legume association).

Free-living nitrogen fixers can further be classified into phototrophic


(Nostoc), chemotrophic (Klebsiella), aerobic (Azotobacter) or
anaerobic (Chromatium) depending upon the state of existence and
mode of nutrition. Agronomically, the most important nitrogen
fixing systems are Rhizobium-legume association, Frankia
(actinorhizal)-woody plant association, Anabaena-Azolla and
Cyanobacteria-rice system. Rhizobium-legume association invariably
occurs in the form of root nodules. This group of nitrogen fixers has
been divided into three categories: i) Rhizobium which includes fast
growing species, ii) Bradyrhizobium which include slow growing
strains and iii) Azorhizobium which is a combination of traits from
Rhizobium and Bradyrhizobium.

14 Consider the following statements: (2019) d Statement 1 is correct: The nitrogen oxides originate from
1. Agricultural soils release nitrogen oxides into microbes in the soil that feed on inorganic forms of nitrogen, such
environment. as the ammonium and nitrate found in chemical fertilizers. The
2. Cattle release ammonia into environment. processes that release nitrous oxide include microbial mediated
3. Poultry industry releases reactive nitrogen nitrification in aerobic soils, denitrification of nitrate in anaerobic
compounds environment. soils. Chemical fertilizers (over 82% of it is urea) account for over
Which of the statements given above is/are correct? 77% of all agricultural N2O emissions in India.
(a) 1 and 3 only
(b) 2 and 3 only

TELEGRAM - UPSC PRELIMS WITH NEELESH


Learn ETHICS, ESSAY, SOCIOLOGY, CSAT BY VISITING THE CHANNEL (INTEGRATED MARATHON FOR 2024)
https://t.me/UPSCPrelimsWithNeelesh ©Copyright For Queries Whatsapp - 9310161970
(c) 2 only Statement 2 is correct: Ruminant animals do not efficiently utilize
(d) 1,2 and 3 dietary nitrogen. Excess nitrogen fed in the form of feed proteins
is excreted in manure (urine + feces). Nitrogen in manure can be
converted to ammonia through bacterial degradation, primarily
the conversion of urinary urea to ammonia. Urease, an enzyme
produced by microorganisms in feces, reacts with urinary urea to
form ammonia. Urease activity in feces is high and rapidly
converts urea to ammonia after excretion. Cattle account for 8O%
of the ammonia production. India is globally the biggest source
of ammonia emission.

Statement 3 is correct: Reactive nitrogen species include oxides


of nitrogen [N; nitric oxide, nitrogen dioxide and nitrous oxide],
anions (nitrate and nitrite), and amine derivatives [ammonia,
ammonium salts and urea]. Air emissions from poultry facilities
include ammonia followed by nitrous oxides. Excreta emissions
include ammonia, ammonium ions, and urea with trace amounts
of nitrate and nitrite. The poultry industry with an annual growth
rate of 60%, accorded an excretion of reactive nitrogen
compounds of 0.415 tonnes in 2016.

Hence, option (d) is the correct answer.

15 ln the nature, which of the following is/are most likely c Among the given choices, Lichens and Moss can survive on a
to be found surviving on a surface without soil? surface without soil.
(2021)
1. Fern 2. Lichen 3. Moss 4. Mushroom Lichens: Lichens are symbiotic associations between algae and
Select the correct answer using the code given below: fungi. Algae prepare food for fungi and fungi provide shelter and
a. 1 and 4 only absorb mineral nutrients and water for its partner.
b. 2 only
TELEGRAM - UPSC PRELIMS WITH NEELESH
Learn ETHICS, ESSAY, SOCIOLOGY, CSAT BY VISITING THE CHANNEL (INTEGRATED MARATHON FOR 2024)
https://t.me/UPSCPrelimsWithNeelesh ©Copyright For Queries Whatsapp - 9310161970
c. 2 and 3 only Please note - Lichens are very good pollution indicators - they do
d. 1, 3 and 4 only not grow in polluted areas.

Moss: They are small non- vascular spore- bearing land plants.
They are distributed throughout the world except in salt water
and are commonly found in moist shady locations especially in
woodland and forest floors.

Mosses along with lichens are the first organisms to colonize


rocks and hence, are of great ecological importance. They
decompose rocks making the substrate suitable for the growth of
higher plants. Since moss form dense mats on the soil, they
reduce the impact of falling rain and prevent soil erosion. Hence,
option (c) is the correct answer.

16 Consider the following statements: (2023) c Statement 1 is correct: A thermophile is an organism, a type of
1. Some microorganisms can grow in environments extremophile, that thrives at relatively high temperatures,
with temperature above the boiling point of water. between 41 and 122 °C. Most thermophiles live at temperatures
2. Some microorganisms can grow in environments between 60 and 80 ° C. Thermophiles are capable of growing,
with temperature below the freezing point of water. carrying out metabolic processes, and reproducing at these
3. Some microorganisms can grow in highly acidic extreme temperatures. One thermophile, Pyrolobus fumarii, has
environment with a pH below 3. been found at temperatures of 113° C /235° F, which is hotter
than the boiling point of water.
How many of the above statements are correct?
(a) Only one Statement 2 is correct: Psychrophiles are organisms capable of
(b) Only two growth and reproduction in cold temperatures and are often
(c) All three xerophilic and halophilic as well. Some microbes will cease
(d) None growing at 00C or even higher while others continue to grow
below the freezing point of food. Growth rate below 00C is very
slow. The reported minimum growth of microorganism varying
TELEGRAM - UPSC PRELIMS WITH NEELESH
Learn ETHICS, ESSAY, SOCIOLOGY, CSAT BY VISITING THE CHANNEL (INTEGRATED MARATHON FOR 2024)
https://t.me/UPSCPrelimsWithNeelesh ©Copyright For Queries Whatsapp - 9310161970
from 100C to – 100C. But for practical cases, the lower limit of
growth for bacteria, yeast or moulds can take as -70C.

Statement 3 is correct: Acidophiles are microorganisms that


show optimal growth in highly acidic environments. These are of
two types. The extreme acidophiles dwell in environments with
a pH value <3, and moderate acidophiles grow optimally in
conditions having pH values ranging between 3 and 5.

Trick: Since it is some, and there is a case of possibility, there are


maximum chances that all of the given options are correct. So,
you could have simply mentioned all the options as correct even
without knowing the answers

17 ln the grasslands, trees do not replace the grasses as c The grasslands are the areas which comprise mainly of the
a part of an ecological succession because of: (2013) grasses with a little number of shrubs and trees. Grazing animals,
a. Insects and fungi insectivores and herbivores are the main types of organisms
b. Limited sunlight and paucity of nutrients which are found in these regions. Grassland ecosystems are
c. Water Limits and Fire typically found in tropical or temperate regions where rainfall is
d None of the above not enough to support the growth of trees.
ln grasslands, trees do not replace the grasses as a part of an
ecological succession because trees require more water than
grass. But grassland ecosystems are typically found in tropical or
temperate regions where rainfall is not enough to support the
growth of trees. Also, trees are more prone to catch fire in these
regions as grasslands have high heat intensity. Hence option (c) is
the correct answer.

TELEGRAM - UPSC PRELIMS WITH NEELESH


Learn ETHICS, ESSAY, SOCIOLOGY, CSAT BY VISITING THE CHANNEL (INTEGRATED MARATHON FOR 2024)
https://t.me/UPSCPrelimsWithNeelesh ©Copyright For Queries Whatsapp - 9310161970
18 "Leaf litter decomposes faster than in any other d The above description is of Tropical rainforest. The tropical
biome and as a result the soil surface is often almost rainforest is a hot, moist biome found near Earth's equator. The
bare. Apart from trees, the vegetation is largely high temperature and moisture of tropical rainforests cause dead
composed of plant forms that reach up into the organic matter in the soil to decompose more quickly than other
canopy vicariously, by climbing the trees or growing biomes. Leaves that fall to the forest floor decay quickly, leaving
as epiphytes, rooted on the upper branches of trees". the soil surface almost bare.
This is the most likely description of: (2021)
a. coniferous forest
b. dry deciduous forest
c. mangrove forest
d. tropical rainforest

19 Which of the following Leaf modifications d Adaptation is any attribute of the organism that enables the
occurs/occur in desert areas to inhibit water loss? organism to survive and reproduce in its habitat. Many
(2018) adaptations have evolved over a long evolutionary period and has
1. Hard and waxy leaves become of a permanent nature.
2. Tiny leaves or no leaves In case of desert plants, these adaptations have evolved in such a
3. Thorns instead of leaves way so as to minimize the loss of water.

Select the correct answer using the codes given Adaptations by desert plants:
below: - they have thick cuticle on their leaf surfaces (thick and waxy
a. 1 and 2 only leaves) to reduce loss of water
b. 2 only - their stomata are arranged in deep pits (sunken) to minimize
c. 1 and 3 only water loss through transpiration.
d. 1, 2 and 3 - they have special photosynthetic pathway (CAM) that enables
their stomata to remain closed during day time
- some desert plants like Opuntia, have no leaves - they are
reduced to spines- and the photosynthetic function is taken over
by the flattened stems
- large, fleshy stems to store water
TELEGRAM - UPSC PRELIMS WITH NEELESH
Learn ETHICS, ESSAY, SOCIOLOGY, CSAT BY VISITING THE CHANNEL (INTEGRATED MARATHON FOR 2024)
https://t.me/UPSCPrelimsWithNeelesh ©Copyright For Queries Whatsapp - 9310161970
- thorns and thin, spiky or glossy leaves to reduce water loss
- deep roots to tap groundwater
- long shallow roots which spread over a wide area
- plants lie dormant for years until rain falls
Hence, option (d) is the correct answer

20 There is a concern over the increase in harmful algal d Harmful algal blooms (HABs) occur when colonies of algae-
blooms in the seawaters of India. What could be the simple plants that live in the sea and freshwater-grow out of
causative factors for this phenomenon? (2011) control while producing toxic or harmful effects on people, fish,
1. Discharge of nutrients from the estuaries. shellfish, marine mammals, and birds.
2. Run-off from the land during the monsoon.
3. Upwelling in the seas Statement 1 is correct: Harmful algal blooms are mainly caused
when nutrients (mainly phosphorus, nitrogen, and carbon) from
Select the correct, answer from the codes given below: sources such as lawns and farmlands flow downriver to the sea
(a)1 and 3 and build up at a rate that 'overfeeds' the algae that exist
(b)1 and 2 only normally in the environment.
(c) 2 and 3 only
(d) 1, 2 and 3 Statement2 is correct: Warmer water temperature in the
summer monsoon and excessive nutrients from fertilizers or
sewage brought by runoff, trigger the development of certain
types of harmful algae bloom events making the oceans less
productive.

Statement 3 is correct: Upwelling occurs when deeper, cooler


ocean water is driven up to the surface. Deeper water gets
pushed upward to take its place, and this water tends to be very
nutrient rich, which can trigger an algal bloom.

TELEGRAM - UPSC PRELIMS WITH NEELESH


Learn ETHICS, ESSAY, SOCIOLOGY, CSAT BY VISITING THE CHANNEL (INTEGRATED MARATHON FOR 2024)
https://t.me/UPSCPrelimsWithNeelesh ©Copyright For Queries Whatsapp - 9310161970
21 If rainforests and tropical forests are the lungs of the d Role of kidney: The primary function of the kidney purifies the
Earth, then surely wetlands function as its kidneys." blood and accordingly make urine. Each kidney removes waste
Which one of the following functions of wetlands best materials, and other chemicals which are not required by the
reflects the above statement? (2022) body.
(a) The water cycle in wetlands involves surface runoff,
subsoil percolation and evaporation. Role of aquatic plants: Most freshwater wetlands are rich in
(b) Algae form the nutrient base upon which fish, aquatic plants. Some aquatic plants can also absorb carbon under
crustaceans, mollusks, birds, reptiles and mammals certain conditions and become peat. These plants are also
thrive. capable of absorbing pollutants such as heavy metals and
(c) Wetlands play a vital role in maintaining cleansing the water system. They absorb excess nutrients and
sedimentation balance and soil stabilization. contribute to achieving a vital balance in the system through the
(d) Aquatic plants absorb heavy metals and excess processes of denitrification and detoxification and oxygenation of
nutrients. water. By reading both statements together and comparing the
functions (which are similar) we can logically arrive at a
conclusion that wetlands function as kidneys of the environment.

22 With reference to the wetlands of India, consider the a As per National Wetland Inventory and Assessment 2021, the
following statements: (2012) total wetland area estimated is 15.98 Mha including rivers and
1. The country's total geographical area under the excluding paddy field areas. It comes to be around 4.86 percent
category of wetlands is recorded more in Gujarat as of the total geographic area of the country.
compared to other States.
2. ln India, the total geographical area of coastal Statement 1 is correct: Gujarat (21.9%) has the highest
wetlands is larger than that of inland wetlands. geographical area under the category of wetlands followed by
Maharashtra (7.2%), Andhra Pradesh (7.14%), West Bengal
Which of the statements given above is/are (7.07%), and Uttar Pradesh (6.9%).
correct?
(a) 1 only Statement 2 is not correct: Inland wetlands are more than 12
(b) 2 only times in numbers as compared to coastal wetlands, whereas the
(c) Both 1 and 2 total area of inland wetlands is around three times the area of
(d) Neither I nor 2 coastal wetlands. Inland area covers 3.59 percent of the total
TELEGRAM - UPSC PRELIMS WITH NEELESH
Learn ETHICS, ESSAY, SOCIOLOGY, CSAT BY VISITING THE CHANNEL (INTEGRATED MARATHON FOR 2024)
https://t.me/UPSCPrelimsWithNeelesh ©Copyright For Queries Whatsapp - 9310161970
geographic area of the country, whereas 1.27 percent is covered
by the coastal wetlands.

23 “Biorock technology" is talked about in which one of a Coral reefs are large underwater structures composed of the
the following situations? (2022) skeletons of colonial marine invertebrates called coral.
(a) Restoration of damaged coral reefs
(b) Development of building materials using plant Biorock Technology, which is a method that applies safe, low
residues voltage electrical currents through seawater, causing dissolved
(c) Identification of areas for exploration/ extraction of minerals to crystallize on structures, growing into a white
shale gas limestone similar to that which naturally makes up coral reefs and
(d) Providing salt licks for wild animals in forests/ tropical white sand beaches. It is a unique method that allows
protected areas coral reefs, and other marine ecosystems including seagrass, salt
marsh, mangrove, and oyster reefs to survive and recover from
damage caused by excessive nutrients, climate change, and
physical destruction etc. Hence, option (a) is the correct answer.

24 Consider the following statements: (2018) d Coral reefs are large underwater structures composed of the
1. Most of the world's coral reefs are in tropical skeletons of colonial marine invertebrates called coral.
waters.
2. More than one-third of the world's coral reefs are Statement 1 is correct: The reef building coral is that rely on a
located in the territories of Australia, Indonesia and symbiotic relationship with algae need shallow, clear water
Philippines. allowing light penetration for photosynthesis, therefore, exist in
3. Coral reefs host far more number of animal phyla a band of 30 degrees north to 30 degrees south of the equator.
than those hosted by tropical rainforests. Thus, most coral reefs are found in tropical destinations around
the equator where the water is warmer.
Which of the statements given above is/are correct?
(a) 1 and 2 only Statement 2 is correct: Over half of the world's coral reefs are
(b) 3 only found within six countries: Australia, Indonesia, Philippines,
(c) 1 and 3 only Papua New Guinea, Fiji, and the Maldives.
(d) 1,2 and 3
TELEGRAM - UPSC PRELIMS WITH NEELESH
Learn ETHICS, ESSAY, SOCIOLOGY, CSAT BY VISITING THE CHANNEL (INTEGRATED MARATHON FOR 2024)
https://t.me/UPSCPrelimsWithNeelesh ©Copyright For Queries Whatsapp - 9310161970
Statement 3 is correct: Coral reefs are believed by many to have
the highest biodiversity of any ecosystem on the planet-even
more than a tropical rainforest. Occupying less than 1% of the
ocean floor, coral reefs are home to more than 25% of all marine
life.

25 Government of India encourages the cultivation of c Sea buckthorn may be used as a landscaping shrub with an
'sea buckthorn'. What is the importance of this plant? aggressive basal shoot system used for barrier hedges and
(2012) windbreaks, and to stabilize riverbanks and steep slopes. Hence,
t. It helps in controlling soil erosion and in preventing statement 1 is correct.
desertification:
2. It is a rich source of bio-diesel. There is no evidence to show that it is a rich source of biodiesel.
3. It has nutritional value and is well-adapted to live Hence statement 2 is not correct.
in cold areas of high attitudes.
4. Its timber is of great commercial value. Sea-buckthorn fruits are bitter and sour in taste and have a
delicate aroma, resembling that of pineapple. Sea buckthorn is
Which of the statements given above is/are correct? known for its immunity-boosting and medical properties. Hence,
(a) 1 only statement 3 is correct.
(b) 2, 3 and 4 only
(c) 1 and 3 only Sea buckthorn is a thorny, dioecious shrub. A shrub is not a
(d) 1,2,3 and 4 timber. Hence, statement 4 is not correct.

26 Consider the following kinds of organisms: b Primary producers or Autotrophs synthesize their own energy
a. Copepods without needing to eat. Many photosynthesize, using the sun's
b. Cyanobacteria energy to build carbohydrates. However, some primary producers
c. Diatoms can create energy without sunlight using chemosynthesis to
d. Foraminifera metabolize chemicals released from hydrothermal vents,
Which of the above are primary producers in the food methane seeps, and other geological features. ln terrestrial
chain of oceans? (2021) ecosystems, major producers are plants and photosynthetic
a. 1 and 2 bacteria.
TELEGRAM - UPSC PRELIMS WITH NEELESH
Learn ETHICS, ESSAY, SOCIOLOGY, CSAT BY VISITING THE CHANNEL (INTEGRATED MARATHON FOR 2024)
https://t.me/UPSCPrelimsWithNeelesh ©Copyright For Queries Whatsapp - 9310161970
b. 2 and 3 ln aquatic ecosystems primary producers are various species of:
c. 3 and 4 1. Phytoplankton: Also known as microalgae, they are similar to
d. 1 and 4 terrestrial plants since they contain chlorophyll and require
sunlight in order to live and grow. Most phytoplankton are
buoyant and float in the upper part of the ocean, where sunlight
penetrates the water.
o The two main classes of phytoplankton are dinoflagellates and
diatoms. Dinoflagellates use a whip-like tail, or flagella, to move
through the water, whereas Diatoms rely on ocean currents to
travel through the water.
2 Algae: Algae are chlorophyll-bearing, simple, and largely
aquatic (both freshwater and marine) organisms. They occur in a
variety of other habitats like moist stones, soils and wood.
3 Cyanobacteria: Cyanobacteria, also called blue-green algae, are
microscopic organisms found naturally in all types of water. These
single-celled organisms live in fresh, brackish (combined salt and
freshwater), and marine water. They have chlorophyll similar to
green plants and are photosynthetic autotrophs. Though they are
commonly called blue-green algae, they are not algae.

TELEGRAM - UPSC PRELIMS WITH NEELESH


Learn ETHICS, ESSAY, SOCIOLOGY, CSAT BY VISITING THE CHANNEL (INTEGRATED MARATHON FOR 2024)
https://t.me/UPSCPrelimsWithNeelesh ©Copyright For Queries Whatsapp - 9310161970
TOPIC 2: POLLUTION (Ch 5 – ENVIRONMENTAL POLLUTION, Ch 6 – RENEWABLE ENERGY, Ch 7 - ENVIRONMENTAL
ISSUES, Ch 8 – ENVIRONMENTAL IMPACT ASSESSMENT, Ch 29 – ENVIRONMENT ISSUES AND HEALTH EFFECTS)
(Appendix – List of Key Activities and likely Associated Air Pollutants, Respiratory Irritants and Toxic Chemicals)
Total Number of Questions
asked from 2011-2023
28
My Instructions related to GENERAL INSTRUCTION
these chapters This is one of the most important portions. India as well as world is struggling from the POLLUTION. So, read it
properly
CHAPTER SPECIFIC INSTRUCTIONS

CHAPTER 5 (ENVIRONMENTAL POLLUTION)


This chapter is fully important. You should know about the terms uses, sources of various pollutions, effects
it causes/produce, associated initiatives, remedies etc. Read it full

CHAPTER 6 (RENEWABLE ENERGY)


This chapter is also very important
You may avoid here
1. types of wind turbine
2. types of hydro power station

CHAPTER 7 (ENVIRONMENTAL ISSUES)


Instead of reading this chapter like remembering each detail which is not possible, read it with
understanding. You will be able to do most of the questions if you use your understanding and common
sense e.g. see the recommendation for regulating tourism – don’t remember recommendations – instead
understand it – like all sites should have adequate provisions of garbage disposal and management. This is
common sensical. So, do not try to remember each recommendation. Rather develop an understanding.
You can avoid the policies of local state government etc. e.g., KERELA PROTECTION OF RIVER BANKS AND
REGULATION OF REMOVAL OF SAND ACTS, 2001. It is not needed. Since, it is given for understanding. Do
TELEGRAM - UPSC PRELIMS WITH NEELESH
Learn ETHICS, ESSAY, SOCIOLOGY, CSAT BY VISITING THE CHANNEL (INTEGRATED MARATHON FOR 2024)
https://t.me/UPSCPrelimsWithNeelesh ©Copyright For Queries Whatsapp - 9310161970
not try to memorize everything. We have a limitation of human mind. But any new term which can address
environmental issues must be looked into.

CHAPTER 8 ENVIRONMENTAL IMPACT ASSESSMENT


Know the basics. You need not learn the exact technical details like screening, scoping, baseline date
collection. Think like a generalist and not environmentalist. Have the idea of its basics.

CHAPTER 29 ENVIONMENTAL ISSUES AND HEALTH EFFECTS


This chapter is fully important. Read it full.

1 Consider the following: (2022) b About Acid rain: Normally rainwater has a pH of 5.6 due to the presence of
1. Carbon monoxide H* ions formed by the reaction of rainwater with carbon dioxide present in
2. Nitrogen oxide the atmosphere. When the pH of the rainwater drops below 5.6, it is called
3. Ozone acid rain. Acid rain refers to the ways in which acid from the atmosphere is
4. Sulphur dioxide deposited on the earth's surface.

Excess of which of the above in the environment Acid rain results when sulfur dioxide (SO2) and nitrogen oxides (NOX) are
is/are the cause(s) of acid rain? emitted into the atmosphere and transported by wind and air currents. The
(a) 1,2 and 3 SO2 and NOX react with water, oxygen and other chemicals to form sulfuric
(b) 2 and 4 only and nitric acids. These then mix with water and other materials before
(c) 4 only falling to the ground. Hence, option (b) is the correct answer.
(d) 1,3 and 4

2 Acid rain is caused by the pollution of d Rainwater that has an acidity content greater than natural pH of about 5.6
environment by (2013) is called Acid rain. It is formed when sulfur dioxides and nitrogen oxides, as
(a) Carbon dioxide and nitrogen gasses or fine particles in the atmosphere, combine with water vapor and
(b) Carbon monoxide and carbon dioxide precipitate as sulfuric acid or nitric acid in rain, snow, or fog.
(c) Ozone and carbon dioxide Hence, option (d) is the correct answer.
(d) Nitrous oxide and sulfur dioxide
TELEGRAM - UPSC PRELIMS WITH NEELESH
Learn ETHICS, ESSAY, SOCIOLOGY, CSAT BY VISITING THE CHANNEL (INTEGRATED MARATHON FOR 2024)
https://t.me/UPSCPrelimsWithNeelesh ©Copyright For Queries Whatsapp - 9310161970
3 Consider the following: (2019) d The term 'biomass' defines any material derived from living or recently
1. Carbon monoxide living material, including animal dung, twigs, grass, crop wastes and wood.
2. Methane Biomass material when burned releases extensive varieties of gasses such
3. Ozone as Carbon monoxide, Particulate matter (PM10, PM 2.5), Oxides of nitrogen
4. Sulphur dioxide (NOx) and sulfur (SOx), Carbon Dioxide, Methane, Ozone, volatile and semi-
Which of the above are released into the volatile organic compounds etc. Hence, option (d) is the correct answer.
atmosphere due to the burning of crop/biomass
residue?
(a) 1 and 2 only
(b) 2, 3 and 4 only
(c) 1 and 4 only
(d) 1, 2,3 and 4

4 Photochemical smog is a resultant of the reaction a Photochemical smog is produced when sunlight reacts with oxides of
among (2013) nitrogen and at least one volatile organic compound (VOC) in the
(a) NO2, O3 and peroxyacetyl nitrate in the presence atmosphere. Paints, gasoline and many cleaning solvents release VOCs into
of sunlight. the atmosphere. The chemical formation of air-borne particles and ground-
(b) CO, O3, and peroxyacetyl nitrate in the presence level ozone takes place in the sunlight. General mechanism of formation of
of sunlight. smog. Hence, option (a) is the correct answer.
(c) CO, CO2, and NO3, at low temperature.
(d) High concentration of NO2, O, and CO in the
evening.

5 With reference to furnace oil, consider following d Statement 1 is correct: Fuel oil, also called furnace oil, is a mixture of refined
statements: (2021) crude oil. They are often yellowish/light brown in color and have strong
1. It is a product of oil refineries. odors.
2. Some industries use it to generate power.
3. Its use causes sulfur emissions into Statement 2 is correct: It is mostly used for steam boilers in power plants,
environment. ships and industrial plants.

TELEGRAM - UPSC PRELIMS WITH NEELESH


Learn ETHICS, ESSAY, SOCIOLOGY, CSAT BY VISITING THE CHANNEL (INTEGRATED MARATHON FOR 2024)
https://t.me/UPSCPrelimsWithNeelesh ©Copyright For Queries Whatsapp - 9310161970
Which of the statements given above are correct? Statement 3 is correct: Using petroleum coke and furnace oil contributes to
(a) 1 and 2 Only high SO2 emission and secondary sulfate formation as particulate matter
(b) 2 and 3 Only (PM)
(c) 1 and 3 Only
(d) 1,2 and 3

6 Which of the following are the reasons/factors for a Benzene is an aromatic compound with a single six-member unsaturated
exposure to benzene pollution? (2020) carbon ring. It is a clear, colorless, volatile, highly flammable liquid with a
1. Automobile exhaust characteristic odor and a density. Benzene exposure can occur from both
2. Tobacco smoke indoor and outdoor exposure
3. Wood burning
4. Using varnished wooden furniture o Indoor exposure - Benzene in indoor air sources such as building
5. Using products made of polyurethane materials, attached garages, heating and cooking systems, stored solvents
and various human activities. Wood Burning leads to exposure of Benzene
Select the correct answer using the code given pollution. Human activities like cleaning, painting, the use of consumer
below: products and mosquito repellents, photocopying and printing, the storage
(a) 1,2 and 3 only and use of solvents, and smoking tobacco also increases benzene exposure.
(b) 2 and 4 only
(c) 1, 3 and 4 only Outdoor exposure - Outdoor benzene concentrations are mainly due to
(d) 1 ,2,3, 4 and 5 traffic sources (Automobile exhaust) and are affected by season and
meteorology. Other outdoor sources of benzene are petrol stations and
certain industries such as those concerned with coal, oil, natural gas,
chemicals and steel.

7 Consider the following statements: (2020) d Statement 1 is correct: Coal ash contains contaminants like mercury,
1. Coal ash contains arsenic, lead and mercury. cadmium and arsenic.
2. Coal-fired power plants release Sulfur dioxide Without proper management, these contaminants can pollute waterways,
and oxides of nitrogen into the environment. ground water, drinking water, and the air.
3. High ash content is observed in Indian coal.

TELEGRAM - UPSC PRELIMS WITH NEELESH


Learn ETHICS, ESSAY, SOCIOLOGY, CSAT BY VISITING THE CHANNEL (INTEGRATED MARATHON FOR 2024)
https://t.me/UPSCPrelimsWithNeelesh ©Copyright For Queries Whatsapp - 9310161970
Which of the statements given above is/are Statement 2 is correct: Coal combustion results into release of pollutants
correct? such as Sulphur dioxide, nitrogen oxides, carbon dioxide, fly ash etc.
(a) 1 only
(b) 2 and 3 only Statement 3 is correct: Indian Coal has comparatively higher ash content
(c) 3 only (25 to 45 percent) than imported coal due to drift theory of formation (it
(d) 1, 2 and 3 states that the plant material was transported with the stream of water from
one place to another, and finally get deposited in a place of swamp of coal
deposits in India. Coal formed due to drift theory contain higher ash as
compared to in-situ theory of formation.

8 Consider the following statements regarding c Statement 1 is correct: Artisanal gold mining currently contributes more
mercury pollution: (2023) than 35 percent of all global mercury emissions created by people.
1. Gold mining activity is a source of mercury
pollution in the world. Statement 2 is correct: Mercury is emitted in the combustion process of coal
2. Coal-based thermal power plants cause mercury and other fossil fuels. Coal has much higher mercury concentrations than
pollution. other fossil fuels, which explains why coal-fired power plants often emit
3. There is no known safe level of exposure to larger quantities of mercury pollution than do power plants that burn other
mercury. fossil fuels.

How many of the above statements are correct? Statement 3 is correct: Mercury is a highly toxic element; there is no known
(a) Only one safe level of exposure. Ideally, neither children nor adults should have any
(b) Only two mercury in their bodies because it provides no physiological benefit.
(c) All three
(d) None

9 Which of the following can be found as pollutants c Only option 1, 3, 5 are correct.
in the drinking water in some parts of India? Water sources can get polluted because, of a range of harmful
(2013) contaminants. Inorganic contaminants in water include metals such as
1. Arsenic fluoride, arsenic, lead, copper, chromium, mercury, antimony, cyanide that
2. Sorbitol
TELEGRAM - UPSC PRELIMS WITH NEELESH
Learn ETHICS, ESSAY, SOCIOLOGY, CSAT BY VISITING THE CHANNEL (INTEGRATED MARATHON FOR 2024)
https://t.me/UPSCPrelimsWithNeelesh ©Copyright For Queries Whatsapp - 9310161970
3. Fluoride can get into drinking water (surface as well as groundwater) from natural
4. Formaldehyde sources, industrial processes, as well as from plumbing systems.
5. Uranium Uranium is a naturally occurring radioactive chemical element that occurs
Select the correct answer using the codes given in low concentrations in nature. It is present in certain types of soils and
below: rocks, especially granites. Uranium in groundwater is result of the
(a) 1 and 3 only dissolution of Uranium bearing minerals that have been in contact with
(b) 2, 4 and 5 only ground water for long periods of time.
(c) 1, 3 and 5 only
(d) 1, 2, 3, 4 and 5

10 ln the context of WHO Air Quality Guidelines, b Statement 1 is correct: World Health Organization (WHO) in its first-ever
consider the following statements: (2022) update since 2005 tightened global air pollution standards in Global Air
1. The 24-hour mean of PM2.5 should not exceed Quality Guidelines (AQGs) 2021. WHO's new guidelines recommend air
15 ug/m3 and annual mean of PM2.5 should not quality levels for six pollutants - particulate matter (PM), ozone (O3),
exceed 5 ug/m3. nitrogen dioxide (NO2) sulfur dioxide (SO2) and carbon monoxide (CO). As
2. ln a year, the highest levels of ozone pollution per the guidelines; the annual average for PM 2.5 should not exceed 5
occur during the periods of inclement weather. micrograms per cubic metre. Whereas, the 24- hour average for PM 2 5
3. PM10 can penetrate the lung barrier and enter should not exceed 15 micrograms per cubic metre.
the bloodstream.
4. Excessive ozone in the air can trigger asthma. Statement 2 is not correct: Tropospheric, or ground level ozone, is not
emitted directly into the air, but is created by chemical reactions between
Which of the statements given above are correct? oxides of nitrogen (NOx) and volatile organic compounds (VOC) in the
(a) 1,3 and 4 only presence of sunlight. Hence, ozone is most likely to reach highest levels on
(b) 1 and 4 only hot sunny days when air is stagnant. Inclement weather means unpleasant
(c) 2, 3 and 4 only weather, especially with cold wind and rain. Ozone production is more
(d) 1 and 2 only limited when it is cloudy, cool, rainy, or windy.

Statement 3 is not correct: PM10 describes inhalable particles, with


diameters that are generally 10 micrometers and smaller. PM 10 can
penetrate and lodge deep inside the lungs, but cannot enter the
TELEGRAM - UPSC PRELIMS WITH NEELESH
Learn ETHICS, ESSAY, SOCIOLOGY, CSAT BY VISITING THE CHANNEL (INTEGRATED MARATHON FOR 2024)
https://t.me/UPSCPrelimsWithNeelesh ©Copyright For Queries Whatsapp - 9310161970
bloodstream. More health- damaging particles are PM 2.5 (particles with a
diameter of 2.5 microns or less). PM2.5 can penetrate the lung barrier and
enter the blood system.

Statement 4 is correct: Depending on the level of exposure, Ozone can


cause coughing & sore throat, damage airways, aggravate lung diseases such
as asthma, emphysema, and chronic bronchitis, and increase the frequency
of asthma attacks etc. People with certain genetic characteristics, and
people with reduced intake of certain nutrients, such as vitamins C and E,
are at greater risk from ozone exposure.

11 In the cities of our country, which among the b Air Quality Index is a tool for effective communication of air quality status
following atmospheric gasses are normally to people in terms, which are easy to understand. It transforms complex air
considered in calculating the value of the Air quality data of various pollutants into a single number (index value),
Quality Index? (2016) nomenclature and color.
1. Carbon dioxide AQ sub-index and health breakpoints are evolved for eight pollutants: PM
2. Carbon monoxide 10, PM 2.5, NO2 , SO2 , CO (and not Carbon dioxide), Ozone, Ammonia and
3. Nitrogen dioxide Pb (lead).
4. Sulfur dioxide
5. Methane
Select the correct answer using the code given
below:
(a) 1,2 and 3 only
(b) 2,3 and 4 only
(c) 1, 4 and 5 only
(d) 1,2,3, 4 and 5

TELEGRAM - UPSC PRELIMS WITH NEELESH


Learn ETHICS, ESSAY, SOCIOLOGY, CSAT BY VISITING THE CHANNEL (INTEGRATED MARATHON FOR 2024)
https://t.me/UPSCPrelimsWithNeelesh ©Copyright For Queries Whatsapp - 9310161970
12 Which one of the following best describes the a Greenwashing is the process of conveying a false impression or providing
term "greenwashing"? (2022) misleading information about how a company's products are more
(a) Conveying a false impression that a company's environmentally sound. Greenwashing is considered an unsubstantiated
products are eco-friendly and environmentally claim to deceive consumers into believing that a company's products are
sound. environmentally friendly. Hence, option (a) is the correct answer
(b) Non-inclusion of ecological/ environmental
costs in the Annual Financial Statements of a
country.
(c) Ignoring the disastrous ecological consequences
while undertaking infrastructure development.
(d) Making mandatory provisions for
environmental costs in a government
project/programme

13 Magnetite particles, suspected to cause b Magnetite is an iron-oxide mineral that occurs naturally on Earth. Because
neurodegenerative problems, are generated as it is also an important component of many anthropogenic materials (e.g.,
environmental pollutants from which of the coal fly ash) and synthetic products (e.g., black toner powders), magnetite
following? (2021) can be released to the environment through human activities.
1. Brakes of motor vehicles Recently, scientists had discovered tiny magnetic particles from air pollution
2. Engines of motor vehicles lodged in human brains. Presence of magnetite nanoparticles in the brain is
3. Microwave stoves within homes linked to several neurodegenerative diseases, including Alzheimer's disease.
4. Power plants Brakes of motor vehicles: Magnetite powder is added in brake pads as a
5. Telephone lines lubricant/filler. Frictional heating of brake pads produces high-temperature
magnetite nanoparticles.
Select the correct answer using the code given Engine of motor vehicles: Magnetite particles have been detected in the
below: particulate matter collected from diesel engine exhaust.
(a) 1,2,3 and 5 only Power plants: Use of coal for industrial applications such as coking in the
(b) 1,2 and 4 only power plants
(c) 3, 4 and 5 only is indicated to be a major source of the spherical magnetic combustion-
(d) 1,2,3,4 and 5 associated magnetite fine particle.
TELEGRAM - UPSC PRELIMS WITH NEELESH
Learn ETHICS, ESSAY, SOCIOLOGY, CSAT BY VISITING THE CHANNEL (INTEGRATED MARATHON FOR 2024)
https://t.me/UPSCPrelimsWithNeelesh ©Copyright For Queries Whatsapp - 9310161970
No evidence has been found out for the magnetite particles from microwave
stoves within homes and telephone lines. Hence, option (b) is the correct
answer.

14 ln the context of which one of the following are d Waste-to-Energy (WtE) technologies consist of any waste treatment
the terms 'pyrolysis and plasma gasification’ process that creates energy in the form of electricity, heat or transport fuels
mentioned? (2019) from a waste source.
(a) Extraction of rare earth elements.
(b) Natural gas extraction technologies. Various WtE technologies are:
(c) Hydrogen fuel-based automobiles. Plasma gasification process: It uses electricity to generate a high
(d) Waste-to-energy technologies. temperature plasma arc (above 3000oC) inside the plasma reactor which
converts the waste into syngas.
Pyrolysis: It uses heat to break down combustible materials in the absence
of oxygen, producing a mixture of combustible gasses (primarily methane,
complex hydrocarbons, hydrogen, and carbon monoxide), liquids and solid
residues. The products of the pyrolysis process are:
(i) a gas mixture; (ii) a liquid (bio-oil/tar); (iii) a solid residue (carbon black).
Biomethanation: lt is anaerobic digestion of organic materials which is
converted into biogas. Anaerobic digestion (AD) is a bacterial fermentation
process that operates without free oxygen and results in a biogas containing
mostly methane (60%), carbon dioxide (40%) and other gasses.
Biomethanation has dual benefits. It gives biogas as well as manure as the
end product.
Incineration: Incineration technology is complete combustion of waste with
the recovery of heat to produce steam that in turn produces power through
steam turbines.
Hence, option (d) is the correct answer.

TELEGRAM - UPSC PRELIMS WITH NEELESH


Learn ETHICS, ESSAY, SOCIOLOGY, CSAT BY VISITING THE CHANNEL (INTEGRATED MARATHON FOR 2024)
https://t.me/UPSCPrelimsWithNeelesh ©Copyright For Queries Whatsapp - 9310161970
15 ln India extender producer responsibility was c Extended Producer Responsibility (EPR) is a policy approach under which
introduced as an important feature in the producers are given a significant responsibility - financial and/ or physical -
following: (2019) for the treatment or disposal of post-consumer products. Extended
(a) The Bio-medica! Waste (Management and producer's responsibility (EPR) is one of the main features of the E-waste
Handling) Rules, 1998 (Management and Handling) Rules, 2011, wherein the producer of electrical
(b) The Recycled Plastic (Manufacturing and Usage) and electronic equipment has the responsibility of managing such
Rules, 1999 equipment after its 'end of life', thus the producer is responsible for their
(c) The E-Waste (Management and Handling) Rules, products once the consumer discards them. Under this EPR, producers are
2011 also entrusted with the responsibility to finance and organize a system to
(d) The Food Safety and Standard Regulations, 2011 meet the costs involved in complying with EPR. Hence, option (c) is the
correct answer.

16 Why is there a concern about copper smelting b Copper smelting is a process where elemental copper is separated from
plants? (2021) copper concentrates through multiple sulfides oxidizing stages.
1. They may release lethal quantities of carbon -
monoxide into the environment. Statement 1 is not correct: Copper smelting plants do release Carbon
2. The copper slag can cause the leaching of some monoxide, but not in lethal quantities.
heavy metals into the environment.
3. They may release sulfur dioxide as a pollutant. Statement 2 is correct: The other byproduct of smelting is slag, the waste
Select the correct answer using the codes given matter separated from metals. This slag may leach heavy metals (arsenic,
below: cadmium, lead or mercury depending on the composition of the ore) into
(a) 1 and 2 Only groundwater reservoirs.
(b) 2 and 3 Only
(c) 1 and 3 Only Statement 3 is correct: Most copper ores are sulphur-based and smelting
(d) 1, 2 and 3 releases sulphur dioxide, an air pollutant known to have many harmful
effects. Sometimes, the concentration of sulphur dioxide is so high that
industries are forced to convert it into sulphuric acid, which itself is a water
contaminant. Hence, option (b) is the correct answer.

TELEGRAM - UPSC PRELIMS WITH NEELESH


Learn ETHICS, ESSAY, SOCIOLOGY, CSAT BY VISITING THE CHANNEL (INTEGRATED MARATHON FOR 2024)
https://t.me/UPSCPrelimsWithNeelesh ©Copyright For Queries Whatsapp - 9310161970
17 Which of the following are some important d The steel industry is one of the highest contributors to air pollution and has
pollutants released by the steel industry in India? among the highest rates of carbon dioxide emissions. Steel particularly
(2014) requires coal for energy, which is a source of emission of Pollutants. Steel
1. Oxides of sulfur plants emit air pollutants such as PM, carbon dioxide, sulphur oxides,
2. Oxides of nitrogen nitrogen oxides, carbon monoxide etc. Hence, option d is the correct
3. Carbon monoxide answer.
4. Carbon dioxide
Select the correct answer using the code given Trick: Since it is some, and there is a case of possibility, there are
below: maximum chances that all of the given options are correct. So, you
(a) 1, 3 and 4 only could have simply mentioned all the options as correct even without
(b) 2 and 3 only knowing the answers
(c) 1 and 4 only
(d) 1,2,3 and 4

18 R2 Code of Practices" constitutes a tool available a R2 Standard provides a common set of criteria to recognize responsible
for promoting the adoption of (2021) reuse and recycling practices, all along the used portion of the electronics
(a) environmentally responsible practices in the lifecycle. R2 Certification is the formal program for evaluating and
electronics recycling industry. monitoring businesses in meeting the R2 Standard, protecting the
(b) ecological management of 'Wetlands of environment, the health and safety of workers, and communities, and
International Importance' under the Ramsar positively impacting the movement toward a sustainable circular economy
Convention. while enriching the lives of people all around the world.
(c) sustainable practices in the cultivation of
agricultural crops in degraded lands.
(d)'Environmental Impact Assessment, in the
exploitation of natural resources.

19 Why is there a great concern about the a Microbeads, a type of microplastic, are very tiny pieces of manufactured
‘microbeads' that are released into the polyethylene plastic that are added as exfoliants to health and beauty
environment? (2019) products, such as some cleansers and toothpastes. Microbeads easily pass
through water filtration systems and end up in the ocean and Lakes, posing
TELEGRAM - UPSC PRELIMS WITH NEELESH
Learn ETHICS, ESSAY, SOCIOLOGY, CSAT BY VISITING THE CHANNEL (INTEGRATED MARATHON FOR 2024)
https://t.me/UPSCPrelimsWithNeelesh ©Copyright For Queries Whatsapp - 9310161970
(a) They are considered harmful to marine a potential threat to aquatic life. Plastic absorbs contaminants in water
ecosystems. leading to a single microbead being 1 million times more toxic that its
(b) They are considered to cause skin cancer in surrounding water. Which is harmful to primary consumer that eats it such
children. as fishes. Hence, option (a) is the correct answer.
(c) They are small enough to be absorbed by crop
plants in irrigated fields.
(d) They are often found to be used as food
adulterants.

20 Biological Oxygen Demand (BOD) is a standard c The amount of oxygen required by bacteria to break down the organic
criterion for (2017) matter present in a certain volume of a sample of water, is called Biological
(a) Measuring oxygen levels in blood. Oxygen Demand (BOD).
(b) Computing- oxygen levels in forest ecosystems.
(c) Pollution assay in aquatic ecosystems. BOD is often used in wastewater treatment plants and is used as an index of
(d) Assessing oxygen levels in high altitude regions. the degree of organic pollution in water. Clean water generally has a BOD
value of less than 5 ppm whereas highly polluted water can have a BOD
value of 17 ppm or more. The presence of a sufficient concentration of
dissolved oxygen is critical to maintaining the aquatic life and aesthetic
quality of streams and lakes. Hence, option (c) is the correct answer.

21 Which of the following is/are the possible b Statement 1 is not correct: Excessive instream sand-and-gravel mining
consequence/s of heavy sand mining in riverbeds? causes the degradation of rivers. Depletion of sand in the streambed and
(2018) along coastal areas causes the deepening of rivers and estuaries, and the
1. Decreased salinity in the river enlargement of river mouths and coastal inlets. It may also lead to saline-
2. Pollution of groundwater water intrusion from the nearby sea.
3. Lowering of the water-table
Statement 2 is correct: Sand mining activities will have an impact upon the
Select the correct answer using the code given river's water quality. Impacts include sedimentation due to stockpiling and
below: dumping of excess mining materials and organic particulate matter, and oil
(a) 1 only spills or leakage from excavation machinery and transportation vehicles. The
TELEGRAM - UPSC PRELIMS WITH NEELESH
Learn ETHICS, ESSAY, SOCIOLOGY, CSAT BY VISITING THE CHANNEL (INTEGRATED MARATHON FOR 2024)
https://t.me/UPSCPrelimsWithNeelesh ©Copyright For Queries Whatsapp - 9310161970
(b) 2 and 3 only impact is particularly significant if water users downstream of the site are
(c) 1 and 3 only abstracting water for domestic use.
(d) 1,2 and 3
Statement 3 is correct: Sand mining results in the destruction of aquatic and
riparian habitat through large changes in the channel morphology. Impacts
include bed degradation, bed coarsening, lowered water tables near the
streambed, and channel instability.

22 ln the context of solving pollution problems, what c Statement 1 is correct: Bioremediation is the use of microorganisms which
is/are the advantages/advantages of occur in nature (bacteria and fungi) to degrade the environmental
bioremediation techniques? (2017) contaminants into less toxic forms.
1. It is a technique for cleaning up pollution by
enhancing the same biodegradation process that Statement 2 is not correct. Bioremediation is a technique for
occurs in nature. removing/converting harmful contaminants like heavy metals into less
2. Any contaminant with heavy metals such as harmful substance (Bioremediation cannot completely remove heavy
cadmium and lead can be readily and completely metals); or degrading organic substances and ultimate mineralization of
treated by bioremediation using microorganisms. organic substances into carbon dioxide, water, nitrogen gas, etc., employing
3. Genetic engineering can be used to create dead or alive biomass.
microorganisms specifically designed for
bioremediation. Statement 3 is correct: Bioremediation analyzes different gene diversity and
metabolic diversity in microorganisms. The genetic construction of such
Select the correct answer using the code given microorganisms makes them beneficial in bioremediation.
below:
(a) 1 only
(b) 2 and 3 only
(c) 1 and 3 only
(d) 1, 2 and 3

TELEGRAM - UPSC PRELIMS WITH NEELESH


Learn ETHICS, ESSAY, SOCIOLOGY, CSAT BY VISITING THE CHANNEL (INTEGRATED MARATHON FOR 2024)
https://t.me/UPSCPrelimsWithNeelesh ©Copyright For Queries Whatsapp - 9310161970
23 Which of the following can be threats to the a Biodiversity is the number and variety of organisms found within a specified
biodiversity of a geographical area? (2012) geographic region. The threat to biodiversity may be caused
1. Global warming in the following 3 ways:
2. Fragmentation of habitat Direct ways: Deforestation, hunting, poaching, commercial exploitation.
3. Invasion of alien species Indirect ways: Loss or modification of the natural habitats, introduction of
4. Promotion of vegetarianism exotic species, pollution, human- induced climate change etc.
Natural causes: Climate change
Select the correct answer using the codes given
below: Habitat loss and fragmentation: When large habitats are broken up into
(a) 1, 2 and 3 only small fragments due
(b) 2 and 3 only to various human activities, mammals and birds requiring large territories
(c) 1 and 4 only and certain animals with migratory habits are badly affected, leading to
(d) 1, 2, 3 and 4 population declines.
Alien species invasions: When alien species are introduced unintentionally
or deliberately for whatever purpose, some of them turn invasive, and cause
decline or extinction of indigenous species. For e.g. The Nile perch
introduced into Lake Victoria in east Africa led eventually to the extinction
of an ecologically unique assemblage of more than 200 species of cichlid
fish in the lake.
Global Climate Change: With the increase in temperature expected from
increasing greenhouse gases, there will be higher levels of air pollution,
greater variability in weather patterns, and changes in the distribution of
vegetation in the landscape. Some species will not be able to adapt to these
changes in the environment and will become extinct.
There is no scientific evidence behind the claim that promotion of
vegetarianism can be a threat to biodiversity. Hence, option [a) is the correct
answer.

TELEGRAM - UPSC PRELIMS WITH NEELESH


Learn ETHICS, ESSAY, SOCIOLOGY, CSAT BY VISITING THE CHANNEL (INTEGRATED MARATHON FOR 2024)
https://t.me/UPSCPrelimsWithNeelesh ©Copyright For Queries Whatsapp - 9310161970
24 Other than poaching, what are the possible c Once present in tens of thousands of numbers, the Ganges River dolphin
reasons for the decline in the population of has dwindled to less than 2000 during the last century because of the
Ganges River Dolphins? (2014) following reasons.
1. Construction of dams and barrages on rivers.
2. lncrease in the population of crocodiles in - Ganges river dolphins are divided into isolated groups because of the
rivers. construction of more than 50 dams and other irrigation-related projects.
3. Getting trapped in fishing nets accidentally. This makes them susceptible to inbreeding and more vulnerable to other
4. Use of synthetic fertilizers and other threats because they cannot move to new areas. Dolphins trapped above a
agricultural chemicals in crop-fields in the vicinity dam are exposed to poaching and those below a dam are threatened by
of rivers. heavy pollution, increased fishing activities and vessel traffic. They also have
less food because dams disturb the migration, breeding cycles and habitat
Select the correct answer using the code given of fish and other prey. Hence, statement 1 is correct.
below:
(a) 1 and 2 only - There is no evidence supporting the increase in population of crocodiles
(b) 2 and 3 only causes decline in population of Ganges River Dolphin. Hence statement 2 is
(c) 1,3 and 4 only not correct.
(d) 1,2,3 and 4
- They die as a result of accidentally being caught in fishing nets. Hence,
statement 3 is correct.

- Hunting and poaching: For their meat and oil, both used medicinally.

- Pollution: lndustrial, agricultural, and human pollution cause


degradation of their habitat. Each year tons of pesticides and fertilizers are
used in the vicinity of the river. And the high levels of these persistent toxic
chemicals have been found in their bodies as well as in the body of their
prey, which adversely affect their health and ultimately result in their death.
Hence, statement 4 is correct.

TELEGRAM - UPSC PRELIMS WITH NEELESH


Learn ETHICS, ESSAY, SOCIOLOGY, CSAT BY VISITING THE CHANNEL (INTEGRATED MARATHON FOR 2024)
https://t.me/UPSCPrelimsWithNeelesh ©Copyright For Queries Whatsapp - 9310161970
25 Consider the following statements: (2022) b Statement 1 is not correct: India’s Bhadla Solar Park is the largest solar
1. Gujarat has the largest solar park in India. power park in the world. It is located in Rajasthan.
2. Kerala has a fully solar powered International
Airport. Statement 2 is correct: Cochin Airport in Kerala has become the first airport
3. Goa has the largest floating solar photovoltaic in the world that completely operates on solar power.
project in India. Statement 3 is not correct: The Ramagundam project in Telangana is India's
Which of the statements given above is/are largest floating solar photovoltaic project with 4.5 lakh ‘Made In India’ solar
correct? PV Modules.
(a) 1 and 2 Only
(b) 2 Only
(c) 1 and 3 Only
(d) 3 only

26 ln the context of proposals to the use of hydrogen- b Blending of hydrogen with CNG provides a blended gas termed as hydrogen-
enriched CNG (H-CNG) as fuel for buses in public enriched natural gas (HCNG).
transport, consider the following statements: Statement 1 is not correct and statement 2 is correct: HCNG fuel emits 70%
(2019) less carbon monoxide, reduces total hydrocarbon emissions by around 15%
1. The main advantage of the use of H-CNG is the and increases fuel efficiency by 3-4%. Thus, it does not completely eliminate
elimination of carbon monoxide emissions. carbon monoxide emissions but reduces it.
2. H-CNG as fuel reduces carbon dioxide and
hydrocarbon emissions. Statement 3 is correct: Studies indicate that HCNG mixtures with 20-30%
3. Hydrogen up to one-fifth by volume can be hydrogen by volume is optimal for vehicle performance and emission
blended with CNG as fuel for buses. reduction. ln India, the Ministry of Road Transport and Highways has
4. H-CNG makes the fuel less expensive than CNG. allowed the use of H-CNG (18% mix of hydrogen i.e., approximately one-
Which of the statements given above is/are fifth) in CNG engines.
correct?
(a) 1 only Statement 4 is not correct: Globally, hydrogen required for blending in CNG
(b) 2 and 3 only is produced through electrolysis of water, followed by high-pressure
(c) 4 only blending with CNG which is a costly process.
(d) 1,2,3 and 4
TELEGRAM - UPSC PRELIMS WITH NEELESH
Learn ETHICS, ESSAY, SOCIOLOGY, CSAT BY VISITING THE CHANNEL (INTEGRATED MARATHON FOR 2024)
https://t.me/UPSCPrelimsWithNeelesh ©Copyright For Queries Whatsapp - 9310161970
27 Consider the following heavy industries: (2023) c Green hydrogen is the hydrogen produced through electrolysis of water
1. Fertilizer plants using electricity from renewable sources. It can play a role in decarbonizing
2. Oil refineries all of the mentioned heavy industries: fertilizer plants, oil refineries, and
3. Steel plants steel plants.

Green hydrogen is expected to play a significant Fertilizer Plants: Fertilizer production is energy-intensive and often relies
role in decarbonizing how many of the above on fossil fuels, particularly natural gas. By using green hydrogen as a clean
industries? energy source, fertilizer plants can reduce their carbon emissions. Green
(a) Only one hydrogen can be used to generate electricity or directly as a feedstock in
(b) Only two the ammonia production process, which is a key component of fertilizer
(c) All three manufacturing.
(d) None
Oil Refineries: Oil refineries are major contributors to greenhouse gas
emissions due to their energy-intensive processes and reliance on fossil
fuels. Green hydrogen can be utilized in oil refineries to replace or
supplement fossil fuels, reducing their carbon footprint. Hydrogen can be
used in refining processes, such as hydrocracking and hydrotreating, to
remove impurities and improve the quality of fuels.

Steel Plants: The steel industry is responsible for a significant amount of


global carbon emissions. Green hydrogen can be used as a reducing agent
in the steelmaking process, replacing coal or coke. This process, known as
direct reduction, enables the production of “green” or low-carbon steel.
Additionally, green hydrogen can be utilized in the steel industry for heat
generation and electricity production, further reducing carbon emissions.

28 Which of the following statements are correct d About Methane hydrate:


about the deposits of 'methane hydrate'? (2019) Huge amounts of methane are stored around the world in the sea floor in
1. Global warming might trigger the release of the form of solid methane hydrates. These hydrates represent a large energy
methane gas from these deposits. reserve for humanity.
TELEGRAM - UPSC PRELIMS WITH NEELESH
Learn ETHICS, ESSAY, SOCIOLOGY, CSAT BY VISITING THE CHANNEL (INTEGRATED MARATHON FOR 2024)
https://t.me/UPSCPrelimsWithNeelesh ©Copyright For Queries Whatsapp - 9310161970
2. Large deposits of 'methane hydrate' are found Methane hydrate is a crystalline solid that consists of a methane molecule
in Arctic Tundra and under the seafloor. surrounded by a cage of interlocking water molecules. Methane hydrate is
3. Methane in the atmosphere oxidizes to carbon an "ice" that only occurs naturally in subsurface deposits where
dioxide after a decade or two. temperature and pressure conditions are favorable for its formation.
Select the correct answer using the code given
below: Statement 1 is correct: Warming of oceans could cause gradual melting of
(a) 1 and 2 only gas hydrates near the sediment - water interface and those below
(b) 2 and 3 only permafrost. Thus, global warming might trigger the release of methane gas
(c) I and 3 only from these deposits.
(d) 1,2 and 3
Statement 2 is correct: Enormous amounts of methane hydrate have been
found at :1) sediment and sedimentary rock units below Arctic permafrost;
2) sedimentary deposits along continental margins; 3) deep-water
sediments of inland lakes and seas; and, 4) under Antarctic ice.

Statement 3 is correct: When methane is anthropogenically emitted,


methane is oxidized in the atmosphere a decade or two later. Once
oxidized, the carbon in each methane molecule is converted to CO2, which
then stays in the atmosphere as CO2, for another century or more

TELEGRAM - UPSC PRELIMS WITH NEELESH


Learn ETHICS, ESSAY, SOCIOLOGY, CSAT BY VISITING THE CHANNEL (INTEGRATED MARATHON FOR 2024)
https://t.me/UPSCPrelimsWithNeelesh ©Copyright For Queries Whatsapp - 9310161970
TOPIC 3: BIODIVERSITY (Ch 9 – BIODIVERSITY, Ch 10 – INDIAN BIODIVERSITY DIVERSE LANDSCAPE, Ch 11 –
SCHEDULED ANIMALS OF WPA 1972, Ch 12 – ANIMAL DIVERSITY OF INDIA, Ch 13 – PLANT DIVERSITY OF INDIA,
CH 14 – MARINE ORGANISM)
Total Number of Questions
asked from 2011-2023 37
My Instructions related to GENERAL INSTRUCTION
these chapters This chapter is also one of the most important sections for environment.

CHAPTER SPECIFIC INSTRUCTIONS

CHAPTER 9 BIODIVERSITY
This is very important chapter. You must know its basics and the terms used. You should know the level of
biodiversity, measurement of biodiversity, modes of conservation (very important – know ex-situ and in-situ,
their meaning and what are included in it), what is red data book and who release it, what is the meaning of
critically endangered.

CHAPTER 10 INDIAN BIODIVERSITY DIVERSE LANDSCAPE


This chapter is overlapping with geography at many places. You should know that India has 2 realms, 5
biomes, 10 biogeographic zones and 25 biogeographic provinces. Have the knowledge about biomes of India.
Know what is lichen. Going in excess details in neither feasible nor advisable.

CHAPTER 11 SCHEDULE ANIMALS OF WPA 1972


All the schedules (1 TO 6) of WPA and their every constituent (conditions, restrictions, permission etc.) is
important. It has been asked many times. Read it properly. It is very very important.
In The table it is neither feasible nor practical to remember each animal details. So, just have a read and focus
more on Brown Antlered deer, cheetah, Indian gazelle (chinkara), dugong, Ganges River dolphin, Golden Cat,
Golden Langur, Hoolock Gibbon, Leopard, Lion tailed Macaque, Himalayan musk deer, Greater One horned
Rhino, Snow leopard, Wild Ass, Tiger, Gharial, Olive Ridley (Please note that – just because I am mentioning

TELEGRAM - UPSC PRELIMS WITH NEELESH


Learn ETHICS, ESSAY, SOCIOLOGY, CSAT BY VISITING THE CHANNEL (INTEGRATED MARATHON FOR 2024)
https://t.me/UPSCPrelimsWithNeelesh ©Copyright For Queries Whatsapp - 9310161970
these to specially focus on – it does not mean others do not have importance. But then we have to choose
due to the limitation of human mind. Anything if you think, you must read, you can definitely read). Try to
recall which are endangered, critically endangered and vulnerable. This you can recall by occasional revisions.
Do not try to remember everything at one go. It won’t be feasible.

CHAPTER 12 ANIMAL DIVERSITY OF INDIA


Atleast know that which are mammals, which are birds etc. as per their names.

CHAPTER 13 PLANT DIVERSITY OF INDIA


Must know the different types of plant classification, what is die back, insectivores’ plants – why they are like
this? names of the Indian hunters, invasive species definition and examples (distribution is not important),
annual rings and their role – indicate tree life

CHAPTER 14 MARINE ORGANISMS


This is very important chapter. Read full

1 'Gadgil Committee Report' and 'Kasturirangan d Gadgil Committee Report' and 'Kasturirangan Committee Report', are
Committee Report', sometimes seen in the news, related to protection of Western Ghats.
are related to (2016)
(a) constitutional reforms
(b) Ganga Action Plan
(c) Linking of rivers
(d) Protection of Western Ghats

2 Consider the following statements: (2011) c Statement 1 is correct: The diversity of plants and animals is not uniform
1. Biodiversity is normally greater in the lower throughout the world but shows a rather uneven distribution. As per
latitudes as compared to the higher latitudes. latitudinal gradient in diversity, species diversity decreases from the
equator towards the poles. With very few exceptions, tropics (latitudinal

TELEGRAM - UPSC PRELIMS WITH NEELESH


Learn ETHICS, ESSAY, SOCIOLOGY, CSAT BY VISITING THE CHANNEL (INTEGRATED MARATHON FOR 2024)
https://t.me/UPSCPrelimsWithNeelesh ©Copyright For Queries Whatsapp - 9310161970
2. Along the mountain gradients, biodiversity is range of 23.5' N to 23.5'S) harbor more species than temperate or polar
normally greater in the lower altitudes as areas.
compared to the higher altitudes.
Which of the statements given above is/are Statement 2 is correct: Mountains at lower altitudes can support
correct? exceptional biodiversity, due to compression of a wide range of
(a) 1 only ecosystems into a relatively short distance. There is a decrease in species
(b) 2 only diversity from lower to higher altitudes on a mountain. Drop in
(c) Both 1 and 2 temperature and greater seasonal variability at higher altitudes are a
(d) Neither 1 nor 2 major factor that reduces diversity.

3 Biodiversity forms the basis for human existence d The various benefits of biological diversity can be grouped under three
in the following ways: (2011) categories:
1. Soil formation (a) Ecosystem services: Protection of water resources, soil protection,
2. Prevention of soil erosion nutrient storage and cycling, pollution reduction, climate stability,
3. Recycling of waste maintenance of ecological processes.
4. Pollination of crops (b) Biological resources: Food, fibre, medicines, fuel wood and
Which of the statements given above is/are ornamental plants, breeding material for crop improvement
correct? (c) Social benefits: Recreation, cultural values
{a) 1,2 and 3 only Biological diversity helps in the formation and maintenance of soil
(b) 2, 3and 4 only structure and the retention of moisture and nutrient levels. Clearing large
(c) 2 and 3 only areas of vegetation cover has been often seen to accelerate soil erosion,
(d) 1,2,3 and 4 reduce its productivity and often result in flash floods.
Micro-organisms decompose the organic waste and recycle nutrients in
the environment.
Birds and nectar-loving insects which roost and breed in natural habitats
are important pollinating agents of crop and wild plants Hence, option
(d) is the correct answer

TELEGRAM - UPSC PRELIMS WITH NEELESH


Learn ETHICS, ESSAY, SOCIOLOGY, CSAT BY VISITING THE CHANNEL (INTEGRATED MARATHON FOR 2024)
https://t.me/UPSCPrelimsWithNeelesh ©Copyright For Queries Whatsapp - 9310161970
4 ln which of the following regions of India are you d Greater lndian Hornbill is the largest member of the horn bill family.
most likely to come across the 'Great Indian Great lndian Hornbills have a wide distribution, occurring in China, lndia,
Hornbill' in its natural habitat? (2016) Nepal, Bhutan, Bangladesh, Myanmar, Thailand, Laos, Vietnam,
(a) Sand deserts of northwest lndia Cambodia, Malaysia and Sumatra (Indonesia). The bulk of the population
(b) Higher Himalayas of Jammu and Kashmir is found in India, where it is restricted to the Himalayan foothills, hill
(c) Salt marshes of western Gujarat forests in northeast lndia and also, in the wet evergreen forests of
(d) Western Ghats Western part of lndia i.e. Western Ghats. Hence, option (d) is the correct
answer.

5 A sandy and saline area is the natural habitat of an b Indian Wild Ass is a strong and majestic animal whose range once
Indian animal species. The animal has no extended from Southern India towards southern Pakistan Afghanistan
predators in that area but its existence is and south-eastern Iran. Now, the world’s last population of Indian Wild
threatened due to the destruction of its habitat. Ass is restricted within their last refuge of Indian desert called Rann of
Which one of the following could be that animal? Kutch They don't have any predators in the area. But the harsh climate
(2011) and dynamic nature of the landscape coupled with the vast barren land
(a) lndian wild buffalo offers multiple challenges towards the survival of the Wild Ass. The loss
(b) lndian wild ass of habitat due to salt manufacturing activities, exotic plants, and cattle
(c) lndian wild boar grazing has also affected their population.
(d) lndian gazelle

6 Recently, for the first time in our country, which of d ln 2015, Maharashtra became the first State in the country to have a
the following States has declared a particular 'State butterfly' The state government declared the Blue Mormon as the
butterfly as 'State Butterfly'? (2016) State butterfly
(a) Arunachal Pradesh
(b) Himachal Pradesh Note – this is a current affairs based questions. But, it shows the
(c) Karnataka mindset – if any such state makes any animal/bird/insect etc its state
(d) Maharashtra symbol, you should know it.

TELEGRAM - UPSC PRELIMS WITH NEELESH


Learn ETHICS, ESSAY, SOCIOLOGY, CSAT BY VISITING THE CHANNEL (INTEGRATED MARATHON FOR 2024)
https://t.me/UPSCPrelimsWithNeelesh ©Copyright For Queries Whatsapp - 9310161970
7 Recently, our scientists have discovered a new and a ln 2015, a new and distinct species of banana plant was discovered by
distinct species of banana plant which attains a the scientists at the Botanical Survey of lndia from l6 km inside the
height of about 11 metres and has orange-colored Krishna Nalah tropical rain forest on the Little Andaman islands.
fruit pulp. ln which part of lndia has it been
discovered? (2016)
(a) Andaman lslands
{b) Anaimalai Forests
(c) Maikala Hills
(d) Tropical rain forests of northeast

8 Consider the following fauna of lndia: (2013) c ln 2013, IUCN status of all the given fauna was endangered, but their
1. Gharial current status has been changed.
2. Leatherback turtle
3. Swamp deer Currently:
Which of the above is/are endangered? Gharial – Critically Endangered
(a) 1 and 2 only Leatherbuck Turtle – Critically Endangered
(b) 3 only Swamp Deer – Vulnerable
(c) 1,2 and 3
(d) None

9 Which one of the following groups of animals a International Union for Conservation of Nature (IUCN)'s Red List of
belongs to the category of endangered species? Threatened Species divides species into nine categories: Not Evaluated,
(2012) Data Deficient, Least Concern, Near Threatened, Vulnerable,
(a) Great Indian Bustard, Musk Deer, Red Panda and Endangered, Critically Endangered, Extinct in the Wild and Extinct.
Asiatic Wild Ass. As per the 2012 data, species given in the option (a) were categorized
(b) Kashmir Stag, Cheetal, BIue Bull and Great under "Endangered" species.
Indian Bustard. Current status of the given species are
(c) Snow Leopard, Swamp Deer, Rhesus Monkey Great Indian Bustard Critically Endangered
and Saras (Crane). Musk Deer Endangered
Red Panda Endangered
TELEGRAM - UPSC PRELIMS WITH NEELESH
Learn ETHICS, ESSAY, SOCIOLOGY, CSAT BY VISITING THE CHANNEL (INTEGRATED MARATHON FOR 2024)
https://t.me/UPSCPrelimsWithNeelesh ©Copyright For Queries Whatsapp - 9310161970
(d) Lion-tailed Macaque, Blue Bull, Hanuman Asiatic Wild Ass Near threatened
Langur and Cheetal. Kashmiri Stag Critically Endangered
Cheetal Least Concern
Blue Bull (Nilgai) Least Concern
Snow Leopard Vulnerable
Swamp Deer Vulnerable
Rhesus Monkey Least Concern
Saras (Crane) Vulnerable
Lion Tailed Macaque Endangered
Hanuman Langur Least Concern
.
10 Consider the following statements: (2019) a Statement 1 is correct: The Asiatic lion, also known as the Indian
1. Asiatic lion is naturally found in lndia only. lion and the Persian lion, today survives in the wild only in India. Since
2. Double-humped camel is naturally found in the turn of the 20th century, its range has been restricted to Gir National
lndia only. Park and the surrounding areas in the Indian state of Gujarat. Historically,
3. One-horned rhinoceros is naturally found in it inhabited much of southwest Asia to northern India.
lndia only.
Statement 2 is not correct: Double humped camels (Bactrian camel) are
Which of the statements given above is/are native to the steppes of Central Asia. lndia has a small population of the
correct? double-humped Bactrian, found mostly in the Nubra valley in Ladakh.
(a) 1 only
(b) 2 only Statement 3 is not correct: The greater one- horned rhino (or "lndian
(c) 1 and 3 only rhino") is naturally found in Assam, India (Kaziranga National Park) and
(d) 1,2 and 3 also in Chitwan National Park of Nepal. They are restricted to a few
pockets in Southern Nepal, northern Bengal, and Brahmaputra Valley.

TELEGRAM - UPSC PRELIMS WITH NEELESH


Learn ETHICS, ESSAY, SOCIOLOGY, CSAT BY VISITING THE CHANNEL (INTEGRATED MARATHON FOR 2024)
https://t.me/UPSCPrelimsWithNeelesh ©Copyright For Queries Whatsapp - 9310161970
11 Consider the following: (2013) a Spider Monkey is naturally found in Latin America while rest are found
1. Star tortoise naturally in India
2. Monitor lizard
3. Pygmy hog
4. Spider monkey

Which of the above are naturally found in India?


(a) 1,2 and 3 only
(b) 2 and 3 only
(c) 1 and 4 only
(d) 1,2,3 and 4

12 Consider the following: (2012) b Among the given choices, Black-necked crane, flying squirrel, and Snow
1. BIack-necked crane leopard are naturally found in India. ln 1952, the Indian government
2. Cheetah officially declared the Cheetah extinct in the country. Hence, option (b)
3. Flying squirrel is the correct answer.
4. Snow leopard
Which of the above are naturally found in India?
(a) 1,2 and 3 only
(b) 1,3 and 4 only
(c) 2 and 4 only
(d) 1,2,3 and 4

13 Consider the following statements regarding the b Squirrels are generally small to medium-size rodents but there are few
Indian squirrels: (2023) species that are giant in size like Malabar giant squirrel and Malayan
1. They build nests by making burrows in the giant squirrel. Giant tree squirrels are the members of the squirrel family,
ground. they are arboreal species and live mostly on trees in the forest but now
2. They store their food materials like nuts and also have adapted to human environments. Hence statement 1 is not
seeds in the ground. correct.
3. They are omnivorous.
TELEGRAM - UPSC PRELIMS WITH NEELESH
Learn ETHICS, ESSAY, SOCIOLOGY, CSAT BY VISITING THE CHANNEL (INTEGRATED MARATHON FOR 2024)
https://t.me/UPSCPrelimsWithNeelesh ©Copyright For Queries Whatsapp - 9310161970
How many of the above statements are correct? To prepare for colder months, squirrels cache food during the spring and
(a) Only one summer by gathering extra nuts. They bury the surplus in the area
(b) Only two surrounding their nests, splitting it into different underground pantries
(c) All three to save for later. Hence statement 2 is correct.
(d) None
While nuts and fruits make up a majority of its diet, the Indian sun
squirrel will also eat insects, other smaller mammals, and reptiles. Hence
statement 3 is correct.

14 lf you want to see gharials in their natural habitat, b Gharials are a type of Asian crocodilian distinguished by their long, thin
which one of the following is the best place to snouts. Gharials live in clear freshwater river systems. Chambal River is a
visit? (2017) shelter of rare crocodile species "Gharial' and the National Aquatic
(a) Bhitarkanika Mangroves Animal Gangetic Dolphin.
(b) Chambal River
(c) Pulicat Lake
(d) Deepor Beel

15 Consider the following statements: (2023) c Statement I is correct: There are about 270 marsupial mammals left in
Statement-I: Marsupials are not naturally found in the world today, and they can be found in Australia, Papua New Guinea,
India. and South America. In India, these Marsupials are not found naturally.
Statement-II: Marsupials can thrive only in Kangaroos, wallabies, wombats, opossums, bandicoot and koalas are the
montane grasslands with no predators. Which one examples of marsupials. The pouch, or marsupium, after which marsupials
of the following is correct in respect of the above are named is not present in all marsupial females. For marsupials, the
statements? gestation period can be as short as 10.5 days.
(a) Both Statement-I and Statement-II are correct
and Statement- Statement-II is incorrect. Marsupials can thrive in a variety of habitats,
II is the correct explanation for Statement-I including forests, grasslands, and deserts. They can also live in areas with
(b) Both Statement-I and Statement-II are correct predators. For example, the opossum is a marsupial that is found in North
and Statement-II is not the correct explanation for America. It is a common prey animal for coyotes, foxes, and hawks.
Statement-I However, the opossum has a number of adaptations that help it to survive
TELEGRAM - UPSC PRELIMS WITH NEELESH
Learn ETHICS, ESSAY, SOCIOLOGY, CSAT BY VISITING THE CHANNEL (INTEGRATED MARATHON FOR 2024)
https://t.me/UPSCPrelimsWithNeelesh ©Copyright For Queries Whatsapp - 9310161970
(c) Statement-I is correct but Statement-II is in the wild, including a strong sense of small, sharp claws, and a thick coat
incorrect (d) Statement-I is incorrect but Statement- of fur.
II is correct

16 Consider the following pairs: (2019) c Pair 1 is correctly matched: Mahseer are freshwater fishes. Cauvery
Wildlife Naturally basin has two sub-specie of Mahseer - orange-finned mahseer (hump-
found it backed mahseer) and Blue-finned Mahseer. Orange-finned mahseer is
Blue-finned Mahseer Cauvery River endemic to the Cauvery River basin. But blue finned mahseer is a non-
Irrawaddy Dolphin Chambal River native, artificially bred fish which was introduced in Cauvery.
Rusty spotted Cat Eastern Ghat
Pair 2 is not correctly matched: Irrawaddy dolphins are found in coastal
areas in South and Southeast Asia, and in three rivers: Ayeyarwady
Which of the pairs given correctly matched? (Myanmar), Mahakam (Indonesian Borneo) and Mekong
(a) 1 and 2 only
(b) 2 and 3 only Pair 3 is correctly matched: Rusty-spotted Cats are one of the few wild
(c) 1 and 3 only cats that inhabit the forests of Andhra Pradesh.
(d) 1,2 and 3

17 With reference to lndian elephants, consider the a Statement 1 is correct: An elephant herd may have 10 to 12 female
following statements: (2020) elephants and young ones and it is led by the oldest female member.
1. The leader of an elephant group is a female. Male elephants live in the herd till they are 14 -15 years old. Then they
2. The maximum gestation period can be 22 leave their herd and move around alone.
months.
3. An elephant can normally go on calving till the Statement 2 is correct: Before giving birth to a calf, the mother elephant
age of 40 years only. carries the fetus for 22 months, the longest gestation period of any
4. Among the States in India, the highest elephant mammal.
population is in Kerala.
Statement 3 is not correct: Most female elephants give birth for the first
Which of the statements given above is/are time when they are between 14 and 15 years old in case of African
correct? elephants, and this period is slightly later for Asian elephants. Fecundity
TELEGRAM - UPSC PRELIMS WITH NEELESH
Learn ETHICS, ESSAY, SOCIOLOGY, CSAT BY VISITING THE CHANNEL (INTEGRATED MARATHON FOR 2024)
https://t.me/UPSCPrelimsWithNeelesh ©Copyright For Queries Whatsapp - 9310161970
(a) 1 and 2 only is fairly constant between the ages of 16 and 40 and then declines
(b) 2 and 4 only slightly, though females over 60 can still give birth.
(c) 3 only
(d) 1,3 and 4 only Statement 4 is not correct: Karnataka has the highest number of
elephants followed by Assam and Kerala.

18 What is/are unique about 'Kharai camel’ breed a Kharai is a rare breed of camel found in the salt marshes of Kutch,
found in India? (2016) Gujarat.
1. lt is capable of swimming up to three kilometres Statements 1 and 2 are correct. They swim up to 3 kilometres in the
in seawater' shallow seas to reach their grazing land - the region's mangroves and
2. lt survives by grazing on mangroves saline plants.
3. lt lives in the wild and cannot be domesticated Statement 3 is not correct: Many nomadic communities like Fakirani Jats,
Select the correct answer using the code given Maaldhari
below: community live a pastoral lifestyle with their livelihood intertwined with
(a) 1 and 2 only these camels. Thus, this camel species has been domesticated for
(b) 3 only quite a long time.
(c) 1 and 3 only
(d) 1,2 and 3

19 With reference to 'dugong', a mammal found in c Statement 1 is correct: Dugongs are the only existing species of
lndia, which of the following statements is/are herbivorous mammal that lives exclusively in the sea, including in lndia.
correct? (2015) Statement 2 is not correct: Dugongs are found in Gulf of Mannar, Palk
l. lt is a herbivorous marine animal. Bay, Gulf of Kutch and Andaman and Nicobar Islands. So, they are not
2. lt is found along the entire coast of lndia. found along the entire coast of India.
3. lt is given legal protection under Schedule I of Statement 3 is correct: Dugong is given legal protection under Schedule
the Wildlife (Protection) Act, 1972. I of the Wildlife (Protection) Act; 1972. lt has also been categorized as
"vulnerable" under the IUCN Red List of Threatened Species.
Select the correct answer using the code given
below:
(a) 1 and 2 only
TELEGRAM - UPSC PRELIMS WITH NEELESH
Learn ETHICS, ESSAY, SOCIOLOGY, CSAT BY VISITING THE CHANNEL (INTEGRATED MARATHON FOR 2024)
https://t.me/UPSCPrelimsWithNeelesh ©Copyright For Queries Whatsapp - 9310161970
(b) 2 only
(c) 1 and 3 only
(d) 3 only

20 What is the difference between the antelopes a About Oryx


Oryx and Chiru? (2012) Oryx comprises four large antelope species with long, spear-like horns
(a) Oryx is adapted to live in hot and arid areas living in herds on deserts and dry plains of Africa and the Arabian
whereas Chiru is adapted to live in steppes Peninsula. They feed on grasses and energetically dig for water-storing
and semi- desert areas of cold high mountains. roots and tubers. They can go without drinking except under the harshest
(b) Oryx is poached for its antlers whereas Chiru is conditions. Local communities hunt these large antelopes for their meat
poached for its musk. and hides, and in many cultures, their horns are sought after as charms.
(c) Oryx exists in western India only whereas Chiru About Chiru
exists in north-east India only. The Chiru or Tibetan Antelope is a medium sized deer. It lives on the high
(d) None of the statements (a), (b) and (c) given alpine steppes of the Tibetan Plateau. It is found in the Ladakh region of
above is correct India. It is known for the fine hair of its underbelly and chest, which is
woven into the softest of woolen shawls. The wool is known as Shahtoosh
or King of Wools and is one fifth the thickness of human hair. Their wool
is so fine that a full-sized shawl can pass through a ring worn on one's
finger. This has led to the decimation of the Chiru in Tibet because several
Chiru are killed to harvest the wool from their underbellies. Hence,
option (a) is the correct answer.

21 Certain species of which one of the following a The ant-fungus mutualism is a symbiosis seen between certain ant and
organisms are well known as cultivators of fungi? fungal species, in which ants actively cultivate fungus as a food source.
(2022) An important example is that of Leaf-cutter ants which cultivate the
(a) Ant fungus by providing them fresh plant material, and protecting them from
(b) Cockroach molds and pests. The fungus in turn is a food source for the ants' larva.
(c) Crab Hence, option (a) is the correct answer.
(d) Spider

TELEGRAM - UPSC PRELIMS WITH NEELESH


Learn ETHICS, ESSAY, SOCIOLOGY, CSAT BY VISITING THE CHANNEL (INTEGRATED MARATHON FOR 2024)
https://t.me/UPSCPrelimsWithNeelesh ©Copyright For Queries Whatsapp - 9310161970
22 Which one of the following makes a tool with a b Orangutans are known to use sticks to extract insects from holes in trees.
stick to scrap insects from a hole in a tree or a log They will often break off a small branch and use it to probe the hole, then
of wood? (2023) lick off any insects that are stuck to the stick. This is an example of tool
(a) Fishing cat use, which is a relatively rare behavior in the animal kingdom.
(b) Orangutan Orangutans are also known to use leaves to wipe their faces and to use
(c) Otter sticks to scratch their backs. They are highly intelligent animals and are
(d) Sloth bear capable of learning new behaviors through observation and imitations.

23 Which of the following organisms perform waggle c There are 2 types of dances performed by bees:
dance for others of their kin to indicate the a) The Round Dance and
direction and the distance to a source of their b) The Tail-wagging or Waggle Dance
food? (2023)
(a) Butterflies For food sources 25 to 100 metres from the hive or closer, the round
(b) Dragonflies dance is used. The round dance provides no directional cues.
(c) Honeybees
(d) Wasps The waggle dance takes the place of the round dance as the distance to
the food source increases. The waggle dance tells the direction and
amount of energyneeded to reach the food source.

The amount of energy spent (or distance) is represented by the time


required to complete one circuit. For a food source which is 200 metres
away, a bee might dance 8–9 circuits in 15 seconds, 4-5 in 15 seconds for
a source that is 1000 metres away, and 3 circuits in 15 seconds for a source
that is 2000 metres away.

24 Which of the following is not a bird? (2022) a Golden Mahseer is an endangered species of cyprinid fish. Mahseer
a. Golden Mahseer roughly translates as mahi - fish and sher - tiger, and hence is also
b. lndian Nightjar referred as tiger among fish. lt is a large cyprinid and known io be the
c. Spoonbill toughest among the freshwater sport fish
d. White lbis
TELEGRAM - UPSC PRELIMS WITH NEELESH
Learn ETHICS, ESSAY, SOCIOLOGY, CSAT BY VISITING THE CHANNEL (INTEGRATED MARATHON FOR 2024)
https://t.me/UPSCPrelimsWithNeelesh ©Copyright For Queries Whatsapp - 9310161970
25 With reference to lndian biodiversity, Ceylon a All of the above are the species of birds.
frogmouth, Coppersmith barbet, Gray-chinned
minivet and White-throated redstart are: (2020)
(a) Birds
(b) Primates
(c) Reptiles
(d) Amphibians

26 Consider the following animals: d Among the given choices, Hedgehog and Pangolin show this survival
1. Hedgehog characteristic where they roll up to protect their vulnerable parts.
2. Marmot About Hedgehog: They are a small mammal with short limbs and a body
3. Pangolin low to the ground. Their most distinctive characteristic is the thousands
To reduce the chance of being captured by of stiff, sharp spines – harder and sharper than those of a porcupine - that
predators, which of the above organisms rolls cover the animal's back and sides, like a pincushion filled with needles.
up/roll up and protects/ protect its/their This small, spiky animal can curl into a tight ball with spines sticking out
vulnerable parts? (2021) in all directions if disturbed or frightened and also they sleep in
(a) 1 and 2 only this position. The hedgehog can live in many different habitats, from
(b) 2 only desert to forest in Europe, Asia, Africa and New Zealand.
(c) 3 only
(d) 1 and 3 only About Pangolin: They are the only mammals wholly-covered in scales.
These are solitary, primarily nocturnal animals. A startled pangolin will
cover its head with its front legs, exposing its scales to any potential
predator. When touched or grabbed, it would roll up completely into a
ball, while the sharp scales on the tail can be used to lash out. Eight
species of pangolins are found on two continents- four in Africa and four
in Asia.
They range from Vulnerable to Critically Endangered. They are the most
trafficked mammal in the world as their meat is considered a delicacy and
pangolin scales are used in traditional medicine and folk remedies.

TELEGRAM - UPSC PRELIMS WITH NEELESH


Learn ETHICS, ESSAY, SOCIOLOGY, CSAT BY VISITING THE CHANNEL (INTEGRATED MARATHON FOR 2024)
https://t.me/UPSCPrelimsWithNeelesh ©Copyright For Queries Whatsapp - 9310161970
27 Consider the following fauna: (2023) b (1). The lion-tailed macaque is a rainforest dweller, often being found in
1. Lion-tailed Macaque the upper canopy of tropical moist evergreen forests or monsoon
2. Malabar Civet forests. It is diurnal, meaning it is active exclusively in daylight hours.
3. Sambar Deer Hence option 1 is not correct.

How many of the above are generally nocturnal or (2). The Malabar civet is considered nocturnal and so elusive that little is
most active after sunset? known about its biology and ecology apart from habitat use. Hence
(a) Only one option 2 is correct.
(b) Only two
(c) All three (3). Sambar deer are either crepuscular (active at dusk and dawn) or
(d) None nocturnal (active during the night) animals. Hence option 3 is correct.

28 lf you walk through the countryside, you are likely b Common Myna is a tropical bird with a strong territorial instinct, which
to see some birds stalking alongside the cattle to has adapted extremely well to urban environments. lt is often seen
seize the insects disturbed by their movement following the plough for earthworms, or with grazing cattle catching
through grasses. Which of the following is/are insects as they are disturbed. Occasionally it is found in large numbers
such bird/birds? (2014) hunting for insects after the monsoon.
1. Painted Stork
2. Common Myna
3. Black-necked Crane

Select the correct answer using the code given


below:
(a) 1 and 2 only
(b) 2 only
(c) 2 and 3 only
(d) 3 only

TELEGRAM - UPSC PRELIMS WITH NEELESH


Learn ETHICS, ESSAY, SOCIOLOGY, CSAT BY VISITING THE CHANNEL (INTEGRATED MARATHON FOR 2024)
https://t.me/UPSCPrelimsWithNeelesh ©Copyright For Queries Whatsapp - 9310161970
29 With reference to "Gucci" sometimes mentioned c Statement 1 is correct: Guchhi mushroom is a species of fungus in the
in the news, consider the following statements: family Morchellaceae of the Ascomycota. They are pale yellow in color
(2022) with large pits and ridges on the surface of the cap, raised on a large
1. It is a fungus. white stem.
2. It grows in some Himalayan Forest areas.
3. It is commercially cultivated in the Himalayan Statement 2 is correct: Found on the foothills of the Himalayas, It is
foothills of north-eastern India. known to be one of the most expensive mushrooms.

Which of the statements given above is/are? Statement 3 is not correct: These mushrooms usually grow in clusters on
(a) 1 only logs of decaying wood, leaves or humus soil. They may or may not grow
(b) 3 only in the same spot the next season, hence these mushrooms cannot be
(c) 1 and 2 cultivated commercially.
(d) 2 and 3

30 Consider the following statements: (2023) d Statement 1 is correct: Medicinal mushrooms such as shiitake, maitake,
1. Some mushrooms have medicinal properties. and reishi have been found to have antitumor and immunostimulant
2. Some mushrooms have psychoactive properties. properties. Men will probably reap health benefits simply from adding
3. Some mushrooms have insecticidal properties. shiitake, maitake, and reishi mushrooms—good sources of B vitamins,
4. Some mushrooms have bioluminescent fiber, and antioxidants—to the diet.
properties.
Statement 2 is correct: Psychoactive properties mean a drug or other
How many of the above statements are correct? substance that affects how the brain works and causes changes in mood,
(a) Only one awareness, thoughts, feelings, or behavior. Examples of psychoactive
(b) Only two substances include alcohol, caffeine, nicotine, marijuana, and certain
(c) Only three pain medicines. Psilocybin is a hallucinogenic chemical in certain
(d) All four mushrooms known as magic mushrooms. Eating mushrooms that
contain psilocybin can have a variety of effects, ranging from euphoria
to hallucinations. Individuals use psilocybin as a recreational drug. It can
provide feelings of euphoria and sensory distortion that are common to
hallucinogenic drugs, such as LSD (lysergic acid diethylamide).
TELEGRAM - UPSC PRELIMS WITH NEELESH
Learn ETHICS, ESSAY, SOCIOLOGY, CSAT BY VISITING THE CHANNEL (INTEGRATED MARATHON FOR 2024)
https://t.me/UPSCPrelimsWithNeelesh ©Copyright For Queries Whatsapp - 9310161970
Statement 3 is correct: Recently, an increasing number of mushrooms
have been found to contain insecticidal compounds.

Statement 4 is correct: Bioluminescence is the production and emission


of light by living organisms. Panellus stipticus is one of the brightest-
glowing bioluminescent mushrooms on Earth. These flat fungi, which
look like a collection of tiny fans growing on sticks, are a dull shade of
yellow-beige during the day, but they transform into dazzling
decorations after dark. A mushroom documentation project in the
forests of Northeast India has revealed not only 600 varieties of fungi,
but also led to a new discovery: a bioluminescent or light emitting variety
of mushroom. The new species — named Roridomyces phyllostachydis
— was first sighted on a wet August night near a stream in Meghalaya’s
Mawlynnong in East Khasi Hills district and later at Krang Shuri in West
Jaintia Hills district.

31 Consider the following statements: (2021) b Statement 1 is not correct: Moringa is indigenous to the Indian sub-
1. Moringa (drumstick tree) is a leguminous continent. It has 13 species growing in tropical and subtropical regions. It
evergreen tree. is a non-leguminous evergreen tree.
2. The Tamarind tree is endemic to South Asia.
3. ln India, most of the tamarind is collected as Statement 2 is not correct: Tamarind (Tamarindus indica), is an
minor forest produce. evergreen tree native to tropical Africa. It is widely cultivated in tropical
4. India exports tamarind and seeds of moringa. and subtropical regions for its edible fruit, the sweet and sour pulp of
5. Seeds of moringa and tamarind can be used in which is extensively used in foods, beverages, and traditional medicines.
the production of biofuels. Statement 3 is correct: According to Forest Rights Act, 2006, "minor
Which of the statements given above are correct? forest produce" includes all nontimber forest produce of plant origin
(a) 1,2,4 and 5 Only including bamboo, brush wood, stumps, cane, tassar, cocoons, honey,
(b) 3, 4 and 5 Only tamarind, wax, lac, tendu or kendu leaves, medicinal plants and herbs,
(c) 1, 3 and 4 Only roots, tubers and the like.
(d) 1 ,2,3 and 5 Only
TELEGRAM - UPSC PRELIMS WITH NEELESH
Learn ETHICS, ESSAY, SOCIOLOGY, CSAT BY VISITING THE CHANNEL (INTEGRATED MARATHON FOR 2024)
https://t.me/UPSCPrelimsWithNeelesh ©Copyright For Queries Whatsapp - 9310161970
Statement 4 is correct: Moringa and tamarind has been in great demand
in many countries; The major countries that import moringa leaves and
tamarind seeds from India are the US, Germany, China, Canada, South
Korea, and other European countries
Statement 5 is correct: Tamarind and moringa seeds can be used to
generate biofuels.

32 Recently, there was a growing awareness in our d Girardinia diversifolia, which is commonly known as Himalayan Nettle or
country about the importance of Himalayan nettle Nilghiri nettle, is a wild shrub available naturally in the foothills of
(Girardinia diversifolia) because it is found to be a Himalayas. It is found abundantly in river sides, forest land and parts like
sustainable source of: (2019) Himachal Pradesh, Jammu & Kashmir, Nepal and also in parts of China.
(a) anti-malarial drug It is a shade tolerant, tall, stout and erect herb growing up to 3m height
(b) biodiesel with perennial rootstock.
(c) pulp for paper industry
(d) textile fibre Properties of Himalayan Nettle:
- According to the Centre for Sustainable Fashion, Himalayan Nettle is the
longest fiber and is considered finer, stronger, and more elastic than
linen.
- Fibers are best suitable for textile production. These fibers are hollow,
which creates natural insulation. It has greater resistance to wrinkling
and antimicrobial properties.
Hence, option (d) is the correct answer.

33 What would happen if phytoplankton of an ocean a Phytoplankton are similar to terrestrial plants in that they contain
is completely destroyed for some reason? (2012) chlorophyll and require sunlight in order to live and grow. Phytoplankton
1. The oceans a carbon sink would be adversely also requires inorganic nutrients such as nitrates, phosphates, and sulfur
affected. which they convert into proteins, fats, and carbohydrates. The two main
2. The food chains in the ocean would be classes of phytoplankton are dinoflagellates and diatoms.
adversely affected. Statement 1 is correct: Phytoplankton is responsible for most of the
transfer of carbon dioxide from the atmosphere to the ocean. Carbon
TELEGRAM - UPSC PRELIMS WITH NEELESH
Learn ETHICS, ESSAY, SOCIOLOGY, CSAT BY VISITING THE CHANNEL (INTEGRATED MARATHON FOR 2024)
https://t.me/UPSCPrelimsWithNeelesh ©Copyright For Queries Whatsapp - 9310161970
3. The density of ocean water would drastically dioxide is consumed during photosynthesis, and the carbon is
decrease. incorporated into the phytoplankton, just as carbon is stored in the wood
Select the correct answer using the codes given and leaves of a tree. Hence the destruction of Phytoplankton would affect
below: the carbon sequestration process in the ocean.
(a) 1 and 2 only
(b) 2 only Statement 2 is correct: Phytoplankton is the foundation of the aquatic
(c) 3 only food web, the primary producers, feeding everything from microscopic,
(d) 1,2 and 3 animal like zooplankton to multi-ton whales. Small fish and invertebrates
also graze on the plant like organisms, and then those smaller animals
are eaten by bigger ones. Thus, any destruction of Phytoplankton would
adversely affect the food chain.
Statement 3 is not correct: There is no direct relation between the
density of ocean water and Phytoplankton. The destruction would have
some effect on density but it would not drastically decrease. Hence,
option (a) is the correct answer.

34 Vultures which used to be very common in Indian b Diclofenac is a nonsteroidal anti-inflammatory drug (NSAID). This
countryside some years ago are rarely seen medicine works by reducing substances in the body that cause pain and
nowadays. This is attributed to: (2012) inflammation. Diclofenac is usually used to treat cattle. When these cattle
(a) the destruction of their nesting sites by new die, the vultures that feed on the carcasses die of renal failure from
invasive species diclofenac poisoning. Vultures are among the top predators and are
(b) a drug used by cattle owners for treating their enlisted in the critically endangered category in the IUCN Red List of
diseased cattle Threatened Species. Hence, option (b) is the correct answer.
(c) scarcity of food available to them
(d) a widespread, persistent and fatal disease
among them

35 ln the context of ecosystem productivity, marine b Prevailing winds, blowing from the land towards the oceans, drive warm
upwelling zones are important as they increase surface water away from the coast resulting in the upwelling of cold
the marine productivity by bringing the: (2011)
TELEGRAM - UPSC PRELIMS WITH NEELESH
Learn ETHICS, ESSAY, SOCIOLOGY, CSAT BY VISITING THE CHANNEL (INTEGRATED MARATHON FOR 2024)
https://t.me/UPSCPrelimsWithNeelesh ©Copyright For Queries Whatsapp - 9310161970
1. decomposer microorganisms to the surface. water from below. Winds blowing across the ocean surface push water
2. nutrients to the surface. away. It occurs in the open ocean and along coastlines.
3. bottom dwelling organisms to the surface.
Which of the statements given above is/are Impacts of Upwelling:
correct? - It influences sea-surface temperature and biological productivity.
(a) 1 and 2 - It lowers the temperature of ocean water as the water that rises to the
(b) 2 only surface as a result of upwelling is typically colder.
(c) 2 and 3 - It also fertilizes surface waters because upwelling brings the nutrients
(d) 3 only from the deep ocean to the surface. Hence statement 2 is correct.
- Good fishing grounds typically are found where upwelling is common.
Example: Peruvian Coast.
- It does not bring decomposer microorganisms to the surface. Organisms
also remain where they are. Upwelling only brings nutrients to the
surface. Hence, statements 1 and 3 are not correct.

36 Which one of the following is a filter feeder? c Filter feeders are a subgroup of suspension feeding animals that feed
(2021) upon suspended particles in the water column. Then generally pass water
(a) Catfish over a specialized filtering structure. Filter feeders can play an important
(b) Octopus role in clarifying water, and are therefore considered ecosystem
(c) Oyster engineers.
(d) Pelican Oysters are among those prominent filter feeders. They are considered
as an important biofilter as they help in maintaining the functioning of
the ecological system.
Some other animals that use this method of feeding are clams, krill,
sponges, baleen whales, and many fish (including some sharks)
Some birds, such as flamingos and certain species of duck, are also filter
feeders. Hence, option (c) is the correct answer.

TELEGRAM - UPSC PRELIMS WITH NEELESH


Learn ETHICS, ESSAY, SOCIOLOGY, CSAT BY VISITING THE CHANNEL (INTEGRATED MARATHON FOR 2024)
https://t.me/UPSCPrelimsWithNeelesh ©Copyright For Queries Whatsapp - 9310161970
37 With reference to the role of biofilters in b Recirculating aquaculture systems (RAS) are unique engineered
Recirculating Aquaculture System, consider the ecosystems that minimize environmental perturbation by reducing
following statements: (2023) nutrient pollution discharge. They are indoor, tank-based systems in
1. Biofilters provide waste treatment by removing which fish are grown at high density under controlled environmental
uneaten fish feed. conditions. A biofilter system purify the water and remove or detoxify
2. Biofilters convert ammonia present in fish waste harmful waste products and uneaten feed. Biofilters use
to nitrate. microorganisms, which are capable of degrading many compounds, fixed
3. Biofilters increase phosphorus as nutrient for to an inorganic/organic medium to break down pollutants present
fish in water. including uneaten fish feed. Hence statement 1 is correct.

How many of the statements given above are Ammonia is removed from an aquarium system through the use of a
correct? biofilter. The biofilter provides a substrate on which nitrifying bacteria
(a) Only one grow. These nitrifying bacteria consume ammonia and produce nitrite,
(b) Only two which is also toxic to fish. Other nitrifying bacteria in the biofilter
(c) All three consume nitrite and produce nitrate. Hence statement 2 is correct.
(d) None
Biofilters are also used to remove phosphorus waste by-products
generated by fish. Thus, biofilters do not increase phosphorous content
in aquatic systems. Hence statement 3 is not correct.

TELEGRAM - UPSC PRELIMS WITH NEELESH


Learn ETHICS, ESSAY, SOCIOLOGY, CSAT BY VISITING THE CHANNEL (INTEGRATED MARATHON FOR 2024)
https://t.me/UPSCPrelimsWithNeelesh ©Copyright For Queries Whatsapp - 9310161970
TOPIC 4: PROTECTED AREA NETWORK, CONVERVATION EFFORTS ETC (Ch 15, 16, APPENDIX (Ramsar wetland, Tiger
Reserve, Elephant Reserve, Mike Sites, Biosphere Reserve, List of India’s Biosphere Reserve in UNESCO WNBR,
Natural World Heritage Sites, Biodiversity Heritage Sites, Sacred Groves, Mangroves Sites in India)

Total Number of Questions


asked from 2011-2023 25
My Instructions related to GENERAL INSTRUCTION
these chapters This is also the important section. You must have on tips – various Protected Area Network, associated
conditions, and also remember the Ramsar wetland, tiger reserves, elephant reserves etc. with states. Many
times, questions are asked to match with states or having the idea of the geography of the biosphere reserve.

CHAPTER SPECIFIC INSTRUCTIONS


CHAPTER 15 PROTECTED AREA NETWORK
This chapter is very important. UPSC has the tendency to ask many times like which activity is allowed in
which protected area or Which is in-situ, ex-situ etc. So, read this chapter full keeping in focus more about
Wildlife sanctuary, National Park, sacred groves, MAB, biosphere reserve, criteria of biodiversity hotspots,
and their distribution in India and world.

CHAPTER 16 CONSERVATION EFFORTS


Focus more on Tiger reserves, NTCA, pugmark is used for? (Remember - Every individual animal species has a
distinct pugmark and as such this is used for identification), Project Lion, MIKE Program, E-8 relates to, diclofenac
and meloxicam in vultures, Project one horned rhino, project snow leopard, crocodile project, project hangul,
project dolphin, what is SAWEN and its members

APPENDICES
Remember the list with states. Most important ones are Ramsar sites, Tiger Reserves, Biosphere Reserves,
List of India’s Biosphere Reserves in UNESCO WNBR, Mangrove sites in India
.
TELEGRAM - UPSC PRELIMS WITH NEELESH
Learn ETHICS, ESSAY, SOCIOLOGY, CSAT BY VISITING THE CHANNEL (INTEGRATED MARATHON FOR 2024)
https://t.me/UPSCPrelimsWithNeelesh ©Copyright For Queries Whatsapp - 9310161970
1 The most important strategy for the conservation a This is one of the easiest questions. If you do not know the difference
of biodiversity together with traditional human between the limitations of these or provision of these, refer your original
life is the establishment of (2014) resources and read it properly
(a) biosphere reserves
(b) botanical gardens
(c) national parks
(d) wildlife sanctuaries

2 ln which one among the following categories of b No human activity is permitted inside the national park except for the
protected areas in India are local people not ones permitted by the Chief Wildlife Warden of the stale under the
allowed to collect and use the biomass? (2012) conditions given in CHAPTER IV WPA 1972. Hence, option (b) is the
(a) Biosphere Reserves correct answer.
(b) National Parks
(c) Wetlands declared under Ramsar Convention
(d) Wildlife Sanctuaries

3 Which one of the following is not a site for in-situ b In-situ method of conservation is the method of conserving all the living
method of conservation of flora? (2011) species, especially the wild and endangered species in their natural
(a) Biosphere Reserve habitats and environment. ln-situ conservation of Biodiversity includes
(b) Botanical Garden Biosphere Reserves, National Parks, Wildlife Sanctuaries etc.
(c) National Park
(d) Wildlife Sanctuary Ex-situ method of conservation of all the living species in the artful
habitats that reflect their natural living habitats. Examples of ex-situ
conservation of biodiversity include Aquariums, Botanical gardens,
Cryopreservation, DNA banks, Zoos, etc. Hence, option (b) is the correct
answer.

TELEGRAM - UPSC PRELIMS WITH NEELESH


Learn ETHICS, ESSAY, SOCIOLOGY, CSAT BY VISITING THE CHANNEL (INTEGRATED MARATHON FOR 2024)
https://t.me/UPSCPrelimsWithNeelesh ©Copyright For Queries Whatsapp - 9310161970
4 Consider the following statements: (2023) c Community reserves in India are terms denoting protected areas of India
Once the Central Government notifies an area as a which typically act as buffer zones to or connectors and migration
'Community Reserve' corridors between established national parks, wildlife sanctuaries and
1. the Chief Wildlife Warden of the State becomes reserved and protected forests of India.
the governing authority of such forest
2. hunting is not allowed in such area Statement 1 is correct: As per the Section 33 of the WildLife Protection
3. people of such area are allowed to collect non- Act (WLPA), once the Centre notifies an area as a community reserve,
timber forest produce the Chief Wildlife Warden of the state becomes the governing authority
4. people of such area are allowed traditional of the forest, whose consent is required for all decisions pertaining to
agricultural practices the area.

How many of the above statements are correct? Statement 2 is correct: After a forest has been made into a community
(a) Only one reserve, people are not allowed to hunt there, thus hunting is prohibited
(b) Only two in community reserves.
(c) Only three
(d) All four Statements 3 is correct and 4 is incorrect: The people of community
reserves are allowed to collect non-timber forest produce and those
people are not allowed to use community reserves for traditional
agricultural practices such as shifting (jhum) cultivation.

5 Three of the following criteria have contributed to c According to Conservation International at present, there are 36
the recognition of Western Ghats-Sri Lanka and biodiversity rich areas in the world that have been qualified as hotspots,
Indo-Burma regions as hotspots of biodiversity: which represent just 2.5% of earth's land surface, but support over 50%
(2011) of the world's endemic plant species, and nearly 43% of bird, mammal,
1. Species richness reptile and amphibian species as endemics.
2. Vegetation density Conservation International has laid down- 2 quantitative criteria, for a
3. Endemism region to qualify as a hotspot:
4. Ethno-botanical importance (i) It must contain at least 1,500 species of vascular plants (> 0.5% of the
5. Threat perception world's total) as endemics;

TELEGRAM - UPSC PRELIMS WITH NEELESH


Learn ETHICS, ESSAY, SOCIOLOGY, CSAT BY VISITING THE CHANNEL (INTEGRATED MARATHON FOR 2024)
https://t.me/UPSCPrelimsWithNeelesh ©Copyright For Queries Whatsapp - 9310161970
6. Adaptation of flora and fauna to warm and (ii) It has to have lost >70% of its original native habitat. Hence, option
humid conditions (c) is the correct answer.
Which three of the above are correct criteria in this
context?
(a) 1,2and 6 only
(b) 2, 4and 6 only
(c) 1,3 and 5 only
(d) 3,4 and 6 only

6 With reference to lndia's Desert National Park, c Statement 1 is correct: Desert National Park is spread over two districts
which of the following statements are correct? of Raiasthan -Jaisalmer and Barmer over an area of more than 3000 km
(2020) square.
1. lt is spread over two districts. It is the largest national park of Rajasthan. It displays the best of the Thar
2. There is no human habitation inside the Park. desert's ecosystem and its varied wildlife.
3. lt is one of the natural habitats of the Great
lndian Bustard. Statement 2 is not correct: Human habitation, although scarce, is not
entirely absent within the Desert National Park Density is as low as (4-5
Select the correct answer using the code given persons per km2). There are numerous villages and also settlements or
below: Dhanis within the Park. They have been inhabited by communities for
(a) 1 and 2 only hundreds of years and with their rich culture and
(b) 2 and 3 only tradition they have become an integral part of the ecosystem.
(c) 1 and 3 only
(d) 1, 2 and 3 Statement 3 is correct: Various species of animals such as black buck,
chinkara and desert fox inhabit the Park. The highly endangered Great
lndian Bustard, one of the world's heaviest flying birds, can also be seen
here.

7 Which one of the following National Parks lies d Valley of Flowers National Park lies completely in the temperate alpine
completely in the temperate alpine zone? (2019) zone. It is located in Chamoli region, Uttarakhand
(a) Manas National Park
TELEGRAM - UPSC PRELIMS WITH NEELESH
Learn ETHICS, ESSAY, SOCIOLOGY, CSAT BY VISITING THE CHANNEL (INTEGRATED MARATHON FOR 2024)
https://t.me/UPSCPrelimsWithNeelesh ©Copyright For Queries Whatsapp - 9310161970
(b) Namdapha National Park
(c) Neora Valley National Park
(d) Valley of Flowers National Park

8 Which one of the following National Parks has a d Namdapha National Park is located in Arunachal Pradesh. The climate of
climate that varies from tropical to subtropical, this region varies from tropical to subtropical temperate and arctic. lt is
temperate and arctic? (2015) tropical and subtropical in southern regions and an arctic-type climate is
(a) Khangchendzonga National Park found in the northern part of the park.
(b) Nanda Devi National Park It is the only park in the World to have the four Feline species of big cat
(c) Neora Valley National Park namely the Tiger (Panthera Tigris), Leopard (Panthera Pardus), Snow
(d) Namdapha National Park Leopard (Panthera Uncia) and Clouded Leopard (Neofelis Nebulosa) and
numbers Lesser cats. Hence, option (d) is the correct answer.

9 From the ecological point of view, which one of a Sathyamangalam Tiger Reserve is a wildlife corridor in the Nilgiri
the following assumes importance in being a good Biosphere Reserve between the Western Ghats and the Eastern Ghats. lt
link between the Eastern Ghats and the Western is located in the Erode District of Tamil Nadu.
Ghats? (2017)
(a) Sathyamangalam Tiger Reserve
(b) Nallamala Forest
(c) Nagarhole National Park
(d) Seshachalam Biosphere Reserve

10 ln which one of the following States is Pakhui a Pakhui Wildlife Sanctuary is located in Arunachal Pradesh.
Wildlife Sanctuary located? (2018)
(a) Arunachal Pradesh
(b) Manipur
(c) Meghalaya
(d) Nagaland

TELEGRAM - UPSC PRELIMS WITH NEELESH


Learn ETHICS, ESSAY, SOCIOLOGY, CSAT BY VISITING THE CHANNEL (INTEGRATED MARATHON FOR 2024)
https://t.me/UPSCPrelimsWithNeelesh ©Copyright For Queries Whatsapp - 9310161970
11 Which of the following Protected Areas are c Except Papikonda National Park, which is located in West Godavari
located in the Cauvery basin? (2020) District of Andhra Pradesh, all the given protected areas are located in
1. Nagarhole National Park the Cauvery Basin
2. Papikonda National Park
3. Sathyamangalam Tiger Reserve
4. Wayanad Wildlife Sanctuary
Select the correct answer using the code given
below:
(a) I and 2 only
(b) 3 and 4 only
(c) 1, 3 and 4 only
(d) 1,2,3 and 4

12 Which of the following are in Agasthyamala a Agasthyamala Biosphere Reserve is located in the Western Ghats in the
Biosphere Reserve? (2019) south of the country. It encompasses tropical forest ecosystems that fall
(a) Neyyar, Peppara and Shendurney Wildlife within the Tirunelveli and Kanyakumari districts of Tamil Nadu and the
Sanctuaries; and Kalakad Mundanthgrai Tiger Thiruvananthapuram and Kollam districts of Kerala.
Reserve. Three wildlife sanctuaries, Shendurney, Peppara and Neyyar, are located
(b) Mudumalai, Sathyamangalam and Wayanad in the site, as well as the Kalakad Mundanthurai Tiger reserve. Hence,
Wildlife Sanctuaries; and Sitent Valley National Park option (a) is the correct answer.
(c) Kaundinya, Gundla Brahmeswaram and
Papikonda Wildlife Sanctuaries; and Mukurthi
National Park.
(d) Kawal and Sri Venkateswara Wildlife
Sanctuaries; and Nagarjunasagar-Srisailam Tiger
Reserve

13 lf you want to see gharials in their natural habitat, b Gharials are a type of Asian crocodilian distinguished by their long, thin
which one of the following is the best place to snouts. Gharials live in clear freshwater river systems. Chambal River is a
visit? (2017)
TELEGRAM - UPSC PRELIMS WITH NEELESH
Learn ETHICS, ESSAY, SOCIOLOGY, CSAT BY VISITING THE CHANNEL (INTEGRATED MARATHON FOR 2024)
https://t.me/UPSCPrelimsWithNeelesh ©Copyright For Queries Whatsapp - 9310161970
(a) Bhitarkanika Mangroves shelter of rare crocodile species "Gharial' and the National Aquatic
(b) Chambal River Animal Gangetic Dolphin.
(c) Pulicat Lake
(d) Deepor Beel

14 Which one of the following is the national aquatic c Gangetic Dolphin is the national aquatic animal of India
animal of lndia? (2015)
(a) Saltwater crocodile
(b) Olive ridley turtle
(c) Gangetic dolphin
(d) Gharial

15 Which one of the following protected areas is a Swamp deer (Barasingha) is the state animal of Madhya Pradesh, and is
well-known for the conservation of a sub-species found in the wild largely in and around Kanha National Park. lt remained
of the lndian swamp deer (Barasingha) that close to extinction for a long time. However, as a result of persistent
thrives well on hard ground and is exclusively conservation work involving habitat improvement and captive breeding,
graminivorous? (2020) there has been a revival in its population in Kanha National Park.
(a) Kanha National Park
(b) Manas National Park
(c) Mudumalai Wildlife Sanctuary
(d) Tal Chhaper Wildlife Sanctuary

16 Which of the following National Parks is unique in b Keibul Lamjao National Park is located in the south western part of the
being a swamp with floating vegetation that Loktak Lake in Manipur. lt is the only floating park in the world. lt is
supports a rich biodiversity? (2015) famous for its unique floating vegetation named "Phumdi". Keibul
(a) Bhitarkanika National Park Lamjao National Park is the Iast natural habitat of the brow-antlered deer
(b) Keibul Lamjao National Park (Sangai), the dancing deer of Manipur. Keibul Lamjao National Park was
(c) Keoladeo Ghana National Park first recognized as a wildlife sanctuary back in 1966, a safe home for the
(d) Sultanpur National Park brow-antlered deer, locally known as the Sangai deer . Later, it became a
national park in 1977.
TELEGRAM - UPSC PRELIMS WITH NEELESH
Learn ETHICS, ESSAY, SOCIOLOGY, CSAT BY VISITING THE CHANNEL (INTEGRATED MARATHON FOR 2024)
https://t.me/UPSCPrelimsWithNeelesh ©Copyright For Queries Whatsapp - 9310161970
17 Which of the following are the most likely places a The Himalayan musk deer is a small primitive cervid (mammal of the deer
to find the musk deer in its natural habitat? (2020) family). The unique feature of this deer is the presence of the musk gland
1. Askot Wildlife Sanctuary in the male whose ingredients are valued for their cosmetic and alleged
2. Gangotri National Park pharmaceutical properties.
3. Kishanpur Wildlife Sanctuary ln the IUCN red list of threatened Species it is listed under "endangered
4. Manas National Park species". Poaching of the musk pod and habitat destruction are two
Select the correct answer using the code given major causes behind its vulnerability.
below: Himalayan musk deer reside in the Himalayan mountain range,
(a) 1 and 2 only particularly within the countries of Bhutan, lndia, Nepal, and a small part
(b) 2 and 3 only of China.
(c) 3 and 4 only Among the given choices, it can be found in Askot wildlife sanctuary
(d) 1 and 4 only (Uttrakhand) and Gangotri National Park (Uttrakhand) only.

Hence, a is the correct answer

18 Recently there was a proposal to translocate some b The central expert committee in 2016 approved the translocation of
of the lions from their natural habitat in Gujarat to Asiatic Lions from Gir National Park in Gujarat to Kuno Sanctuary in
which one of the following sites? (2017) Madhya Pradesh.
(a) Corbett National Park Experts cited that Gir has become overcrowded with lions and there is
(b) Kuno Palpur Wildlife Sanctuary need to spread them out to other locations to ensure their genetic
(c) Mudumalai Wildlife Sanctuary stability and health. Hence, option (b) is the correct answer.
(d) Sariska National Park

19 ln which of the following States is lion-tailed a Lion tailed macaque is one of the smallest macaque species.
macaque found in its natural habitat? (2013)
1. Tamil Nadu Distribution: lt is endemic to the Western Ghats hill ranges in
2. Kerala southwestern lndia in the states of Karnataka, Kerala, and Tamil Nadu.
3. Karnataka Hence, (a) is the correct answer.
4. Andhra Pradesh

TELEGRAM - UPSC PRELIMS WITH NEELESH


Learn ETHICS, ESSAY, SOCIOLOGY, CSAT BY VISITING THE CHANNEL (INTEGRATED MARATHON FOR 2024)
https://t.me/UPSCPrelimsWithNeelesh ©Copyright For Queries Whatsapp - 9310161970
Select the correct answer using the codes given Habitat: Broadleaf trees, growing in monsoon forests as well as
below: evergreen and semi-evergreen rainforests. lt is mainly arboreal,
(a) 1,2 and 3 only therefore, prefers the upper canopy of primary tropical evergreen
(b) 2 only rainforest referred to as Sholas.
(c) 1,3 and 4 only
(d) 1,2,3 and 4 As a result of its shy and solitary nature, this animal doesn't tend to
venture from its usual range, travelling only within its rainforest habitat.
Like all other macaques, lion tailed macaques have cheek pouches which
can store the same amount of food as their stomach.
Protection Status:
IUCN Red List of Threatened Species: Endangered.
CITES: Appendix-l
Wildlife Protection Act,1972: Schedule – I

20 Consider the following protected areas: (2012) b As per National Tiger Conservation Authority (NTCA)'s data, Bandipur,
1. Bandipur Manas and Sundarbans have been declared as tiger reserves in India.
2. Bhitarkanika Bhitarkanika National Park has not been declared as a tiger reserve.
3. Manas You are advised to prepare the list of tiger reserve with their states and
4. Sunderbans learn it properly

Which of the above are declared Tiger Reserves?


(a) 1 and 2 only
(b)1, 3 and 4 only
(c) 2, 3 and 4 only
(d) 1,2,3 and 4

21 Among the following Tiger Reserves, which one c Critical Tiger Habitats are also known as core areas of tiger reserves. They
has the largest area under "Critical Tiger Habitat"? are identified under the Wild Life Protection Act (WLPA), 1972 based on
(2020) scientific evidence that "such areas are required to be kept as inviolate
(a) Corbett
TELEGRAM - UPSC PRELIMS WITH NEELESH
Learn ETHICS, ESSAY, SOCIOLOGY, CSAT BY VISITING THE CHANNEL (INTEGRATED MARATHON FOR 2024)
https://t.me/UPSCPrelimsWithNeelesh ©Copyright For Queries Whatsapp - 9310161970
(b) Ranthambore for the purpose of tiger conservation, without affecting the rights of the
(c) Nagarjunsagar-Srisailam Scheduled Tribes or such other forest dwellers".
(d) Sunderbans
Nagarjunsagar- Srisailam Tiger reserve spread over the parts Andhra
Pradesh and Telangana has the largest area under "Critical Tiger Habitat"

22 The term 'M-STRIPES' is sometimes seen in the b M-STrIPES, short for Monitoring System for Tigers - Intensive Protection
news in the context of (2017) and Ecological Status, is a software-based monitoring system launched
(a) Captive breeding of Wild Fauna across Indian tiger reserves by the Indian government's National Tiger
(b) Maintenance of Tiger Reserves Conservation Authority (NTCA) in 2010
(c) lndigenous Satellite Navigation System
(d) Security of National Highways

23 Consider the following statements: (2018) a Statement 1 is correct: The Critical Wildlife Habitats have been defined
1. The definition of "Critical Wildlife Habitat" is in Scheduled Tribes and Other Traditional Forest Dwellers (Recognition of
incorporated in the Forest Rights Act, 2006. Forest Rights) Act, 2006. The Act defines Critical Wildlife Habitats as areas
2. For the first time in lndia, Baigas have been "required to be kept inviolate for the purposes of wildlife conservation".
given Habitat Rights. These areas are to be identified within national parks and sanctuaries on
3. Union Ministry of Environment, Forest and a case by case basis.
Climate Change officially decides and declares
Habitat Rights for Primitive and Vulnerable Tribal Statement 2 is correct: Baiga is one of the Particularly Vulnerable Tribal
Groups in any part of India. Groups living in central lndia. ln 2016, the government of Madhya
Pradesh for the first time recognised the habitat rights of seven villages
Which of the statements given above is/are in Dindori district, which are mostly inhabited by the Baigas.
correct?
(a) 1 and 2 only Statement 3 is not correct - As per the Scheduled Tribes and Other
(b) 2 and 3 only Traditional Forest Dwellers (Recognition of Forest Rights) Act, 2006, the
(c) 3 only Ministry of Environment and Forests has been identified as the agency to
(d) 1,2 and 3 determine and notify Critical Wildlife Habitats. The Ministry of Tribal

TELEGRAM - UPSC PRELIMS WITH NEELESH


Learn ETHICS, ESSAY, SOCIOLOGY, CSAT BY VISITING THE CHANNEL (INTEGRATED MARATHON FOR 2024)
https://t.me/UPSCPrelimsWithNeelesh ©Copyright For Queries Whatsapp - 9310161970
Affairs is the nodal ministry for recognition and vesting of individual and
community forest rights.

24 Consider the following pairs (2022) b Check the location from your original source and remember the wetland
Wetland Lake Location and the state. It is often asked in UPSC.
1. Hokera Wetland Punjab Renuka Wetland is in Himachal Pradesh, Rudrasagar Lake is in Tripura.
2. Renuka Wetland Himachal Pradesh Rest are not correctly matched. Hokera Wetland is in Jammu and Kashmir
3. Rudrasagar Lake Tripura and Sasthamkotta Lake is in Kerala.
4. Sasthamkotta Lake Tamil Nadu

How many pairs given above are correctly


matched?
(a) Only one pair
(b) Only two pairs
(c) Only three pairs
(d) All four pairs

25 Consider the following pairs (2014) a Pair 1 is correct: Harike is one of the largest man- made wetlands of
northern India. It shares its area with the Tarntaran, Ferozpur, and
Wetlands Confluence of rivers Kapurthala districts of Punjab. It came into existence in 1952 after the
1. Harike Wetlands Confluence of Beas construction of a barrage near the confluence of rivers Sutlej and Beas.
and Satluj/Sutlej It has an area of about 86 km2. The site was accorded wetland status in
2.Keoladeo Ghana Confluence of Banas 1990 by the Ramsar Convention.
National Park and Chambal Pair 2 is not correct: Keoladeo Ghana National Park was formerly known
as the Bharatpur Bird Sanctuary. It is a famous avifauna sanctuary in
3. Kolleru Lake Confluence of Musi Bharatpur, Rajasthan. Initially, it was a natural depression and was
and Krishna flooded after the Ajan Bund was constructed by Maharaja Suraj Mal. The
bund was created at the confluence of two rivers, the Gambhir and
Banganga. Keoladeo National Park was listed as a World Heritage Site by
UNESCO in 1985.
TELEGRAM - UPSC PRELIMS WITH NEELESH
Learn ETHICS, ESSAY, SOCIOLOGY, CSAT BY VISITING THE CHANNEL (INTEGRATED MARATHON FOR 2024)
https://t.me/UPSCPrelimsWithNeelesh ©Copyright For Queries Whatsapp - 9310161970
Which of the above pairs is/are correctly Pair 3 is not correct: Kolleru Lake is the largest freshwater lake and is
matched? located in Andhra Pradesh. Kolleru is located between Krishna and
a. 1 only Godavari delta and covers an area of 308 km2. Hence, option (a) is the
b. 2 and 3 only correct answer.
c. 1 and 3 only
d. 1, 2 and 3

TELEGRAM - UPSC PRELIMS WITH NEELESH


Learn ETHICS, ESSAY, SOCIOLOGY, CSAT BY VISITING THE CHANNEL (INTEGRATED MARATHON FOR 2024)
https://t.me/UPSCPrelimsWithNeelesh ©Copyright For Queries Whatsapp - 9310161970
TOPIC 5: CLIMATE CHANGE (Ch 17 - Climate Change, Ch 18 – Ocean Acidification, Ch 19 – Ocean Depletion, Ch 20 –
Impact of Climate Change – India, Ch 21 – Mitigation Strategies)
Total Number of Questions
asked from 2011-2023 18
My Instructions related to GENERAL INSTRUCTION
these chapters This section is also important. But instead of mugging up facts, develop an understanding. It will help you to
recall and attempt the right answer

CHAPTER SPECIFIC INSTRUCTIONS


CHAPTER 17 CLIMATE CHANGE
This chapter is fully important. Read it full

CHAPTER 18 OCEAN ACIDIFICATION


This chapter is also important. Know the reasons of ocean acidification – it is due to carbon dioxide only (not
nitrogen compounds, nor Sulphur compounds).

CHAPTER 19 OZONE DEPLETION


Know the reasons of ozone depletion – this is important and also the effects it would produce

CHAPTER 20 IMPACT OF CLIMATE CHANGE


There is no need to remember things word by word. But you must have the general understanding of the
effects of climate change

CHAPTER 21 MITIGATION STRATEGIES


Know about carbon sequestration (basics), carbon sink (meaning), what is blue carbon, what is carbon credit,
what is carbon offsetting, what is carbon tax, what is geo-engineering

TELEGRAM - UPSC PRELIMS WITH NEELESH


Learn ETHICS, ESSAY, SOCIOLOGY, CSAT BY VISITING THE CHANNEL (INTEGRATED MARATHON FOR 2024)
https://t.me/UPSCPrelimsWithNeelesh ©Copyright For Queries Whatsapp - 9310161970
1 The increasing amount of carbon dioxide in the d Infrared part of solar radiation is related to temperature increase. And
air is slowly raising the temperature of the hence d is the answer.
atmosphere, because it absorbs (2012)
(a) the water vapor of the air and retains its heat
(b) the ultraviolet part of the solar radiation
(c) all the solar radiations
(d) the infrared part of the solar radiation

2 The scientific view is that the increase in global b Intergovernmental Panel on Climate Change (IPCC) is the United Nations
temperature should not exceed 2oC above pre- body for assessing the science related to climate change. As per the IPCC
industrial level. If the global temperature special report, the rise of 3'C in global temperatures above pre-industrial
increases beyond 3oC above the pre-industrial levels by 2100 would be disastrous. Its effects would be felt differently
level, what can be its possible impact/impacts on around the world, but nowhere would be immune. Prolonged heat waves,
the world? (2014) droughts, and extreme weather events could all become increasingly
l. Terrestrial biosphere tends toward a net carbon common and severe.
source. Impacts of the rise of the global temperature beyond 3'C above the pre-
2. Widespread coral mortality will occur. industrial level: (As per IPCC data) are -
3. All the global wetlands will permanently -Impacts on Biodiversity and Ecosystems: Loss and Extinction of insects,
disappear. plants and vertebrates.
4. Cultivation of cereals will not be possible Due to the loss of forests and their ecological productivity, the terrestrial
anywhere in the world biosphere will tend toward a net carbon source. Hence statement 1 is
correct.
Select the correct answer using the code given
below: - Impact on precipitation: More of Earth's land areas will also be affected
(a) 1 only by flooding and increased runoff. Heavy rainfall from tropical cyclones is
(b) 1 and 2 only projected to be higher. Sea level will continue to rise, as heat already
(c) 2,3, and 4 only stored in the oceans from human-produced warming
(d) 1,2,3, and 4 causes them to expand.

TELEGRAM - UPSC PRELIMS WITH NEELESH


Learn ETHICS, ESSAY, SOCIOLOGY, CSAT BY VISITING THE CHANNEL (INTEGRATED MARATHON FOR 2024)
https://t.me/UPSCPrelimsWithNeelesh ©Copyright For Queries Whatsapp - 9310161970
Ocean warming, acidification, and more intense storms will cause coral
reefs to decline. Hence statement 2 is correct.

New ecosystems will appear, and there will be more damage to marine
ecosystems. Fisheries and aquaculture (wetlands) will be less productive
(but won't disappear permanently). Hence statement 3 is not correct.

Loss of mangrove trees increases at both temperature thresholds,


reducing their ability to serve as natural barriers that provide coastal
protection from storms, rising seas, and waves.
Rise in temperature will impact the crop yield in negative ways, but it does
not mean that the cultivation of cereals will not be possible anywhere in
the world. Hence statement 4 is not correct.

3 Among the following crops, which one is the b Rice cultivation is the third highest source (17.5 percent) of Greenhouse
most important anthropogenic source of both gas (GHG) emissions in Indian agriculture after enteric fermentation (54.6
methane and nitrous oxide? (2022) percent) and fertilizer use (19 %). Paddy fields are anthropogenic sources
(a) Cotton of atmospheric nitrous oxide as well as methane, which have been
(b) Rice reckoned as Greenhouse gases with significantly higher Global Warming
(c) Sugarcane Potential than carbon dioxide.
(d) Wheat Methane emissions from rice: Rice is grown in flooded fields, which
create the ideal anaerobic conditions for bacteria to thrive on
decomposing organic matter (mainly rice straw residue) and release
methane. Methane is also emitted due to field burning of rice straws.
Nitrous oxide emissions from rice: Poor absorption by the rice plant of
nitrogen-based fertilizers, often overused by farmers, leads to nitrous
oxide emissions. Nitrous oxide is also emitted due to field burning of rice
straws. Hence, option (b) is the correct answer.

TELEGRAM - UPSC PRELIMS WITH NEELESH


Learn ETHICS, ESSAY, SOCIOLOGY, CSAT BY VISITING THE CHANNEL (INTEGRATED MARATHON FOR 2024)
https://t.me/UPSCPrelimsWithNeelesh ©Copyright For Queries Whatsapp - 9310161970
4 The formation of ozone holes in the Antarctic b The severe depletion of the Antarctic ozone layer known as the "ozone
region has been a cause of concern. What could hole" occurs because of the spectral atmospheric and chemical conditions
be the reason for the formation of this hole? in Antarctica. The very low winter temperatures in the Antarctic
(2011) stratosphere cause polar stratospheric clouds (PSCs) to form. Special
(a) Presence of prominent tropospheric reactions that occur on PSCs, combined with the relative isolation of polar
turbulence; and inflow of chlorofluorocarbons stratospheric air, allow chlorine and bromine reactions to produce the
(b) Presence of prominent polar front and ozone hole in Antarctic. Hence, option (b) is the correct answer
stratospheric clouds; and inflow of
chlorofluorocarbons
(c) Absence of polar front and stratospheric
clouds; and inflow of methane and
chlorofluorocarbons.
(d) Increased temperature at polar region due to
global warming

5 Consider the following statements: (2012) c Chlorofluorocarbons (CFCs) are nontoxic, nonflammable chemicals
Chlorofluorocarbons, known as ozone-depleting containing atoms of carbon, chlorine, and fluorine. They are used in the
substances, are used manufacture of aerosol sprays, blowing agents for foams and packing
1. in the production of plastic foams materials, as solvents, and as refrigerants.
2. in the production of tubeless tyres Chemical inertness insures against damage to the surfaces to be cleaned,
3. in cleaning certain electronic components. and their relatively high vapor pressure makes drying the surface after
4. as pressurizing agents in aerosol cans. cleaning simple hence CFCs are used extensively in the electronics and
aerospace industries to clean metal parts. Hence, option (c) is the correct
Which of the statements given above is/are
correct?
(a) 1,2 and 3 only
(b) 4 only
(c) 1, 3 and 4 only
(d) 1, 2, 3 and 4

TELEGRAM - UPSC PRELIMS WITH NEELESH


Learn ETHICS, ESSAY, SOCIOLOGY, CSAT BY VISITING THE CHANNEL (INTEGRATED MARATHON FOR 2024)
https://t.me/UPSCPrelimsWithNeelesh ©Copyright For Queries Whatsapp - 9310161970
6 Consider the following: (2023) d Hydrofluorocarbons (HFCs) are a group of industrial chemicals primarily
1. Aerosols used for cooling and refrigeration. HFCs are entirely man-made. They are
2. Foam agents primarily produced for use in refrigeration, air-conditioning, insulating
3. Fire retardants foams and aerosol propellants, with minor uses as solvents and for fire
4. Lubricants protection. Also, synthetic lubricants used in stationary HFC-based
In the making of how many of the above are refrigeration equipment.
hydrofluorocarbons used? (a) Only one
(b) Only two HFCs were developed to replace stratospheric ozone-depleting
(c) Only three substances that are currently being phased out under the Montreal
(d) All four Protocol on Substances that Deplete the Ozone Layer. Though HFCs
currently represent around 1% of total greenhouse gases, their impact
on global warming can be hundreds to thousands of times greater than
that of carbon dioxide per unit of mass. Kigali Amendment to phase
down HFCs under the Montreal Protocol entered into force in 2019.
Under the amendment, countries commit to cut the production and
consumption of HFCs by more than 80% over the next 30 years. Hence,
all the four options are correct.

7 Which one of the following statements best a According to the Organization for Economic Co-operation and
describes the term 'social Cost of Carbon’? (2020) Development (OECD), the Social Cost of Carbon is usually estimated as
(a) Long-term damage done by a tonne of carbon the net present value of climate change impacts over the next 100 years
dioxide emissions in a given year. (or longer) of one additional tonne of carbon emitted to the atmosphere
(b) Requirement of fossil fuels for a country to today. It is the marginal global damage costs of carbon emissions. Hence
provide goods and services to its citizens, based on Option (a) is the correct answer.
the burning of those fuels.
(c) Efforts put in by a climate refugee to adapt to
live in a new place.
(d) Contribution of an individual person to the
carbon footprint on the planet Earth.

TELEGRAM - UPSC PRELIMS WITH NEELESH


Learn ETHICS, ESSAY, SOCIOLOGY, CSAT BY VISITING THE CHANNEL (INTEGRATED MARATHON FOR 2024)
https://t.me/UPSCPrelimsWithNeelesh ©Copyright For Queries Whatsapp - 9310161970
8 In the context of which of the following do some d The increase of stratospheric sulphate aerosol and cirrus cloud thinning
scientists suggest the use of cirrus cloud thinning are methods to reduce global warming. A collaborative study by
technique and the injection of sulphate aerosol researchers from India, China, and the U.S. published in Geophysical
into stratosphere? (2019) Research Letters claimed that the combination of these methods would
(a) Creating the artificial rains in some regions. lead to reduce global warming and precipitation rates to pre-industrial
(b) Reducing the frequency and intensity of levels. Hence, Option (d) is the correct answer
tropical cyclones
(c) Reducing the adverse effects of solar wind on About stratospheric aerosol injection (SAI): lt is a theoretical solar
the Earth. geoengineering proposal to spray large quantities of tiny reflective
(d) Reducing the global warming. particles into the atmosphere. It includes spraying reflective particles,
such as sulphur-dioxides, finely powdered salt, or calcium carbonate, from
aircraft, shooting particles from artillery guns, or using large hoses to
reach the sky. This could reduce the warming to pre-industrial levels, but
would also reduce the precipitation at a much higher rate than required
to balance the effect of carbon dioxide.
Risk involved: Scientists have said SAI could cause droughts or extreme
weather in other parts of the world, harm crop yields as well as potential
public health and governance issues.
About cirrus cloud thinning: It is a solar geoengineering proposal that
aims to eliminate or: thin cirrus clouds to allow heat to escape into space.
It involves injecting ice nuclei such as bismuth triiodide or aerosol
particles as sulfuric or nitric acid into regions where cirrus clouds form.
Thinning the clouds could allow more heat to escape into space.
Risk involved: The injection of "too many" ice-nucleating particles into
cirrus clouds may lead
to more and thicker clouds being formed. Hence more heat would be
trapped. Hence, option (d) is the correct answer.

9 The acidification of oceans is increasing. Why is a Ocean acidification is the reduction in the pH of seawater as a
this phenomenon a cause of concern? (2013) consequence of the absorption of large amounts of carbon dioxide by the
TELEGRAM - UPSC PRELIMS WITH NEELESH
Learn ETHICS, ESSAY, SOCIOLOGY, CSAT BY VISITING THE CHANNEL (INTEGRATED MARATHON FOR 2024)
https://t.me/UPSCPrelimsWithNeelesh ©Copyright For Queries Whatsapp - 9310161970
1. The growth and survival of calcareous oceans. Ocean acidification is largely the result of loading Earth's
phytoplankton will be adversely affected. atmosphere with large quantities of CO2, produced by vehicles and
2. The growth and survival of coral reefs will be industrial and
adversely affected. agricultural processes.
3. The survival of some animals that have Statement 1 is correct: Ocean acidification decreases the calcifying ability
phytoplanktonic larvae will be adversely affected. of corals, calcareous plankton, crustaceans etc.
4. The cloud seeding and formation of clouds will Statement 2 is correct: Ocean acidification will affect corals and this will,
be adversely affected. in turn, affect one million species that have made corals their homes.
Statement 3 is correct: Coral reefs will erode faster than they can rebuild.
Which of the statement(s) given above is/are When shelled organisms are at risk, the entire food web may also be at
correct? risk.
(a) 1,2 and 3 only Statement 4 is not correct: Cloud seeding is an artificial way of inducing
(b) 2 only moisture in the clouds to cause rainfall. It improves a cloud's ability to
(c) 1 and 3 only produce rain or snow by introducing tiny ice nuclei into certain types of
(d) 1,2,3 and 4 subfreezing clouds. These nuclei provide a base for snowflakes to form.
There is no conclusive evidence that Ocean acidification adversely affects
cloud formation and cloud seeding.

10 What is blue carbon? (2021) a Blue carbon refers to the carbon stored in coastal and marine ecosystems.
(a) Carbon captured by oceans and coastal Blue carbon ecosystems include mangroves, tidal and salt marshes, and
ecosystems. seagrasses. They are highly productive coastal ecosystems that are
(b) Carbon sequestered in forest biomass and particularly important for their capacity to store carbon within the plants
agricultural soils. and in the sediments below. Hence, option (a) is the correct answer.
(c) Carbon contained in, petroleum and natural
gas.
(d) Carbon present in atmosphere

11 Which of the following statements best describes a Carbon fertilization refers to an increase in the photosynthesis process
"carbon fertilization"? (2018) under higher atmospheric carbon dioxide levels. The speeding-up of
photosynthesis leads to increased plant growth.
TELEGRAM - UPSC PRELIMS WITH NEELESH
Learn ETHICS, ESSAY, SOCIOLOGY, CSAT BY VISITING THE CHANNEL (INTEGRATED MARATHON FOR 2024)
https://t.me/UPSCPrelimsWithNeelesh ©Copyright For Queries Whatsapp - 9310161970
(a) Increased plant growth due to increased Photosynthesis is the process by which plants use sunlight, water, and
concentration of carbon dioxide in the carbon dioxide to create oxygen and energy in the form of sugar. While
atmosphere. sunlight is abundant, water, atmospheric Carbon dioxide, and other input
(b) Increased temperature of Earth due to factors are limited, thereby determining the scale at which photosynthesis
increased concentration of carbon dioxide in can occur. Thus, in theory, if more carbon di oxide is available, the
the atmosphere. photosynthesis process should scale up, leading to better plant growth.
(c) Increased acidity of oceans as a result-of Hence, option (a) is the correct answer.
increased concentration of carbon dioxide in the
atmosphere.
(d) Adaptation of all living beings on Earth to the
climate change brought about by the increased
concentration of carbon dioxide in the
atmosphere.

12 The "Miyawaki method" is well known for the: c Miyawaki method, which was devised by Japanese botanist Akira
(2022) Miyawaki in the 1980s, is a technique to create micro forests over small
(a) Promotion of commercial farming in arid and plots of land.
semi-arid areas Under this, dozens of native species are planted in the same area, close to
(b) Development of gardens using genetically each other, which ensures that the plants receive sunlight only from the
modified flora top and grow upwards than sideways It requires very little space (a
(c) Creation of mini forests in urban areas minimum of 20 square feet), plants grow ten times faster, and the forest
(d) Harvesting wind energy on coastal areas and becomes maintenance-free in three years. Hence, option (c) is the correct
on sea surfaces answer.

13 What is the use of biochar in farming? (2020) d The term "bio" is in reference to organic residues and "char" for charcoal.
1. Biochar can be used as a part of the growing Biochar is a high carbon form of charcoal. Biochar is the result of the
medium in vertical farming. thermal degradation of organic materials in the absence of air (pyrolysis).
2. When biochar is a part of the growing medium, Pyrolysis consists of heating biomass in an environment that totally or
it promotes the growth of nitrogen-fixing partly excludes oxygen. The lack of oxygen prevents complete combustion
microorganisms. of the material.
TELEGRAM - UPSC PRELIMS WITH NEELESH
Learn ETHICS, ESSAY, SOCIOLOGY, CSAT BY VISITING THE CHANNEL (INTEGRATED MARATHON FOR 2024)
https://t.me/UPSCPrelimsWithNeelesh ©Copyright For Queries Whatsapp - 9310161970
3. When biochar is a part of the growing medium, Several sources of biomass can be used to produce biochar, including
it enables the growing medium to retain water animal manure, certain industrial biomass waste, and forest, horticultural,
for a longer time. and agricultural residues.
Which of the statements given above is/are Biochar is not a standardized (uniform) product, and its composition can
correct? vary depending on the source of biomass used and the temperature at
which the thermal decomposition occurred.
(a) 1 and 2 only Statement 1 is correct: Biochar is used as a growing medium in urban
(b) 2 only farming- vertical farming.
(c) 1 and 3 only Statement 2 is correct: Biochar-based controlled release nitrogen
(d) 1,2 and 3 fertilizers (BCRNFs) enhance the yield and nitrogen- use efficiency by
slowly releasing nitrogen and modulating the abundance of functional
microbes through increased soil nitrification and reduced denitrification.
Statement 3 is correct: As a source of organic matter in soils, biochar also
increases water holding capacity and reduces the need for irrigation.

14 In the context of mitigating the impending global d Carbon sequestration is the process of capturing and storing atmospheric
warming due to anthropogenic emissions of carbon dioxide. It is one method of reducing the amount of carbon dioxide
carbon dioxide, which of the following can be the in the atmosphere with the goal of reducing global climate change.
potential sites for carbon sequestration? (2017) One common technique of Carbon sequestration is Geologic carbon
1. Abandoned and uneconomic coal seams sequestration, which refers to storing carbon dioxide in underground
2. Depleted oil and gas reservoirs geologic formations. The carbon dioxide is usually pressurized until it
3. Subterranean deep saline formations becomes a liquid, and then it is injected into porous rock formations.
Select the correct answer using the code given Geologic formations with a certain structure and porosity present an
below: opportunity for underground carbon dioxide storage. Examples of such
(a) 1 and 2 only formations with CO2 sequestration potential include unmineable coal
(b) 3 only seams, depleted oil reservoirs, depleted gas reservoirs, saline formations,
(c) 1 and 3 only and shale formations. Hence, option (d) is the correct answer.
(d) 1,2 and 3

TELEGRAM - UPSC PRELIMS WITH NEELESH


Learn ETHICS, ESSAY, SOCIOLOGY, CSAT BY VISITING THE CHANNEL (INTEGRATED MARATHON FOR 2024)
https://t.me/UPSCPrelimsWithNeelesh ©Copyright For Queries Whatsapp - 9310161970
15 Consider the following activities: (2023) c Statement 1 is correct: Adding crushed rock dust to farmland could draw
1. Spreading finely ground basalt rock on down up to two billion tonnes of carbon dioxide (CO2) from the air per
farmlands extensively year. The technique, known as enhanced rock weathering, involves
2. Increasing the alkalinity of oceans by adding spreading finely crushed basalt, a natural volcanic rock, on fields to boost
lime the soil’s ability to extract CO2 from the air.
3. Capturing carbon dioxide released by various
industries and pumping it into abandoned Statement 2 is correct: When you add lime to seawater, it reacts with that
subterranean mines in the form of carbonated acid and neutralizes it, forming a carbonate ion. The effect is a boost in
waters alkalinity to the ocean, and greater ocean alkalinity means more CO2 can
be absorbed.
How many of the above activities are often
considered and discussed for carbon capture and Statement 3 is correct: Capturing carbon dioxide released by various
sequestration? industries and pumping it into abandoned subterranean mines in the form
(a) Only one of carbonated waters: This activity refers to carbon capture and storage
(b) Only two (CCS), where carbon dioxide emissions from industrial processes are
(c) All three captured and stored underground. Abandoned subterranean mines can
(d) None serve as suitable storage sites, with the captured carbon dioxide pumped
into these locations in the form of carbonated waters.

16 Consider the following agricultural practices: b Carbon sequestration is the process of capturing and storing atmospheric
(2012) carbon dioxide. It is one method of reducing the amount of carbon dioxide
1. Contour bunding in the atmosphere with the goal of reducing global climate change.
2. Relay cropping Option (b) is correct: Among the given choices, only Zero tillage helps in
3. Zero tillage carbon sequestration in
ln the context of global climate change, which of the soil. Zero tillage refers to the process where the crop seed will be sown
the above helps/help in carbon through drillers without prior land preparation and disturbing the soil
sequestration/storage in the soil? where previous crop stubbles are present, It is also called No Tillage or Nil
{a) 1 and 2 only Tillage.
(b) 3 only Advantages of zero tillage:
(c) 1,2 and 3
TELEGRAM - UPSC PRELIMS WITH NEELESH
Learn ETHICS, ESSAY, SOCIOLOGY, CSAT BY VISITING THE CHANNEL (INTEGRATED MARATHON FOR 2024)
https://t.me/UPSCPrelimsWithNeelesh ©Copyright For Queries Whatsapp - 9310161970
(d) None of them - Reduction in the crop duration and thereby early cropping can be
obtained to get higher yields.
- Reduction in the cost of inputs for land preparation and therefore a
saving of around 80%.
- Residual moisture can be effectively utilized and number of irrigations
can be reduced.
- Dry matter and organic matter get added to the soil.
- Environmentally safe - Greenhouse effect will get reduced due to carbon
sequestration.
- No tillage reduces the compaction of the soil and reduces the water loss
by runoff and prevent soil erosion.
- As the soil is intact and no disturbance is done, No Till lands have more
useful flora and fauna.
Hence, option (b) is the correct answer.

17 Which of the following has/have shrunk a Aral Sea:


immensely/dried up in the recent past due to It is situated in Central Asia, between the Southern part of Kazakhstan and
human activities? (2018) Northern Uzbekistan. Up until the third quarter of the 20th century it was
1. Aral Sea the world’s 4th largest saline lake and contained 10 grams of salt per liter.
2. Black Sea o The two rivers that feed it are the Amu Darya and Syr Darya rivers,
3. Lake Baikal respectively reaching the Sea through the South and the North.
Shrinking of Aral Sea:
Select the correct answer using the code given The Soviet Government decided in the 1960s to divert Amu Darya and Syr
below: Darya rivers to favor agriculture rather than supply the Aral Sea basin.
(a) 1 only Many irrigational dams and canals were built on the arteries of the Aral
(b) 2 and 3 only Sea. However, they were poorly built, leading to the wastage of water at
(c) 2 only a severe level.
(d) 1 and 3 only The water level in the Aral Sea started drastically decreasing from the
1960s onward, ln 1987 the continuous shrinkage caused the Aral Sea to
be divided into 2 parts- the North Aral Sea was the smaller part (known as
TELEGRAM - UPSC PRELIMS WITH NEELESH
Learn ETHICS, ESSAY, SOCIOLOGY, CSAT BY VISITING THE CHANNEL (INTEGRATED MARATHON FOR 2024)
https://t.me/UPSCPrelimsWithNeelesh ©Copyright For Queries Whatsapp - 9310161970
the Lesser Sea), and the South Aral Sea became the larger part of the Lake
(the Greater Sea). It has now shrunk by more than 90% of its size over 50
years ago. Hence, option (a) is the correct answer.

18 Which one of the following lakes of West Africa b The Lake Faguibine System, four interlinked lakes 80 km west of Timbuktu
has become dry and turned into a desert? (2022) (region in Mali), was historically one of Mali's most fertile areas. But over
(a) Lake Victoria seven years, droughts in the 1970s dried up the lakes. Hence, option (b) is
(b) Lake Faguibine the correct answer.
(c) Lake Oguta
(d) Lake Vorta

TELEGRAM - UPSC PRELIMS WITH NEELESH


Learn ETHICS, ESSAY, SOCIOLOGY, CSAT BY VISITING THE CHANNEL (INTEGRATED MARATHON FOR 2024)
https://t.me/UPSCPrelimsWithNeelesh ©Copyright For Queries Whatsapp - 9310161970
TOPIC 6: INDIA AND CLIMATE CHANGE (Ch 22 – India and Climate Change, Ch 25 – Acts and
Policies, Ch 26 – Institution and Measures, Ch 27 – Environmental Organizations)
Total Number of Questions
asked from 2011-2023 22
My Instructions related to GENERAL INSTRUCTION
these chapters This is also the important portion. Focus should be more on India National Action Plan on Climate Change, INDC,
Wildlife Protection Act 1972, Environment Protection Act 1986, National Forest Policy 1988, Biodiversity Act
2002, Forest Rights Act 2006, Waste Management Rules, CAMPA

CHAPTER SPECIFIC INSTRUCTIONS


CHAPTER 22 INDIA AND CLIMATE CHANGE
Do not try to mug up each data that is given. Reports tend to change. So, focus on the general and broader
understanding of facts. Focus on NAPCC (its 8 missions and their brief details), what is climate equity monitor,
INDC (India commitments and its basics), what is fame, bonn challenge is related to?

CHAPTER 25 ACTS AND POLICIES


This is very important chapter. You must know about Wildlife Protection Act (its schedules – covered in earlier
chapter), Environment Protection Act, National Forest Policy, Biodiversity Act 2002, Forest Rights Act 2006,
Coastal Regulation Zone, Solid Waste Management Rules, Hazardous Waste Management Rules, Biomedical
Waste Management Rules, E-waste Management Rules, Plastic Waste Management Rules, Wetland Rules
2017, NGT.

CHAPTER 26 INSTITUTIONS AND MEASURES


Focus on what is CAMPA, Joint Forest Management, Social Forestry, CEPI, ECO Mark, National Clean Energy
Fund, National Mission for Electric Mobility
CHAPTER 27 ENVIRONMENT ORGANISATIONS
This is an important chapter. Focus on the status (Statutory/constitutional/autonomous/non-governmental
etc and if set up under which Act)
TELEGRAM - UPSC PRELIMS WITH NEELESH
Learn ETHICS, ESSAY, SOCIOLOGY, CSAT BY VISITING THE CHANNEL (INTEGRATED MARATHON FOR 2024)
https://t.me/UPSCPrelimsWithNeelesh ©Copyright For Queries Whatsapp - 9310161970
S. Question Answer Explanation
No.
1 Which of the following best describes/describe c National Mission for a Green India (GIM) is one of the eight Missions
the aim of 'Green India Mission, of the under the National Action Plan on Climate Change. It aims at protecting,
Government of India? (2016) restoring and enhancing India's forest cover and responding to Climate
1. Incorporating environmental benefits and costs Change. It is a centrally sponsored programme.
into the Union and State Budgets thereby Funding pattern: Central share of 90% for NE and special category States
implementing the green accounting, while 60% for rest of lndia.
2. Launching the second green revolution to Hence, statement 3 is correct.
enhance agricultural output so as to ensure food Statement 1 and statement 3 is not related to the Green India Mission
security to one and alt in the future
3. Restoring and enhancing forest cover and
responding to climate change by a combination of
adaptation and mitigation measures
Select the correct answer using the code given
below:
(a) 1 only
(b) 2 and 3 only
(c) 3 only
(d) 1,2, and 3

2 If National Water Mission is properly and b The National Water Mission was launched in2011. It was launched as a
completely implemented, how will it impact the part of the National Action Plan on Climate Change (NAPCC).
country? (2012)
1. Part of the water needs of urban areas will be Objectives of NWM:
met through recycling of waste-water. - The main objective of the National Water Mission is conservation of
2. The water requirements of coastal cities with water, minimizing wastage and ensuring a more equitable distribution
inadequate alternative sources of water will be both across and within states through integrated water resources
met by adopting appropriate technologies that development and management.
allow for the use of ocean water.
TELEGRAM - UPSC PRELIMS WITH NEELESH
Learn ETHICS, ESSAY, SOCIOLOGY, CSAT BY VISITING THE CHANNEL (INTEGRATED MARATHON FOR 2024)
https://t.me/UPSCPrelimsWithNeelesh ©Copyright For Queries Whatsapp - 9310161970
3. All the rivers of Himalayan origin will be linked - It seeks to ensure that a considerable share of the water needs of urban
to the rivers of peninsular India areas are met through the recycling of wastewater. Hence statement 1 is
4. The expenses incurred by farmers for digging correct.
bore-wells and for installing motors and pump- -It also aims to ensure that the water requirements of coastal cities with
sets to draw ground-water will be completely inadequate alternative sources of water are met through the adoption of
reimbursed by the Government. new and appropriate technologies such as low-temperature desalination
technologies that allow for the use of ocean water. Hence statement 2 is
Select the correct answer using the codes given correct.
below: There is no provision in NWM regarding the linking of river and
(a) 1 only compensation to farmers in installing pumping sets or digging borewells.
(b) 1 and 2 only
(c) 3 and 4 only
(d) 1,2,3 and 4

3 According to India's National Policy on Biofuels, a 'Biofuels' are produced from biomass resources and used in place of, or in
which of the following can be used as raw addition to, diesel petrol or other fossil fuels. ln order to promote biofuels
materials for the production of biofuels? (2020) in the country, a National Policy on Biofuels was made by the Ministry of
1, Cassava New and Renewable Energy during the year 2009. Again, in 2018 a new
2. Damaged wheat grains National Policy on Biofuels was brought.
3. Groundnut seeds
4. Horse gram Among the given choices, Cassava, Damaged wheat grains, Rotten
5. Rotten potatoes potatoes and Sugar beet can be used as raw materials for the production
6. Sugar beet of biofuels under the National Policy on Biofuels 2018.

Select the correct answer using the code given Main features of National Policy on Biofuels 2018 are:
below: The Policy categorizes biofuels as:
(a) 1,2,5 and 6 only (a)"Basic Biofuels"- First Generation (1G) bio-ethanol & biodiesel
(b) 1, 3, 4 and 6 only (b)"Advanced Biofuels" - Second Generation (2G) ethanol, Municipal
(c) 2,3,4 and 5 only Solid Waste (MSW)
(d) 1,2,3,4,5 and 6 to drop-in fuels and Third Generation (3G) biofuels, bio-CNG etc.
TELEGRAM - UPSC PRELIMS WITH NEELESH
Learn ETHICS, ESSAY, SOCIOLOGY, CSAT BY VISITING THE CHANNEL (INTEGRATED MARATHON FOR 2024)
https://t.me/UPSCPrelimsWithNeelesh ©Copyright For Queries Whatsapp - 9310161970
The Policy expands the scope of raw material for ethanol production by
allowing use of Sugarcane Juice, Sugar containing materials like Sugar
Beet, Sweet Sorghum, Starch containing materials like Corn, Cassava,
Damaged food grains like wheat, broken rice, Rotten Potatoes, unfit for
human consumption. The Policy allows use of surplus food grains for
production of ethanol for blending with petrol with the approval of
National Biofuel Coordination Committee.

4 On which of the following can you find the Bureau d Bureau of Energy Efficiency initiated the Standards & Labeling
of Energy Efficiency Star Label? (2016) programme for equipment and appliances in 2006 to provide the
1. Ceiling fans consumer an informed choice about the energy saving and thereby the
2. Electric geysers cost saving potential of the relevant marketed product. The scheme
3. Tubular fluorescent lamps targets display of energy performance labels on high energy end use
Select the correct answer using the code given equipment & appliances and lays down minimum energy performance
below: standards.
(a) 1 and 2 only It is currently invoked for equipments /appliances including: Room Air
(b) 3 only Conditioner (Fixed Speed), Ceiling Fan, Colour Television, Computer,
(c) 2 and 3 only Direct Cool Refrigerator, Domestic Gas Stove, Frost Free Refrigerator,
(d) 1,2 and 3 Tubular fluorescent lamps, General Purpose lndustrial Motor, Stationary
Storage Type Electric Water Heater, Submersible Pump Set, Washing
machine (Semi /Top Load/Front Load), Led Lamps, Room Air Conditioner
(Variable Speed), Agricultural Pumpset, Microwave Oven, Solar Water
Heater, High Energy Li-Battery, Hence, option (d) is the correct answer.

5 ln rural road construction, the use of which of the a Copper Slag is a major problem in the metallurgical industries as it is
following is preferred for ensuring environmental dumped into heaps which have accumulated into millions of tons over the
sustainability or to reduce carbon footprint? years, and they pose a danger to the environment as they occupy vacant
(2020) land. Hence using them in road construction is environmentally
1. Copper slag sustainable.
2. Cold mix asphalt technology
TELEGRAM - UPSC PRELIMS WITH NEELESH
Learn ETHICS, ESSAY, SOCIOLOGY, CSAT BY VISITING THE CHANNEL (INTEGRATED MARATHON FOR 2024)
https://t.me/UPSCPrelimsWithNeelesh ©Copyright For Queries Whatsapp - 9310161970
3. Geotextiles Cold asphalt mix is produced by mixing unheated mineral aggregate with
4. Hot mix asphalt technology either emulsified bitumen or foamed bitumen. Unlike hot mix asphalt
5. Portland cement (HMA), cold asphalt mix does not require any heating of aggregate which
makes it economical and relatively pollution-free.
Select the correct answer using the code given Geotextiles, a permeable fabric, natural, strong, highly durable, resistant
below: to rots, molds and moisture, free from any microbial attack, are very much
{a) 1,2 and 3 only suited to road construction. Using geotextiles will reduce cost, reduces
(b) 2, 3 and 4 only greenhouse gasses and pollution (no heating is required)
(c) 4 and 5 only Hot mix asphalt technology requires a temperature of 155 degrees C that
(d) 1 and 5 only leads to emission of hydrocarbons and suspended particulate matter.
Portland cement usage can lead to dust and particulate matter. Hence,
option (a) is the correct answer.

6 Steel slag can be the material for which of the d Steel slag is generated as a byproduct of the steel manufacturing
following? (2020) processes. It is generated from a steel furnace burning at around 1,500-
1. Construction of base road 1,600 degree centigrade in the form of molten flux material as an
2. Improvement of agricultural soil impurity. The molten material is poured into the slag pits for cooling and
3. Production of cement further processed to develop stable steel slag aggregates.
Select the correct answer using the code given Steel Slag
below It is a by-product of the steel making process produced during the
separation of the molten steel from impurities in steel-making furnaces.
(a) 1 and 2 only It is used as a base course material under the surface layer of an asphalt
(b) 2 and 3 only road. It can be used in the agricultural sector due to its ability to correct
(c) 1 and 3 only soil acidity. It can be used to produce cement; slag cement used in
(d) 1,2 and 3 only concrete, either as a separate cementitious component or as part of a
blended cement.

TELEGRAM - UPSC PRELIMS WITH NEELESH


Learn ETHICS, ESSAY, SOCIOLOGY, CSAT BY VISITING THE CHANNEL (INTEGRATED MARATHON FOR 2024)
https://t.me/UPSCPrelimsWithNeelesh ©Copyright For Queries Whatsapp - 9310161970
7 With reference to Indian laws about wildlife a Statement 1 is correct: Section 39 of the Wildlife Protection Act 1972
protection, consider the following statements: states every wild animal-other than vermin- kept or bred in captivity or
(2022) hunted shall be the
1. Wild animals are the sole property of the property of the State Government. lf such an animal is hunted in a
government. sanctuary or National Park declared by the Central Government such
2. When a wild animal is declared protected, such animal or any article, trophy, meat etc shall be the property of the Central
animal is entitled for equal protection whether it Government.
is found in protected areas or outside. Statement 2 is correct: Wildlife (Protection) Act, 1972, does not
3. Apprehension of a protected wild animal discriminate between animals found in protected areas and outside. It
becoming a danger to human life is sufficient provides for equal protection for wild animals irrespective of where they
ground for its capture or killing. are found.
Which of the statements given above is/are Statement 3 is not correct: Mere apprehension of a protected wild animal
correct? becoming a danger cannot be a sufficient ground for the capture or killing
(a) 1 and 2 only of a protected animal.
(b) 2 only As per the provisions of the act, the Chief Wildlife Warden's order is
(c) I and 3 only required in such
(d) 3 only cases. He/She has to be satisfied enough to give order in writing to allow
for killing/hunting of the concerned animal. Hence, option (a) is the
correct answer.

8 lf a particular plant species is placed under a Schedule Vl of the Wildlife Protection Act 1972 provides for regulation of
Schedule Vl of The Wildlife Protection Act, 1972, certain plants which can be cultivated only after acquiring a license.
what is the implication? (2020) Hence, option (a) is the correct answer.
(a) A licence is required to cultivate that plant. Instruction – Prepare the schedules of WPA 1972 properly.
(b) Such a plant cannot be cultivated under any
circumstances.
(c) lt is a Genetically Modified crop plant.
{d) Such a plant is invasive and harmful to the
ecosystem

TELEGRAM - UPSC PRELIMS WITH NEELESH


Learn ETHICS, ESSAY, SOCIOLOGY, CSAT BY VISITING THE CHANNEL (INTEGRATED MARATHON FOR 2024)
https://t.me/UPSCPrelimsWithNeelesh ©Copyright For Queries Whatsapp - 9310161970
9 ln lndia, if a species of tortoise is declared a Wild Life (Protection) Act,1972 provides for the protection of wild
protected under Schedule I of the Wildlife animals, birds, and plants. The Act provides for stringent punishment for
(Protection) Act,1972, what does it imply? (2017) violation of its provisions.
(a) lt enjoys the same level of protection as the tiger. It has six schedules which give varying degrees of protection. Schedule I
(b) lt no longer exists in the wild, a few individuals and Schedule ll provide the highest protection - offences under these are
are under captive protection; and now it is prescribed the highest penalties. So, Option (a) is correct. lf a species of
impossible to prevent its extinction. tortoise is declared protected under Schedule l, it will enjoy the same level
(c) lt is endemic to a particular region of lndia of protection as the tiger, as Tiger is also listed under Schedule I of the Act.
(d) Both (b) and (c) stated above are correct in this
context

10 With reference to Eco-Sensitive Zones, which of d Statement l is not correct: Eco-Sensitive Zones are the ecologically fragile
the following statements is/are correct? (2014) areas around protected areas and wildlife corridors areas that are
1. Eco-Sensitive Zones are the areas that are declared under the Environment Protection Act of 1986 by the Ministry
declared under the Wildlife (Protection) Act, 1972. of Environment, Forest and Climate Change.
2. The purpose of the declaration of Eco-Sensitive Statement 2 is not correct: The purpose of declaration of ESZs is to create
Zones is to prohibit all kinds of human activities in "shock
those zones except agriculture. absorbers" for protected areas in order to lessen the detrimental effects
of the surrounding human activities. Such activities are categorized in 3
groups as follows:
Select the correct answer using the code given
below: 1. Prohibited activities: Commercial mining, saw mills, industries causing
(a) 1 only pollution (air,
(b) 2 only water, soil, noise, etc), the establishment of major hydroelectric projects
(c) Both 1 and 2 (HEP) commercial use of wood, tourism activities like hot-air balloons over
(d) Neither 1 nor 2 the National Park,
discharge of effluents or any solid waste or production of hazardous
substances
2. Regulated activities: Felling of trees, the establishment of hotels and
resorts, commercial use of natural water, erection of electrical cables,
TELEGRAM - UPSC PRELIMS WITH NEELESH
Learn ETHICS, ESSAY, SOCIOLOGY, CSAT BY VISITING THE CHANNEL (INTEGRATED MARATHON FOR 2024)
https://t.me/UPSCPrelimsWithNeelesh ©Copyright For Queries Whatsapp - 9310161970
drastic change of agriculture system, e.g. adoption of heavy technology,
pesticides, etc. widening of roads.

3. Permitted activities: Ongoing agricultural or horticultural practices,


rainwater harvesting, organic farming, use of renewable energy sources,
and adoption of green technology for all activities.

11 Consider the following statements: (2019) b Statement 1 is not correct: As per section 3 of the Environment Protection
The Environment Protection Act, 1986 empowers Act 1986, the Union
the Government of India to Government shall have the power to take all such measures as it deems
1. State the requirement of public participation in necessary or expedient for the purpose of protecting and improving the
the process of environmental protection, and the quality of time environment and preventing, controlling and abating
procedure and manner in which it is sought. environmental pollution. But, the requirement of public participation in
2. Lay down the standards for emission or the process of environmental protection is not mentioned explicitly.
discharge of environmental pollutants from Statement 2 is correct: According to the Environment protection Act
various sources. 1986, the central government may lay down standards for emission or
Which of the statements given above is/ are discharge of environmental pollutants from various types of resources.
correct?
(a) 1 only
(b) 2 only
(c) Both 1 and 2
(d) Neither 1 nor 2

12 Which one of the following has been constituted c Central Ground Water Authority has been constituted under Section 3 (3)
under the Environment (Protection) Act, 1986? of the Environment (Protection) Act, 1986 to regulate and control the
(2022) development and management of groundwater resources in the country.
(a) Central Water Commission Hence, option (c) is the correct answer.
(b) Central Ground Water Board It notifies over-exploited areas in the country for regulation of
(c) Central Ground Water Authority groundwater development and management. ln these notified areas, the
(d) National Water Development Agency installation of new groundwater abstraction structures is being regulated
TELEGRAM - UPSC PRELIMS WITH NEELESH
Learn ETHICS, ESSAY, SOCIOLOGY, CSAT BY VISITING THE CHANNEL (INTEGRATED MARATHON FOR 2024)
https://t.me/UPSCPrelimsWithNeelesh ©Copyright For Queries Whatsapp - 9310161970
13 Consider the following statements (2020) b Statement 1 is not correct: Central Ground Water Board (CGWB)
36% of lndia's districts are classified as periodically assesses the ground water resources pan-India in
'overexploited" or critical" by the Central Ground collaboration with States/UTs.
Water Authority (CGWA) (2020) 'Over-exploited' units are those where groundwater extraction
2. CGWA was formed under the Environment substantially exceeds (more than 100%) the annually replenishable
(Protection) Act. groundwater recharge. If it is between 90-100%, it comes under the
3. India has the largest area under groundwater 'critical' category. Those between 70-90% are categorized as 'semi-critical'
irrigation in the world. while the units where extraction is less than 7O% of the annually
Which of the statements given above is/are replenishable groundwater recharge are categorized as 'safe' category.
correct? According to the 2020 assessment, 16% districts are 'over- exploited' and
(a) 1 only 3.9% fall under the 'critical' category.
(b) 2 and 3 only
(c) 2 only Statement 2 is correct: Central Ground Water Authority is a Statutory
(d) 1 and 3 only body that has been constituted under the Environment (Protection) Act,
1986 to regulate and control development and management of
groundwater resources in the country.

Statement 3 is correct: The countries with the largest extent of areas


equipped for irrigation with groundwater, in absolute terms, are India (39
million ha), China (19 million ha) and the USA (17 million ha).

14 The Genetic Engineering Appraisal Committee is c The Genetic Engineering Appraisal Committee is a statuary body. It was
constituted under the: (2015) constituted under the Environment Protection Act 1986.It functions
(a) Food Safety and Standards Act, 2006. under the Ministry of Environment, Forest and Climate Change.
(b) Geographical Indication of Goods (Registration
and Protection) Act, 1999.
(c) Environment (Protection) Act, 1986.
(d) Wildlife (Protection) Act, 1972.

TELEGRAM - UPSC PRELIMS WITH NEELESH


Learn ETHICS, ESSAY, SOCIOLOGY, CSAT BY VISITING THE CHANNEL (INTEGRATED MARATHON FOR 2024)
https://t.me/UPSCPrelimsWithNeelesh ©Copyright For Queries Whatsapp - 9310161970
15 With reference to India, consider the following c Biodiversity refers to the variety of all forms of life and it is essential to
Central Acts: (2011) the existence and proper functioning of all ecosystems. Biodiversity
1. Import and Export (Control) Act,1947 conservation refers to the protection, preservation, and management of
2. Mining and Mineral Development (Regulation) ecosystems and natural habitats and ensuring that they are healthy and
Act, 1957 functional.
3. Customs Act, 1962 Acts and Rules which have a bearing on biodiversity conservation in
4. Indian Forest AcL,1927 India are:
- The Indian Forest Act1927
Which of the above Acts have relevance to/bearing - Mining and Mineral Development (Regulation) Act 1957
on the biodiversity conservation in the country? - The Customs Act, 1962
(a) 1 and3only - Import and Export (Control) Act,1947
(b) 2,3 and 4 only - The Biological Diversity AcL,2002
(c) 1,2,3 and 4 - The National Green Tribunal Act 2010
(d) None of the above Acts - The Indian Fisheries Act. 1897
- The Patents Act, 1970
- Forest (Conservation) Act 1980
- Environment (Protection) Act, 1986. Hence, option (c) is the correct
answer.

16 Consider the following statements: (2014) b Statement 1 is not correct: The Animal Welfare Board of India was set up
1. Animal Welfare Board of India is established in 1962, in accordance with Section 4 of the Prevention of Cruelty to
under the Environment (Protection) Act, 1986. Animals Acts 1960. It was the first of its kind to be established by any
2. National Tiger Conservation Authority is a Government in the world.
statutory body.
3. National Ganga River Basin Authority is chaired Statement 2 is correct: The National Tiger Conservation Authority (NTCA)
by the Prime Minister. is a statuary body that has been constituted under section 38 L (1) of the
Which of the statements given above is/are Wildlife (Protection) Act,1972. The authority consists of the Minister in
correct? charge of the Ministry of Environment and Forests (as Chairperson), the
(a) 1 only Minister of State in the Ministry of Environment and Forests (as Vice-
(b) 2 and 3 only
TELEGRAM - UPSC PRELIMS WITH NEELESH
Learn ETHICS, ESSAY, SOCIOLOGY, CSAT BY VISITING THE CHANNEL (INTEGRATED MARATHON FOR 2024)
https://t.me/UPSCPrelimsWithNeelesh ©Copyright For Queries Whatsapp - 9310161970
(c) 2 only Chairperson), three members of Parliament, the Secretary of Ministry OF
(d) 1,2 and 3 Environment and Forest and other members.

Statement 3 is correct: National Ganga River Basin Authority (NGRBA) was


constituted under the provisions of the Environment (Protection) Act
(EPA),1986. The Prime Minister is the chair of the Authority. Other
members include the cabinet ministers of ministries that include the
Ganges among their direct concerns and the chief ministers of states
through which the Ganges River flows.

17 How does the National Biodiversity Authority c National Biodiversity Authority (NBA) was established by the Central
(NBA) help in protecting the Indian agriculture? Government in 2003 to implement lndia's Biological Diversity Act (2002).
(2012) It is a Statutory Body and it performs facilitative, regulatory and advisory
1. NBA checks the biopiracy and protects the functions for the Government of India on issues of conservation,
indigenous and traditional genetic resources. sustainable use of biological resources and fair and equitable sharing of
2. NBA directly monitors and supervises the benefits arising out of the use of biological resources.
scientific research on genetic modification of crop
plants. Statement 1 is correct: Biopiracy refers to the unethical or unlawful
3. Application for intellectual Property Rights appropriation or commercial exploitation of biological materials (such as
related to genetic/biological resources cannot be medicinal plant extracts) that are native to a particular country or territory
made without the approval of NBA. without providing fair financial compensation to the people or
government of that country or territory.
Which of the statements given above is/are
correct? ln India, the approval of the National Biodiversity Authority has to be
(a) 1 only obtained for any biological resource occurring in India or knowledge
(b) 2 and 3 only associated thereto for research or for commercial utilization or for bio-
(c) 1 and 3 only survey and bio-utilisation. No person shall, without the previous approval
(d) 1,2 and 3 of the NBA, transfer, the results of any research relating to any biological
resources occurring in, or obtained from, lndia for monetary

TELEGRAM - UPSC PRELIMS WITH NEELESH


Learn ETHICS, ESSAY, SOCIOLOGY, CSAT BY VISITING THE CHANNEL (INTEGRATED MARATHON FOR 2024)
https://t.me/UPSCPrelimsWithNeelesh ©Copyright For Queries Whatsapp - 9310161970
consideration. Thus, NBA checks the biopiracy and protects the
indigenous and traditional genetic resources.

Statement 2 is not correct: GEAC (Genetic Engineering Appraisal


Committee) directly monitors and supervises the scientific research on
genetic modification of crop plants.

Statement 3 is correct: Application for intellectual property rights related


to genetic/biological resources cannot be made without approval of the
National Biodiversity Authority.

18 As per the Solid Waste Management Rules, 2016 in c Option (a) is not correct: As per the Solid Waste Management Rules,
India, which one of the following statements is 2016, waste generator would now have to segregate waste into three
correct? (2019) categories (not five categories) namely Biodegradables, Dry (Plastic,
(a) Waste generator has to segregate waste into five Paper, metal, Wood etc.) and domestic Hazardous waste (Diapers,
categories. napkins, mosquito repellants, cleaning agents) before handing it over to
(b) The Rules are applicable to notified urban local the collector.
bodies, notified towns and all industrial townships
only. Option (b) is not correct: The Rules are applicable to urban
(c) The Rules provide for exact and elaborate agglomerations, census towns, notified industrial townships, areas under
criteria for the identification of sites for landfills and the control of Indian Railways, airports, airbase, Port and harbor, defence
waste processing facilities. establishments, special economic zones, State and Central government
(d) It is mandatory on the part of the waste organizations, places of pilgrims, religious & historical importance. Hence,
generator that the waste generated in one district rules are not limited to notified urban local bodies, notified towns and all
cannot be moved to another district industrial townships only.

Option (c) is correct: The rules provide for exact and elaborate criteria for
the identification of sites for landfills and waste processing facilities.

TELEGRAM - UPSC PRELIMS WITH NEELESH


Learn ETHICS, ESSAY, SOCIOLOGY, CSAT BY VISITING THE CHANNEL (INTEGRATED MARATHON FOR 2024)
https://t.me/UPSCPrelimsWithNeelesh ©Copyright For Queries Whatsapp - 9310161970
Option (d) is not correct: It is not mandatory for the waste generator that
the waste generated in one district cannot be moved to another district.

19 How is the National Green Tribunal (NGT) different b Statement I is not correct: The National Green Tribunal has been
from the Central Pollution Control Board (CPCB)? established under the National Green Tribunal Act 2010. The Central
(2018) Pollution Control Board (CPCB) is a statutory organization that was
1. The NGT has been established by an Act constituted in September, 1974 under the Water (Prevention and Control
whereas the CPCB has been created by an of Pollution) Act,1974.
executive order of the Government. Statement 2 is correct: National Green Tribunal's dedicated jurisdiction in
2. The NGT provides environmental justice and environmental matters shall provide speedy environmental justice and
helps reduce the burden of litigation in the higher help reduce the burden of litigation in the higher courts
courts whereas the CPCB promotes cleanliness of
streams and wells, and aims to improve the quality
of air in the country.
Which of the statements given above is/are
correct?
(a) 1 only
(b) 2 only
(c) Both 1 and 2
(d) Neither 1 nor 2

20 With reference to the Indian Renewable Energy c Both statements 1 and 2 are correct. IREDA is a Public Limited
Development Agency Limited (IREDA), which of Government Company established as a Non-Banking Financial Institution
the following statements is/are correct? (2015) in 1987. It is a Mini Ratna (Category - l) Enterprise. It comes under the
1. It is a Public Limited Government Company. administrative control of the Ministry of New and Renewable Energy
2. It is a Non-Banking Financial Company. (MNRE).

Select the correct answer using the code given


below:
(a) 1 only
TELEGRAM - UPSC PRELIMS WITH NEELESH
Learn ETHICS, ESSAY, SOCIOLOGY, CSAT BY VISITING THE CHANNEL (INTEGRATED MARATHON FOR 2024)
https://t.me/UPSCPrelimsWithNeelesh ©Copyright For Queries Whatsapp - 9310161970
(b) 2 only
(c) Both 1 and 2
(d) Neither 1 nor 2

21 With reference to Bombay Natural History Society c Statement I is not correct: Bombay Natural History Society is one of the
(BNHS), consider the following statements: (2014) largest non-governmental organizations in India engaged in conservation
1. It is an autonomous organization under the and biodiversity research. Hence, it does not come under the Ministry of
Ministry of Environment and Forests. Environment and Forests.
2. It strives to conserve nature through action- Statement 2 is correct: BNHS's Mission includes the conservation of
based research, education and public awareness. nature, primarily biological diversity through action based on research,
3. It organizes and conducts nature trails and education, and public awareness.
camps for the general public. Statement 3 is correct: It also aims to spread awareness about nature
Which of the statements given above is/are through science-based research, conservation advocacy, education,
correct? scientific publications, nature tours, and other programmes. It organizes
(a) 1 and 3 only and conducts nature trails and camps for the general public.
(b) 2 only
(c) 2 and 3 only
(d) 1,2 and 3

22 Under the Scheduled Tribes and Other Traditional d Section 11 of the Scheduled Tribes and Other Traditional Forest Dwellers
Forest Dwellers (Recognition of Forest Rights) Act, (Recognition of Forest Rights) Act, 2006 says that the Gram Sabha shall
2006, who shall be the authority to initiate the be the authority to initiate the process for determining the nature and
process for determining the nature and extent of extent of individual or community forest rights or both that may be given
individual or community forest rights or both? to the forest dwelling Scheduled Tribes and other traditional forest
(2013) dwellers within the local limits of its jurisdiction under this Act. The Forest
(a) State Forest Department Rights Committee shall assist the Gram Sabha. Hence, option (d) is the
(b) District Collector/Deputy Commissioner correct answer.
(c) Tahsildar/Block Development Officer/Mandal
Revenue Officer
(d) Gram Sabha
TELEGRAM - UPSC PRELIMS WITH NEELESH
Learn ETHICS, ESSAY, SOCIOLOGY, CSAT BY VISITING THE CHANNEL (INTEGRATED MARATHON FOR 2024)
https://t.me/UPSCPrelimsWithNeelesh ©Copyright For Queries Whatsapp - 9310161970
TOPIC 7: AGRICULTURE (Ch 24 – Agriculture)

Total Number of Questions


asked from 2011-2023 3
My Instructions related to INSTRUCTIONS RELATED TO THIS CHAPTER
these chapters Focus more on terms used (e.g. meaning of silviculture, apiculture etc), types of revolution in agriculture, tillage
meaning and uses (subtypes not important), cropping – meaning of cropping intensity, multiple cropping, ratoon
cropping, intercropping(with examples), mixed cropping, mixed farming (it has animal+crop), crop rotation,
permaculture, integrated farming system, what are the macro and micro nutrients – remember them, what is
green manuring, nitrogen fixers with examples, what are weeds. Read the various terms and their meanings given
in the end of the chapter.

1 "System of Rice Intensification" of cultivation, in d System of Rice Intensification (SRI) is a methodology for increasing the
which alternate wetting and drying of rice fields is productivity of irrigated rice by changing the management of plants, soil,
practiced, results in: (2022) water and nutrients particularly by eliciting greater root growth.
1. Reduced seed requirement Statement 1 is correct: In the SRI method single seedling is planted at
2. Reduced methane production wider spacing. Seed requirement in SRI is only 7-8 kg per hectare
3. Reduced electricity consumption (compared to 50 kg per hectare normally).
Statement 2 is correct: SRI method uses regular water application to keep
Select the correct answer using the code given soil moist but not saturated, with intermittent dryings, alternating aerobic
below: and anaerobic soil conditions. This reduces methane emissions. Also, less
(a) 1 and 2 only carbon dioxide is generated due to reduced reliance on and transport of
(b) 2 and 3 only manufactured inputs.
(c) 1 and 3 only Statement 3 is correct: SRI method do not use tradition flood irrigation
(d) 1, 2 and 3 process and hence cultivation requires less water which leads to less
consumption of electricity for pump sets.

TELEGRAM - UPSC PRELIMS WITH NEELESH


Learn ETHICS, ESSAY, SOCIOLOGY, CSAT BY VISITING THE CHANNEL (INTEGRATED MARATHON FOR 2024)
https://t.me/UPSCPrelimsWithNeelesh ©Copyright For Queries Whatsapp - 9310161970
2 In India, the use of carbofuran, methyl parathion, a Carbofuran, methyl parathion, phorate and triazophos are used as
phorate and triazophos is viewed with pesticides in agriculture.
apprehension. These chemicals are used as (2019) Pesticides are chemical compounds that are used to kill pests, including
a. pesticides in agriculture insects, rodents, fungi and unwanted plants (weeds). They are used in
b. preservatives in processed foods public health to kill vectors of disease, such as mosquitoes, and in
c. fruit-ripening agents agriculture to kill pests that damage crops.
d. moisturising agents in cosmetics
Hence, option (a) is the correct answer.

3 With reference to the circumstances in Indian c Conservation Agriculture is a concept in support of sustainable land
agriculture, the concept of “Conservation management, environmental protection and climate change adaptation
Agriculture" assumes significance. Which of the and mitigation.
following fall under the Conservation Agriculture? Conservation Agriculture is based on 3 interlinked principles adapted to
(2018) reflect local conditions and needs:
1. Avoiding monoculture practices
2. Adopting minimum tillage 1. Minimum mechanical soil disturbance (i.e. no/zero tillage) through
3. Avoiding the cultivation of plantation crops direct seed and/or fertilizer placement. This reduces soil erosion and
4. Using crop residues to cover soil surface preserves soil organic matter. Hence statement 2 is correct.
5. Adopting spatial and temporal crop
sequencing/crop rotations 2. Permanent soil organic cover (at least 30%) with crop residues and/or
cover crops. Maintaining a protective layer of vegetation on the soil
Select the correct answer using the code given surface suppresses weeds, protects the soil from the impact of extreme
below: weather patterns, helps to preserve soil moisture, and avoids compaction
of the soil. Hence statement 4 is correct.
(a) 1, 3 and 4 only
(b) 2,3,4 and 5 only 3. Species diversification through varied crop sequences and associations
(c) 2,4 and 5 only involving at least different crop species. A well-designed crop rotation
(d) 1,2,3 and 5 only promotes good soil structure, fosters a diverse range of soil flora and
fauna that contributes to nutrient cycling and improved plant nutrition,
and helps to prevent pests and diseases. Hence statement 5 is correct.
TELEGRAM - UPSC PRELIMS WITH NEELESH
Learn ETHICS, ESSAY, SOCIOLOGY, CSAT BY VISITING THE CHANNEL (INTEGRATED MARATHON FOR 2024)
https://t.me/UPSCPrelimsWithNeelesh ©Copyright For Queries Whatsapp - 9310161970
Conservation Agriculture is 20 to 50% less labor intensive and thus
contributes to reducing greenhouse gas emissions through lower energy
inputs and improved nutrient use efficiency. At the same time, it stabilizes
and protects soil from breaking down and releasing carbon to the
atmosphere.

TELEGRAM - UPSC PRELIMS WITH NEELESH


Learn ETHICS, ESSAY, SOCIOLOGY, CSAT BY VISITING THE CHANNEL (INTEGRATED MARATHON FOR 2024)
https://t.me/UPSCPrelimsWithNeelesh ©Copyright For Queries Whatsapp - 9310161970
TOPIC 8: WORLD ORGANISATIONS (Ch 23 – Climate Change Organizations, Ch 28 – International organizations and
Conventions)
Total Number of Questions
asked from 2011-2023 32
My Instructions related to GENERAL INSTRUCTION
these chapters This is also an important section. But mostly UPSC tend to ask about organizations related to UN or the one in
NEWS. So, this section has a mix of evergreen topics (such as KYOTO, Annex I etc) vs the topics in News.

CHAPTER SPECIFIC INSTRUCTIONS


CHAPTER 23 CLIMATE CHANGE ORGANIZATIONS
Focus more on UNFCCC, Kyoto Protocol (very important), Green Climate Fund, Adaptation Fund, Paris climate
change COP 21 and latest COPs, INDC, Orphan issues, Talanoa dialogue relates to, COP 26 Glasgow, REDD
and REDD+, GEF, IPCC, Ecological Footprint, Strategic Climate Fund, Biocarbon Fund, Clean Technology Fund,
Forest Carbon Partnership Facility, Special Climate Change Fund, Green Climate Fund

CHAPTER 28 INTERNATIONAL ORGANIZATIONS AND CONVENTIONS


Focus more on UNEP, UNCED, AGENDA 21, RIO+5, RIO+20, CBD, Cartegena Protocol, Nagoya Protocol, Aichi
Targets (done go in details – know what is it), Ramsar convention, CITES, IUCN, Stockholm Convention, Basel
Convention, Rotterdam Convention, UNCCD, Vienna Convention, Kigali Agreement, Minamata Convention.
While reading them focus on
1. Legal binding or not
2. Governmental or not
3. It relates to?
4. Do not go in excess details. But know the basics

TELEGRAM - UPSC PRELIMS WITH NEELESH


Learn ETHICS, ESSAY, SOCIOLOGY, CSAT BY VISITING THE CHANNEL (INTEGRATED MARATHON FOR 2024)
https://t.me/UPSCPrelimsWithNeelesh ©Copyright For Queries Whatsapp - 9310161970
S. No. Question Answer Explanation
1 With reference to the Agreement at the UNFCCC b Statement 1 is not correct: The Paris Agreement is a legally binding
Meeting in Paris in 2015, which of the following international treaty on climate change. It was adopted by 196 Parties at
statements is/are correct? (2016) COP 21 in Paris, on 12 December 2015. It entered into force on 4
1. The Agreement was signed by all the member November 2016.
countries of the UN and it will go into effect in
2017. Statement 2 is correct: The Paris Agreement, in seeking to strengthen the
2. The Agreement aims to limit the greenhouse gas global response to climate change, reaffirms the goal of limiting global
emissions so that the rise in average temperature increase to well below 2 degrees Celsius, while pursuing
global temperature by the end of this century does efforts to limit the increase to 1.5 degrees.
not exceed 2 degree C or even 1.5 degree C above
pre-industrial levels. Statement 3 is not correct: The Paris Agreement reaffirms the obligations
3. Developed countries acknowledged their of developed countries to support the efforts of developing country
historical responsibility in global warming and Parties to build clean, climate-resilient futures. It includes a provision
committed to donate $1000 billion a year from requiring developed countries to send $100 billion annually to their
2020 to help developing countries to cope with developing counterparts from 2020 onwards.
climate change.
Select the correct answer using the code given
below:
(a) 1 and 3 only
(b) 2 only
(c) 2 and 3 only
(d) 1,2, and 3

2 Which of the following statements regarding a Statement 1 is correct: The Green Climate Fund (GCF) is the world's
'Green Climate Fund' is/are correct? (2015) largest climate fund. GCF was established by the Cancun Agreements in
1. It is intended to assist the developing countries 2010 under the UNFCCCs financial mechanism to channel funding from
in adaptation and mitigation practices to counter developed countries to developing countries to allow them to mitigate
climate change. climate change and also adapt to disruptions arising from a changing
climate. GCF is headquartered in the Songdo district in South Korea.
TELEGRAM - UPSC PRELIMS WITH NEELESH
Learn ETHICS, ESSAY, SOCIOLOGY, CSAT BY VISITING THE CHANNEL (INTEGRATED MARATHON FOR 2024)
https://t.me/UPSCPrelimsWithNeelesh ©Copyright For Queries Whatsapp - 9310161970
2. It is founded under the aegis of UNEP, OECD, Statement 2 is not correct: GCF is a financial mechanism of UNFCCC. The
Asian Development Bank and World Bank. Paris agreement provides that the Green Climate Fund (GCF) with other
arrangements shall serve as the financial mechanism for the Agreement.
Select the correct answer using the code given GCF operates through a network of over 200 Accredited Entities and
below: delivery partners who work directly with developing countries for project
(a) 1 only design and implementation.
(c) Both 1 and 2
(b) 2 only
(d) Neither 1 nor 2

3 Consider the following statements: (2016) a Statement 1 is correct: The International Solar Alliance (ISA) is a treaty-
1. The International Solar Alliance was launched at based intergovernmental organization working to create a global market
the United Nations Climate Change Conference in system to tap the benefits of solar power and promote clean energy
2015. application. International Solar Alliance (ISA) was launched by PM Modi
2. The Alliance includes all the member countries and former French president Francois Hollande in November 2015 at the
of the United Nations. 21st session of the United Nations Climate Change Conference of the
Which of the statements given above is /are Parties (COP-21) in Paris. The ISA Framework Agreement was opened for
correct? signature during the CoP-22 at Marrakesh on 15th November 2016.
(a) 1 only Statement 2 is not correct: A total of 106 countries have signed the ISA
(b) 2 only Framework Agreement. All member states of the United Nations are
(c) Both 1 and 2 eligible to join the ISA. Recently, USA has become the 101st member to
(d) Neither 1 nor 2 ratify the ISA Group.

4 Which of the following statements is/are correct? d REDD+ (Reducing emissions from deforestation and forest degradation) is
(2016) a mechanism developed by Parties to the United Nations Framework
Proper design and effective implementation of UN Convention on Climate Change
REDD+ Programme can significantly contribute to (UNFCCC). It creates a financial value for the carbon stored in forests by
1. protection of biodiversity offering incentives for developing countries to reduce emissions from
2. resilience of forest ecosystems forested lands and invest in low-carbon paths to sustainable
3. poverty reduction
TELEGRAM - UPSC PRELIMS WITH NEELESH
Learn ETHICS, ESSAY, SOCIOLOGY, CSAT BY VISITING THE CHANNEL (INTEGRATED MARATHON FOR 2024)
https://t.me/UPSCPrelimsWithNeelesh ©Copyright For Queries Whatsapp - 9310161970
Select the correct answer using the code given development. Developing countries would receive results-based
below: payments for results-based actions.
(a) 1 and 2 only Option (d) is correct: The REDD+ mechanism contributes directly to
(b) 3 only achieving Sustainable Development Goals (SDGs) 13 and 15 which
(c) 2 and 3 only address climate change, reducing deforestation and sustainable use of
(d) 1, 2 and 3 ecosystems. lt also contributes to achieving other SDGs – including those
which address poverty reduction, health and well-being, hunger
alleviation, and improving institutions.

5 What is/are the importance/ importances of the c Statement 1 is correct: The United Nations Convention to Combat
'United Nations Convention to Combat Desertification (UNCCD) was established in 1994 to protect and restore
Desertification? (2016) our land and ensure a safer, just, and more sustainable future. It aims to
1. It aims to promote effective action through promote effective action through innovative national programmes and
innovative national programs and supportive supportive international partnerships.
international partnerships. Statement 2 is not correct: The implementation of the UNCCD is
2. It has a special/particular focus on South Asia organized around these five regional implementation annexes: Africa,
and North Africa regions, and its Secretariat Asia, Latin America and the Caribbean (LAC), Northern Mediterranean,
facilitates the allocation of major portion of Central and Eastern Europe. There is no special/particular focus on South
financial resources to these regions. Asia and North Africa regions.
3. It is committed to bottom-up approach, Statement 3 is correct: UNCCD is particularly committed to a bottom-up
encouraging the participation of local people in approach, encouraging the participation of local people in combating
combating the desertification. desertification and land degradation.

Select the correct answer using the code given


below:
(a) 1 only
(b) 2 and 3 only
(c) 1 and 3 only
(d) 1,2 and 3

TELEGRAM - UPSC PRELIMS WITH NEELESH


Learn ETHICS, ESSAY, SOCIOLOGY, CSAT BY VISITING THE CHANNEL (INTEGRATED MARATHON FOR 2024)
https://t.me/UPSCPrelimsWithNeelesh ©Copyright For Queries Whatsapp - 9310161970
6 The term 'Intended Nationally Determined b INDC, or Intended Nationally Determined Contribution, is a climate
Contributions' is sometimes seen in the news in action plan to cut emissions and adapt climate impacts. Each Party to the
the context of (2016) Paris Agreement is required to establish an INDC and update it every five
(a) pledges made by the European countries to years. INDCs are submitted every five years to the, UNFCCC secretariat.
rehabilitate refugees from the war-affected Middle Hence, option (b) is the correct answer.
East.
(b) plan of action outlined by the countries of the
world to combat climate change.
(c) capital contributed by the member countries in
the establishment of Asian Infrastructure
Investment Bank
(d) plan of action outlined by the countries of the
world regarding Sustainable Development Goals.

7 Consider the following pairs: (2016) c Pair 1 is not correctly matched: Cartagena protocol is a supplementary
Terms sometimes Their origin agreement to the Convention on Biological Diversity. It is an international
seen in News treaty governing the movements of living modified organisms (LMOs)
1. Annex-I Countries Cartagena resulting from modern biotechnology from one country to another.
Protocol Whereas, Annex-l Countries are related to the Kyoto protocol. Annex l
2.Certified Emissions Nagoya countries are
Reductions Protocol those countries that are mandated to make legally-binding emission cuts
3.Clean Kyoto under the Kyoto
Development Protocol Protocol.
Mechanism
Pair 2 is not correctly matched: The Nagoya protocol on Access to Genetic
Which of the pairs given above is/are correctly Resources and the Fair and Equitable Sharing of Benefits Arising from
matched? their Utilization (ABS) to the Convention on Biological Diversity is a
(a) 1 and 2 only supplementary agreement to the Convention on Biological Diversity.
(b) 2 and 3 only Whereas, Certified emission reductions (CERs) are related to the Kyoto
(c) 3 only protocol CERs are electronic certificates issued for greenhouse gas
TELEGRAM - UPSC PRELIMS WITH NEELESH
Learn ETHICS, ESSAY, SOCIOLOGY, CSAT BY VISITING THE CHANNEL (INTEGRATED MARATHON FOR 2024)
https://t.me/UPSCPrelimsWithNeelesh ©Copyright For Queries Whatsapp - 9310161970
(d) 1, 2 and 3 emission reductions from clean development mechanism (CDM) project
activities.

Pair 3 is correctly matched: Kyoto Protocol operationalizes the United


Nations Framework Convention on Climate Change by committing
industrialized countries and economies in transition to limit and reduce
greenhouse gas (GHG) emissions in accordance with agreed individual
targets. Clean Development Mechanism (CDM) is one of the flexibility
mechanisms defined in the Kyoto protocol that provides for emissions
reduction projects which generate Certified Emission Reduction units
which may be traded in emissions trading schemes. Hence,
option (c) is the correct answer.

8 Consider the following statements: (2023) c Statement 1 is correct: Under Section 41(1) of the Biological Diversity Act,
1. In India, the Biodiversity Management 2002, every local body in the State shall constitute a Biodiversity
committees are key to the realization of the Management Committee within its area of jurisdiction for the purpose of
objectives of Nagoya Protocol. promoting conservation, sustainable use and documentation of biological
2. The Biodiversity Management committees have diversity including preservation of habitats, conservation of land races,
important functions in determining access and folk varieties & cultivars, domesticated stocks and breeds of animals and
benefit sharing, including the power to levy micro-organisms and chronicling of knowledge relating to biological
collection fees on the access of biological resources diversity. The Nagoya Protocol on Access to Genetic Resources and the
within its jurisdiction. Fair and Equitable Sharing of Benefits Arising from their Utilization to the
Convention on Biological Diversity, also known as the Nagoya Protocol on
Which of the statements given above is/are Access and Benefit Sharing is a 2010 supplementary agreement to the
correct? 1992 Convention on Biological Diversity. It aims to share the benefits that
(a) 1 only arise from the utilization of genetic resources fairly and equitably thus
(b) 2 only BMCs have important role to play in the implementation of the provisions
(c) Both 1 and 2 of the Nagoya Protocol.
(d) Neither 1 nor 2

TELEGRAM - UPSC PRELIMS WITH NEELESH


Learn ETHICS, ESSAY, SOCIOLOGY, CSAT BY VISITING THE CHANNEL (INTEGRATED MARATHON FOR 2024)
https://t.me/UPSCPrelimsWithNeelesh ©Copyright For Queries Whatsapp - 9310161970
Statement 2 is correct: The main function of the BMC is to prepare
People's Biodiversity Register (PBR) in consultation with local people. The
Register shall contain comprehensive information on availability and
knowledge of local biological resources, their medicinal or any other use
or any other traditional knowledge associated with them. The BMC may
levy charges by way of collection fees from any person for accessing or
collecting any biological resource for commercial purposes from areas
falling within its territorial jurisdiction
9 Consider the following statements: (2023) a Statement 1 is correct: This statement states that carbon markets are
Statement-I: Carbon markets are likely to be one of likely to be one of the most widespread tools in the fight against climate
the most widespread tools in the fight against change. This statement is generally true. Carbon markets, also known as
climate change. emissions trading systems or cap-and-trade systems, are mechanisms that
Statement-II: Carbon markets transfer resources put a price on carbon emissions. They create economic incentives for
from the private sector to the State. reducing emissions by allowing entities to buy and sell carbon credits. This
helps to create a market-based approach to reducing greenhouse gas
Which one of the following is correct in respect of emissions and has been adopted by many countries and regions as a
the above statements? strategy to mitigate climate change.
(a) Both Statement-I and Statement-II are correct
and Statement-II is the correct explanation for Statement 2 is correct: Carbon markets are a mechanism designed to
Statement-I reduce greenhouse gas emissions by putting a price on carbon. They
(b) Both Statement-I and Statement-II are correct create a market for buying and selling carbon credits, which represent the
and Statement-II is not the correct explanation for right to emit a certain amount of greenhouse gases. In carbon markets,
Statement-I companies or entities that emit fewer greenhouse gases than their
(c) Statement-I is correct but Statement-II is allotted limit can sell their excess credits to those who exceed their limit.
incorrect This allows for a more efficient allocation of emission reduction efforts.
(d) Statement-I is incorrect but Statement-II is While carbon markets involve financial transactions and can transfer
correct. resources from the private sector to the state. Instead, they facilitate the
exchange of emission allowances between entities, which can include
both private and public entities.

TELEGRAM - UPSC PRELIMS WITH NEELESH


Learn ETHICS, ESSAY, SOCIOLOGY, CSAT BY VISITING THE CHANNEL (INTEGRATED MARATHON FOR 2024)
https://t.me/UPSCPrelimsWithNeelesh ©Copyright For Queries Whatsapp - 9310161970
10 With reference to 'Agenda 21.', sometimes seen in a Statement 1 is correct: Agenda 21 is a non- binding action plan of the
the news, consider the following statements: United Nations with regard to sustainable development. Agenda 21 is a
(2016) comprehensive plan of action to be taken globally, nationally and locally
1. It is a global action plan for sustainable by organizations of the United Nations System, Governments, and Major
development. Groups in every area in which human impacts on the environment.
2. It originated in the World Summit on Sustainable Statement 2 is not correct: Agenda 21 was established at the 1992 United
Development held in Johannesburg in 2002. Nations Conference on Environment and Development, or "Earth
Which of the statements given above is / are 5ummit", in Rio de Janeiro, Brazil.
correct?

(a) 1 only
(b) 2 only
(c) Both 1 and 2
(d) Neither 1 nor 2

11 Consider the following statements: (2016) b Club of Rome is a global think tank that deals with a variety of
1. The Sustainable Development Goals were first international political issues.
proposed in 1972 by a global think tank called the The first report of the Club of Rome was the famous "The limits to
'Club of Rome’ growth". The report did talk about the idea of sustainability, but it didn't
2. The Sustainable Development Goals have to be propose Sustainable Development Goals.
achieved by 2030. The United Nations Conference on Sustainable Development - or Rio+20
– took place in Rio de Janeiro, Brazil on 20-22 June, 2012. ln Rio 2012,
Which of the statements given above is / are member states decided to launch a process to develop a set of
correct? Sustainable Development Goals (SDGs) which will build upon the
(a) 1 only Millennium Development Goals.
(b) 2 only
(c) Both 1 and 2
(d) Neither 1 nor 2

TELEGRAM - UPSC PRELIMS WITH NEELESH


Learn ETHICS, ESSAY, SOCIOLOGY, CSAT BY VISITING THE CHANNEL (INTEGRATED MARATHON FOR 2024)
https://t.me/UPSCPrelimsWithNeelesh ©Copyright For Queries Whatsapp - 9310161970
12 Which one of the following is associated with the b Montreal Protocol is associated with the issue of control and phasing out
issue of control and phasing out of the use of of the use of ozone-depleting substances.
ozone-depleting substances? (2015)
(a) Bretton Woods Conference
(b) Montreal Protocol
(c) Kyoto Protocol
(d) Nagoya Protocol

13 With reference to the lnternational Union for b Statement 1 is not correct: lnternational Union for Conservation of
Conservation of Nature and Natural Resources Nature and Natural Resources (IUCN) is a membership Union composed
(IUCN) and the Convention on International Trade of both government and civil society organizations. IUCN is not a UN
in Endangered Species of Wild Fauna and Flora organ; rather it has UN Official observer status granted to it by the UN
(CITES), which of the following statements is/are General Assembly in 1999.
correct? (2015) Statement 2 is correct: Working with many partners and supporters, IUCN
1. IUCN is an organ of the United Nations and CITES implements a large and diverse portfolio of conservation projects
is an international agreement between worldwide. These projects combine the latest science with traditional
governments. knowledge of local communities to work to reverse habitat loss, restore
2. IUCN runs thousands of field projects around the ecosystems and improve people's well-being.
world to better manage natural Statement 3 is correct: CITES (the Convention on lnternational Trade in
environments. Endangered Species of Wild Fauna and Flora) is an international
3. CITES is legally binding on the States that have agreement between governments that aims to ensure that international
joined it, but this Convention does not take the trade in specimens of wild animals and plants does not threaten the
place of national laws. survival of the species. Although CITES is legally binding on the Parties - in
other words they have to implement the Convention - it does not take the
Select the correct answer using the code given place of
below: national laws. Rather it provides a framework to be respected by each
(a) 1 only Party, which has to adopt its own legislation to ensure that CITES is
(b) 2 and 3 only implemented at the national level.
(c) 1 and 3 only
(d) 1, 2 and 3
TELEGRAM - UPSC PRELIMS WITH NEELESH
Learn ETHICS, ESSAY, SOCIOLOGY, CSAT BY VISITING THE CHANNEL (INTEGRATED MARATHON FOR 2024)
https://t.me/UPSCPrelimsWithNeelesh ©Copyright For Queries Whatsapp - 9310161970
14 The "Red Data Books" published by the b The International Union for Conservation of Nature (IUCN) Red List of
International Union for Conservation of Nature Threatened Species, also known as the IUCN Red List or Red Data Book,
and Natural Resources (IUCN) contain lists of: founded in 1964, is the world's most comprehensive inventory of the
(2011) global conservation status of biological species.[1] It uses a set of precise
1. Endemic plant and animal species present in the criteria to evaluate the extinction risk of thousands of species and
biodiversity hotspots. subspecies. These criteria are relevant to all species and all regions of the
2. Threatened plant and animal species. world. With its strong scientific base, the IUCN Red List is recognized as
3. Protected sites for conservation of nature and the most authoritative guide to the status of biological diversity. A series
natural resources in various countries. of Regional Red Lists are produced by countries or organizations, which
assess the risk of extinction to species within a political management unit.
Select the correct answer using the codes given
below:
(a) 1 and 3 only
(b) 2 only
(c) 2 and 3 only
(d) 3 only
15 Invasive species Specialist Group' (that develops a The Invasive Species Specialist Group (ISSG) is a global network of
Global Invasive Species Database) belongs to scientific and policy experts on invasive species, organized under the
which one of the following organizations? (2023) auspices of the Species Survival Commission (SSC) of the International
(a) The International Union for Conservation of Union for Conservation of Nature (IUCN). Hence option (a) is the correct
Nature answer.
(b) The United Nations Environment Programme
(c)The United Nations World Commission for
Environment and Development.
(d) The World Wide

16 Regarding "carbon credits", which one of the d Carbon credit is a tradable permit or certificate that provides the holder
following statements is not correct? (2011) of the credit the right to emit one ton of carbon dioxide or an equivalent
(a) The carbon credit system was ratified in of another greenhouse gas.
conjunction with the Kyoto Protocol.
TELEGRAM - UPSC PRELIMS WITH NEELESH
Learn ETHICS, ESSAY, SOCIOLOGY, CSAT BY VISITING THE CHANNEL (INTEGRATED MARATHON FOR 2024)
https://t.me/UPSCPrelimsWithNeelesh ©Copyright For Queries Whatsapp - 9310161970
(b) Carbon credits are awarded to countries or Option (a) is correct: United Nations Intergovernmental Panel on Climate
groups that have reduced greenhouse gases Change (IPCC) developed a carbon credit proposal to reduce worldwide
below their emission quota. carbon emissions in a 1997 agreement known as the Kyoto Protocol.
(c) The goal of the carbon credit system is to limit Option (b) is correct: A country earns carbon credits if it reduces
the increase of carbon dioxide emission. greenhouse gases below its emission quota.
(d) Carbon credits are traded at a price fixed from Option (c) is correct: The main goal for the creation of carbon credits is
time to time by the United Nations Environment the reduction of emissions of carbon dioxide and other greenhouse gases
Program me. from industrial activities to reduce the effects of global warming. One
carbon credit allows for the emission of one tonne of Carbon dioxide or
the equivalent in other greenhouse gases. Carbon credits can be traded
on both the public and private markets.
Option (d) is not correct: There is no such mechanism by the United
Nations Environment Programme to fix the price of carbon credits from
time to time. The price depends on the supply and demand in the
markets.

17 With reference to 'WaterCredit', consider the c Statements 1 and 3 are correct: WaterCredit Initiative is a powerful
following statements: (2020) solution and the first to put microfinance tools to work in the water and
1. It puts microfinance tools to work in the water sanitation sector. WaterCredit helps bring small loans to those who need
and sanitation sector. access to affordable financing and expert resources to make household
2. It is a global initiative launched under the aegis water and toilet solutions a reality.
of the World Health Organization and The World Statement 2 is not correct - It has been launched by Water.org, a global
Bank. nonprofit organization working to bring water and sanitation to the world.
3. It aims to enable the poor people to meet their How 'WaterCredit' works: . A region is identified where people need
water needs without depending on subsidies. access to water and sanitation and that is ready for a microfinance
Which of the statements given above are correct? solution. Partnering with selected institutions is done to provide
(A) 1 and 2 Only affordable financing for water and sanitation to families in need. These
(b) 2 and 3 Only microfinance partners establish water and sanitation loans in their
(c) 1 and 3 Only portfolio of offerings. People in need use these small, affordable loans to
(d) 1, 2 and 3 put a tap or toilet in their homes and access local resources to do the
TELEGRAM - UPSC PRELIMS WITH NEELESH
Learn ETHICS, ESSAY, SOCIOLOGY, CSAT BY VISITING THE CHANNEL (INTEGRATED MARATHON FOR 2024)
https://t.me/UPSCPrelimsWithNeelesh ©Copyright For Queries Whatsapp - 9310161970
work. Every repaid loan can be lent to another family in need of safe water
or sanitation.

18 The 'Common Carbon Metric', supported by UNEP a Common Carbon Metric is a protocol for measuring energy use and
has been developed for (2021) reporting greenhouse gas (GHG) emissions from building operations. It is
(a) assessing the carbon footprint of building developed by the United Nations Environment Program's Sustainable
operations around the world. Buildings & Climate Initiative (UNEP-SBCI).
(b) enabling commercial farming entities around the Carbon footprint is the amount of greenhouse gas that is emitted as a
world to enter carbon emission trading. result of the production and use of services and products that affect the
(c) enabling governments to assess the overall earth and cause pollution and change in the climate. Hence, option (a) is
carbon footprint caused by their countries. the correct answer.
(d) assessing the overall carbon footprint caused by
the use of fossil fuels in a unit time

19 BioCarbon Fund Initiative for Sustainable Forest d The BioCarbon Fund Initiative for Sustainable Forest Landscapes (ISFL) is
Landscapes' is managed by the: a multilateral facility. It is supported by donor governments and managed
(a) Asian Development Bank by the World Bank. Hence option (d) is the correct answer.
(b) International Monetary Fund
(c) United Nations Environment Programme
(d) World Bank

20 With reference to an organization known as c Statement 1 is correct: BirdLife lnternational is a global partnership of
'BirdLife lnternational', which of the following non-governmental organizations that promotes the conservation of birds
statements is/are correct? (2015) and their habitats.
1. lt is a Global Partnership of Conservation
Organizations. Statement 2 is not correct: The concept of 'biodiversity hotspots' was
2. The concept of 'biodiversity hotspots' originated introduced by Norman Myers in 1988. Conservation lnternational (a
from this organization. nonprofit environmental organization) adopted Myers’ hotspots as its
3. ]t identifies the sites known/ referred to as institutional blueprint in 1989.
'lmportant Bird and Biodiversity Areas'.
TELEGRAM - UPSC PRELIMS WITH NEELESH
Learn ETHICS, ESSAY, SOCIOLOGY, CSAT BY VISITING THE CHANNEL (INTEGRATED MARATHON FOR 2024)
https://t.me/UPSCPrelimsWithNeelesh ©Copyright For Queries Whatsapp - 9310161970
Select the correct answer using the code given Statement 3 is correct: Birdlife lnternational has identified and
below: documented more than 13,000 important Bird and Biodiversity Areas
(a) 1 only (lBAs) which are places of global significance for the conservation of birds
(b) 2 and 3 only and other biodiversity.
(c) 1 and 3 only
(d) 1, 2 and 3

21 With reference to an initiative called "The c ln March 2007, G8+5 environment ministers called for a global initiative
Economics of Ecosystems and Biodiversity (TEEB)" to study the economics of biodiversity loss. So, the TEEB initiative was
which of the following statements is/are correct? launched under the leadership of Pavan Sukhdev.
(2016)
1. lt is an initiative hosted by UNEP, IMF and World Statement 1 is not correct: This major international initiative is funded by
Economic Forum. the European Commission, Germany, the United Kingdom, Norway, the
2. lt is a global initiative that focuses on drawing Netherlands and Sweden, and managed by the United Nations
attention to the economic benefits of Environment Programme (UNEP) as part of its Green Economy lnitiative
biodiversity. (GEI).
3. lt presents an approach that can help decision- Statement 2 is correct: lt seeks to draw attention to the global economic
makers recognize, demonstrate and capture the benefits of biodiversity, to highlight the growing costs of biodiversity loss
value of ecosystems and biodiversity. and ecosystem degradation.
Statement 3 is correct: lt aims to achieve the above goal by following a
Select the correct answer using the code given structured approach to valuation that helps decision-makers recognize
below: the wide range of benefits provided by ecosystems and biodiversity,
(a) 1 and 2 only demonstrate their values in economic terms and suggest how to use
(b) 3 only those values in decision-making.
{c) 2 and 3 only
(d) 1, 2 and 3
22 Consider the following statements in respect of b Statement 1 is not correct: TRAFFIC (Trade Records Analysis of Flora and
Trade Related Analysis of Fauna and Flora in Fauna in Commerce), the Wildlife Trade Monitoring Network, is a
Commerce (TRAFFIC): (2017) global non-governmental organisation monitoring the trade in wild
animals and plants that focuses on biodiversity and sustainable
TELEGRAM - UPSC PRELIMS WITH NEELESH
Learn ETHICS, ESSAY, SOCIOLOGY, CSAT BY VISITING THE CHANNEL (INTEGRATED MARATHON FOR 2024)
https://t.me/UPSCPrelimsWithNeelesh ©Copyright For Queries Whatsapp - 9310161970
1. TRAFFIC is a bureau under United Nations development. It was originally created in 1976 as a specialist group of the
Environment Programme (UNEP). Species Survival Commission of the International Union for Conservation
2. The mission of TRAFFIC is to ensure that trade in of Nature (IUCN), and evolved into a strategic alliance of the World Wide
wild plants and animals is not a threat to the Fund for Nature (WWF) and the IUCN.
conservation of nature.
Statement 2 is correct: lt aims to ensure that trade in wild plants and
Which of the above statements is/are correct? animals is not a threat to the conservation of nature.
(a) 1 only
(b) 2 only
(c) Both 1 and 2
(d) Neither 1 nor 2

23 "Climate Action Tracker" which monitors the a Climate Action Tracker is an independent scientific analysis that tracks
emission reduction pledges of different countries government climate action and measures it against the globally agreed
is (2022) Paris Agreement aim of "holding warming well below 2"C, and pursuing
(a) Database created by coalition of research efforts to limit warming to 1.5'C." The Climate Action Tracker is a
organization collaboration of two organizations, Climate Analytics and New Climate
(b) Wing of "International Panel of Climate Change" Institute. Lt has been providing this independent analysis to policymakers
(c) Committee under "United Nations Framework since 2009. Hence, option (a) is the correct answer.
Convention on Climate Change".
(d) Agency promoted and financed by the United
Nations Environment Programme and World Bank.

24 What is Greenhouse Gas Protocol? (2016) a The Greenhouse Gas Protocol (GHG Protocol) is the most widely used
(a) It is an international accounting tool for international accounting tool for government and business leaders to
government and business leaders to understand, understand, quantify, and manage greenhouse gas emissions. Hence
quantify and manage greenhouse gas emissions option (a) is the correct answer.
(b) It is an initiative of the United Nations to offer More about the GHG Protocol:
financial incentives to developing countries to

TELEGRAM - UPSC PRELIMS WITH NEELESH


Learn ETHICS, ESSAY, SOCIOLOGY, CSAT BY VISITING THE CHANNEL (INTEGRATED MARATHON FOR 2024)
https://t.me/UPSCPrelimsWithNeelesh ©Copyright For Queries Whatsapp - 9310161970
reduce greenhouse gas emissions and to adopt eco- It establishes comprehensive global standardized frameworks to measure
friendly technologies and manage greenhouse gas (GHG) emissions from private and public
(c) It is an inter-governmental agreement ratified by sector operations, value chains, and mitigation actions.
all the member countries of the United Nations to These standards were framed in 1998.The GHG Protocol is a partnership
reduce greenhouse gas emissions to specified levels between the World Resources Institute (WRI) and the World Business
by the year 2022 Council for Sustainable Development (WBCSD)
(d) It is one of the multilateral REDD+ initiatives
hosted by the World Bank

25 Consider the following statements: (2022) b Statement 1 is correct: The Climate Group is an international non-profit
1. "The Climate Group" is an international non- founded in 2003. It works with business and government leaders around
profit organization that drives climate action by the world to solve the issue of climate change. It aims at a world of net
building large networks and running them. zero carbon emissions by 2050, with greater prosperity for all.
2. The International Energy Agency in partnership Statement 2 is not correct: EP100 is a global initiative led by The Climate
with the Climate Group launched a global initiative Group and the Alliance to Save Energy.
"EPl00". Statement 3 is correct: EP100 brings together a group of over 120 energy-
3. EP100 brings together leading companies smart companies committed to using energy more productively, lowering
committed to driving innovation in energy greenhouse gas emissions, and accelerating a clean economy.
efficiency and increasing competitiveness while Statement 4 is correct: JSW Cement and Godrej Industries are some of
delivering on emission reduction goals. the Indian members of E100.
4. Some Indian companies are members of EP1 00. Statement 5 is not correct: Under2 Coalition is the largest global network
5. The International Energy Agency is the of states, regions, provinces and other subnational governments
Secretariat to the "Under12 Coalition”. committed to achieving net zero emissions by 2050 at the latest. Climate
Group is the Secretariat to the Under2 Coalition and works with
Which of the statements given above are correct? governments to accelerate climate action.
(a) 1,2,4 and 5
(b) 1,3 and 4 only
(c) 2,3 and 5 only
(d) 1,2,3, 4 and 5

TELEGRAM - UPSC PRELIMS WITH NEELESH


Learn ETHICS, ESSAY, SOCIOLOGY, CSAT BY VISITING THE CHANNEL (INTEGRATED MARATHON FOR 2024)
https://t.me/UPSCPrelimsWithNeelesh ©Copyright For Queries Whatsapp - 9310161970
26 Consider the following statements: (2017) b Statement 1 is not correct: The Climate and Clean Air Coalition is a
1. Climate and Clean Air Coalition (CCAC) to Reduce voluntary partnership of governments, intergovernmental organizations,
Short Lived Climate pollutants is a unique initiative businesses, scientific institutions and civil society organizations
of the G20 group of countries. committed to improving air quality and protecting the climate through
2. The CCAC focuses on methane, black carbon and actions to reduce short-lived climate pollutants.
hydrofluorocarbons.
Statement 2 is correct: CCAC focuses on short-lived climate pollutants
Which of the statements given above is/are correct? such as black carbon, methane, tropospheric ozone, and
(a) 1 only hydrofluorocarbons which are the most important contributors to the
(b) 2 only man-made global greenhouse effect after carbon dioxide, responsible for
(c) Both 1 and 2 up to 45% of current global warming.
(d) Neither 1 nor 2

27 With reference to 'Global Climate Change a Statement I is correct: The Global Climate Change Alliance (GCCA) is an
Alliance', which of the following statements is/are initiative of the European Union. Its overall objective is to build a new
correct? (2017) alliance on climate change between the European Union and the poor
1. It is an initiative of the European Union. developing countries that are most affected and that have the least
2. It provides technical and financial support to capacity to deal with climate change.
targeted developing countries to integrate
climate change into their development policies and Statement 2 is correct: GCCA also provides technical and financial
budgets. support to partner countries to integrate climate change into their
3. It is coordinated by World Resources Institute development policies and budgets, and to implement projects that
(WRI) and World Business Council for Sustainable address climate change on the ground, promoting climate-resilient, low
Development (WBCSD) emission development.

Select the correct answer using the code given Statement 3 is not correct: There is no role of the World Resources (WRI)
below: and World Business Council for Sustainable Development (WBCSD) in the
coordination of GCCA. Hence, a is the correct answer.
(a) 1 and 2 only
(b) 3 only
TELEGRAM - UPSC PRELIMS WITH NEELESH
Learn ETHICS, ESSAY, SOCIOLOGY, CSAT BY VISITING THE CHANNEL (INTEGRATED MARATHON FOR 2024)
https://t.me/UPSCPrelimsWithNeelesh ©Copyright For Queries Whatsapp - 9310161970
(c) 1 and 3 only
(d) 1,2 and 3

28 With reference to the 'New York Declaration on a Statements 1 and 2 are correct: New York Declaration on Forests is a
Forests'. Which of the following statements are political declaration that brings together governments, companies, and
correct? (2021) civil society actors including indigenous peoples organizations with the
1. It was first endorsed at the United Nations common aim of halving the loss of natural forests by 2020 and striving to
Climate Summit in 2O14. end it by 2030. It was endorsed at the UN Secretary General's Climate
2. It endorses a global timeline to end the loss of Summit in September 2014.
forests.
3. It is a legally binding international declaration. Statement 3 is not correct: It is voluntary and non-legally binding.
4. It is endorsed by governments, big companies Statement 4 is correct: The Declaration has been endorsed by dozens of
and indigenous communities. governments, over 30 of the world's biggest companies, and more than
5. India was one of the signatories at its inception. 50 influential civil society and indigenous organizations.

Select the correct answer using the code given Statement 5 is not correct: India was not one of the signatories at its
below: inception. India objected to the declaration as it interlinks trade to climate
{al 1,2 and 4 only change and forest issues. Besides India, Argentina, Mexico, Saudi Arabia
(b) 1,3 and 5 only and South Africa are the only G20 countries that did not sign the
(c) 3 and 4 only declaration.
(d) 2 and 5 only

29 Under Ramsar Convention, it is mandatory on the c Statement 1 is not correct: Each Contracting Party of Ramsar Convention
part of the Government of India to protect and shall promote the conservation of wetlands and waterfowl by establishing
conserve all the wetlands in the territory of India. nature reserves on wetlands, whether they are included in the List or not,
2. The Wetlands (Conservation and Management) and provide adequately for their wardening. It is not mandatory on the
Rules, 2010 were framed by the Government of part of the Government of India to protect and conserve all the wetlands
India based on the recommendations of Ramsar in the territory of India.
Convention. Statement 2 is not correct: The Wetlands (Conservation and
Management) Rules, 2010 was notified by the Ministry of Environment
TELEGRAM - UPSC PRELIMS WITH NEELESH
Learn ETHICS, ESSAY, SOCIOLOGY, CSAT BY VISITING THE CHANNEL (INTEGRATED MARATHON FOR 2024)
https://t.me/UPSCPrelimsWithNeelesh ©Copyright For Queries Whatsapp - 9310161970
3. The Wetlands (Conservation and Management) and Forest to ensure better conservation and management and to
Rules, 2010 also encompass the drainage area or prevent degradation of existing wetlands in India. However, the rules
catchment regions of the wetlands as determined were not framed on the basis of recommendations of the Ramsar
by the authority. Convention.

Which of the statements given above is/are correct? Statement 3 is correct: The Wetlands (Conservation and Management)
(a) 1 and 2 only Rules, 2010 also encompass the drainage area or catchment regions of
(b) 2 and 3 only the wetlands as determined by the authority. Under section 2 (g) of rules,
(c) 3 only Wetland includes marsh, pen, lake, lagoons, estuaries, drainage area and
(d) 1, 2 and 3 catchment region of wetlands.

30 Consider the following statements: (2021) d Statement 1 is not correct: Tree city certification is given by Arbor Day
Statement 1: The United Nations Capital foundation (Nonprofit Conservation And Education organization) and FAO
Development Fund (UNCDF) and the Arbor Day (Food and Agriculture Organization).
Foundation have recently recognized Hyderabad Statement 2 is correct: To receive recognition, a town or city must meet
as 2020 Tree City of the World. five core standards by Arbor foundation for which Hyderabad City has
Statement 2: Hyderabad was selected for the pledged its commitment by meeting five programme standards:
recognition for a year following its commitment to 1. The city has a written statement by city leaders delegating
grow and maintain the urban forests. responsibility for the care of trees within the municipal boundary.
2. The city has in place a law or an official policy that governs the
Which one of the following is correct in respect of management of forests and trees.
the above statements? 3. The city has an updated inventory or assessment of the local tree
(a) Both Statement 1 and Statement 2 are correct resource so that an effective long-term plan for planting, care, and
and Statement 2 is the correct explanation of removal of city trees can be established.
Statement 1. 4. The city has a dedicated annual budget for the routine implementation
(b) Both Statement 1 and Statement 2 are correct of the tree management plan.
and Statement 2 is not the correct explanation of 5. The city holds an annual celebration of trees to raise awareness among
Statement 1 residents and to acknowledge citizens and staff members who carry out
(c) Statement 1 is correct but Statement 2 is not the city tree programme. Hence, option (d) is the correct answer.
correct
TELEGRAM - UPSC PRELIMS WITH NEELESH
Learn ETHICS, ESSAY, SOCIOLOGY, CSAT BY VISITING THE CHANNEL (INTEGRATED MARATHON FOR 2024)
https://t.me/UPSCPrelimsWithNeelesh ©Copyright For Queries Whatsapp - 9310161970
(d) Statement 1 is not correct but Statement 2 is
correct

31 The Millennium Ecosystem Assessment describes c The Millennium Ecosystem Assessment (MEA) was called for by United
the following major categories of ecosystem Nations Secretary-General Kofi Annan in 2000. Initiated in 2001, the
services-provisioning, supporting, regulating, objective of the MEA was to assess the consequences of ecosystem
preserving, and cultural. Which one of the change for human well-being and the scientific basis for action needed to
following is supporting service? (2012) enhance the conservation and sustainable use of those systems and their
(a) Production of food and water contribution to human well-being.
(b) Control of climate and disease
(c) Nutrient cycling and crop pollination Defining the ecosystem: It can be visualized as a functional unit of nature,
(d) Maintenance of diversity where living organisms interact among themselves and also with the
surrounding physical environment. Ecosystem varies greatly in size from
a small pond to a large forest or a sea.
Four Types of Ecosystem Services: The Millennium Ecosystem
Assessment (MEA) identified four major categories of ecosystem services:
provisioning, regulating, cultural, and supporting services.
Provisioning Services: A provisioning service is any type of benefit to
people that can be extracted from nature.
- Fruits, vegetables, trees, fish, and livestock are available to us as direct
products of ecosystems.
- Along with food other types of provisioning services include drinking
water, timber, wood fuel, natural gas oils, plants that can be made into
clothes and other materials, and medicinal benefits.
Regulating Services: A regulating service is the benefit provided by
ecosystem processes that moderate natural phenomena. Regulating
services include pollination, decomposition, water purification, erosion
and flood control, and carbon storage and climate regulation.
Cultural Services: A cultural service is a non-material benefit that
contributes to the development and cultural advancement of people. It
TELEGRAM - UPSC PRELIMS WITH NEELESH
Learn ETHICS, ESSAY, SOCIOLOGY, CSAT BY VISITING THE CHANNEL (INTEGRATED MARATHON FOR 2024)
https://t.me/UPSCPrelimsWithNeelesh ©Copyright For Queries Whatsapp - 9310161970
includes how ecosystems play a role in local, national, and global cultures;
the building of knowledge and the spreading of ideas; creativity born from
interactions with nature (music, art, architecture); and recreation.
Supporting Services: Ecosystems themselves couldn't be sustained
without the consistency of underlying natural processes (supporting
services). It includes photosynthesis, nutrient cycling, the creation of soils
and the water cycle. Hence option c is the correct answer.

32 Consider the following statements regarding 'Earth c Statement 1 is not correct: Earth Hour is one of the world's largest
Hour' (2014) grassroots movements for the environment. It was started in 2007 by
1. It is an initiative of UNEP and UNESCO. "WWF (World Wildlife Fund for Nature) and partners as a symbiotic light-
2. It is a movement in which the participants out event in Sydney to raise awareness of climate change.
switch off the lights for one hour on a certain day
every year. Statement 2 is correct: It is held every year on the last Saturday of March
3. It is a movement to raise the awareness about and encourages individuals, households, communities and businesses to
the climate change and the need to save the turn off their non-essential lights for one hour as a symbol for their
planet. commitment to the planet.
Which of the statements given above is/are correct? Statement 3 is correct: It is a symbolic event to raise awareness about
climate change and the need to save the planet.
(a) 1 and 3 only
(b) 2 only
(c) 2 and 3 only
(d) 1,2 and 3

TELEGRAM - UPSC PRELIMS WITH NEELESH


Learn ETHICS, ESSAY, SOCIOLOGY, CSAT BY VISITING THE CHANNEL (INTEGRATED MARATHON FOR 2024)
https://t.me/UPSCPrelimsWithNeelesh ©Copyright For Queries Whatsapp - 9310161970
Our other initiatives
1. PRELIMS AND INTEGRATED MENTORSHIP (Details on our telegram channel)
2. FREE ETHICS AND ESSAY WRITING PRACTICE (TELEGRAM CHANNEL – ER NEELESH AIR
442 UPSC_CSE2021)
3. FREE SOCIOLOGY ANSWER FRAMEWORK (TELEGRAM CHANNEL - SOCIOLOGY WITH
NEELESH)
4. FREE CSAT MARATHON – TELEGRAM CHANNEL - CSAT WITH NEEELESH
5. Learn Ethics with Neelesh (Every 11th of the Month)
6. FREE CSAT VIDEOS ON YOUTUBE CHANNEL – CIVIL SERVICES WITH NEELESH
UPCOMING PLANNARS
INTEGRATED MARATHON FOR 2024 AND 2025 (ON TELEGRAM CHANNEL – UPSC PRELIMS
WITH NEELESH)
ESSAY COURSE
SOCIOLOGY LECTURES AND MUCH MORE
ETHICS VIDEO COURSE
Visit our website – neeleshair442.com, www.csetopper.com for best PYQs and test series

TELEGRAM - UPSC PRELIMS WITH NEELESH


Learn ETHICS, ESSAY, SOCIOLOGY, CSAT BY VISITING THE CHANNEL (INTEGRATED MARATHON FOR 2024)
https://t.me/UPSCPrelimsWithNeelesh ©Copyright For Queries Whatsapp - 9310161970
TELEGRAM - UPSC PRELIMS WITH NEELESH
Learn ETHICS, ESSAY, SOCIOLOGY, CSAT BY VISITING THE CHANNEL (INTEGRATED MARATHON FOR 2024)
https://t.me/UPSCPrelimsWithNeelesh ©Copyright For Queries Whatsapp - 9310161970

You might also like